全部 1- 101- 201- 301- 401- 501- 601- 701- 801- 901- 最新50  

面白い問題おしえて〜な 十五問目

1 名前:132人目の素数さん:2009/01/05(月) 08:00:00
面白い問題、教えてください


2 名前:132人目の素数さん:2009/01/05(月) 08:04:00
過去ログ
http://www3.tokai.or.jp/meta/gokudo-/omoshi-log/
まとめwiki
http://www6.atwiki.jp/omoshiro2ch/

1 http://cheese.2ch.net/test/read.cgi/math/970737952/
2 http://natto.2ch.net/test/read.cgi/math/1004839697/
3 http://science.2ch.net/test/read.cgi/math/1026218280/
4 http://science.2ch.net/test/read.cgi/math/1044116042/
5 http://science.2ch.net/test/read.cgi/math/1049561373/
6 http://science.2ch.net/test/read.cgi/math/1057551605/
7 http://science2.2ch.net/test/read.cgi/math/1064941085/
8 http://science3.2ch.net/test/read.cgi/math/1074751156/
9 http://science3.2ch.net/test/read.cgi/math/1093676103/
10 http://science4.2ch.net/test/read.cgi/math/1117474512/
11 http://science4.2ch.net/test/read.cgi/math/1134352879/
12 http://science6.2ch.net/test/read.cgi/math/1157580000/
13 http://science6.2ch.net/test/read.cgi/math/1183680000/
14 http://science6.2ch.net/test/read.cgi/math/1209732803/


3 名前:132人目の素数さん:2009/01/05(月) 18:06:50
立ってる立ってる

4 名前:132人目の素数さん:2009/01/05(月) 19:08:45
1乙

それにしてもまた下4桁0か

5 名前:132人目の素数さん:2009/01/06(火) 14:16:53
sage

6 名前:132人目の素数さん:2009/01/08(木) 12:25:20
自作問題。群論っぽいけど、群論の理論は全くと言っていいほど使わない。
使っても出来るかもしれんが。

問:Snをn対称群とする。τ∈Snを互換の積で表すことを考える。
このとき必要な互換の個数の最小値をS(τ)と書くことにする。
i=1,2,…,nに対してXi:={τ^k(i)|k∈Z}とおく。
集合X1,X2,…,Xnのうち、(集合として)異なるものの個数を
C(τ)と書くとき、S(τ)=n−C(τ)となることを示せ。

7 名前:132人目の素数さん:2009/01/08(木) 12:26:59
用語の訂正。
× n対称群
○ n次対称群

8 名前:132人目の素数さん:2009/01/08(木) 22:37:38
sage

9 名前:132人目の素数さん:2009/01/09(金) 19:53:55
転載、しかしよく意味わからん

1000 名前: 132人目の素数さん [sage] 投稿日: 2009/01/09(金) 15:43:35 
平面上に半径の等しい4つの円がある。 
この4つの円は互いに重なることなく自由に動くものとする。 
このとき、4つの異なる円周によって作られる領域の最大値を求めなさい。 

10 名前:132人目の素数さん:2009/01/09(金) 20:15:13
アステロイドみたいな形の部分の面積?

11 名前:132人目の素数さん:2009/01/10(土) 07:29:15
閉曲線によって囲まれた部分の面積の最大値を求めよ。ってことじゃね?

…てか、意味は分かるんだが、言葉で伝えにくいな。

12 名前:132人目の素数さん:2009/01/10(土) 07:59:19
図を書けばよかったんでないかい

13 名前:132人目の素数さん:2009/01/10(土) 14:29:59
(1)xを実数の定数とするとき
a[n+1]=a[n]/2+x/a[n]
で定義される数列a[n]が収束するa[1]の条件を求めよ

(2)x,yを実数の定数とするとき
a[n+1]=a[n]/y+x/a[n]
で定義される数列a[n]が収束するa[1]の条件を求めよ

14 名前:132人目の素数さん:2009/01/10(土) 16:08:25
>>11
円を隙間なく並べたときにできる三角や四角(ただし辺は直線ではなく円弧)の空間ってこと?

15 名前:132人目の素数さん:2009/01/11(日) 11:48:06
http://imepita.jp/20090111/414300
これで伝わるはず。

16 名前:132人目の素数さん:2009/01/11(日) 12:13:31
なんかパッと見で
1変数の関数で表せそうな気がするんだが

17 名前:132人目の素数さん:2009/01/11(日) 12:37:47
これ見ると4つの円が他の2つの円と接触していて、かつ
正方形の頂点上に円の中心が来るような並び方が最大になるようにしか見えない・・・
何処に引っ掛けがあるんだろう。

18 名前:132人目の素数さん:2009/01/11(日) 14:29:42
>>17
証明するのが大変なんじゃないの?

19 名前:132人目の素数さん:2009/01/11(日) 15:06:33
こんな解答はどうか?

円の半径は1としてよい。一辺の長さが2である任意の平行四辺形Tを考える。
Tの頂点を中心とした半径1の円を各頂点に対して書く(4つ書ける)。
[これら4つの円]∪T の面積をS=S(T)とすれば、Tを動かしたときのS(T)の
最大値が求める値である。S(T)=[Tの面積]+[半径1の円の面積]=[Tの面積]+π
であるから、Tの面積の最大値を求めればよい。Tが正方形のとき
最大であることは明らか。

20 名前:132人目の素数さん:2009/01/11(日) 15:21:19
>>19
ちょうど同似たような回答書こうとしてたら先こされた
ちなみに俺は問題の面積は[これら4つの円以外の部分]∧Tと思ってた(否定の記号の出し方がわからんかった)
まあ大して変わらんが
多分あってるんじゃないかな
ちなみにある円の中心角をθとすれば(0<θ<=π/2)
T=4sinθだからθ=π/2で最大値だね

21 名前:132人目の素数さん:2009/01/11(日) 16:41:17
改題でもして遊ぼうか

平面上に半径rの3つの円と半径nrの円がある。
この4つの円は互いに重なることなく自由に動くものとする。
このとき、4つの異なる円周によって作られる領域の最大値を求めなさい。
ただし、n及びrは実数で、非0

22 名前:132人目の素数さん:2009/01/11(日) 16:41:57
×非0 ○非負

23 名前:132人目の素数さん:2009/01/11(日) 20:12:42
円の個数をM個にしたバージョンも考えられるな。
>>19と同じやり方で解けちゃうけど。

24 名前:132人目の素数さん:2009/01/11(日) 20:36:46
線形台数の本をゲットした。

「人口の推移を予測するのに、行列を用いた
人口推移モデルが使われる。
ある地域には、100万人が住んでおり、
そのうち都市に60万人、
農村に40万人が住んでいる。
毎年、、都市では9割がそのまま住みつづけ、
1割が農村に移住する。

農村では8割が住みつづけ、
2割が都市に転出する。
この場合、人口の推移はある点(均衡点)に向かい、
そこで安定する。」

って書いてあるんだが、本当?
へぇ〜って感じなんだけど。

25 名前:132人目の素数さん:2009/01/11(日) 21:24:19
均衡点での都市の人口をx万人とすると農村の人口は(100-x)万人で、
都市残留+上京=都市人口で立式して
0.9x+0.2(100-x)=x
0.7x+20=x
20=0.3x
x=200/3
こうか?本当は均衡点の存在証明をこれの前にやらないかんのだろう
ロジスティック式の極端に簡単な例に見えた
詳しくやったことないから知らんけど

26 名前:132人目の素数さん:2009/01/11(日) 22:10:27
>>13 (1)

・x>0 のときは 定符号。σ=Sgn(a[1]) とおく。
 σ・a[n]/√(2x) = b[n] とおくと b[n] >0, 漸化式は
 b[n+1] = (1/2){b[n] + 1/b[n]},
 ・b[1] =1 のときは b[n] =1  (収束)
 ・b[1] ≠1 のとき
  b[n] = 1/tanh(2^(n-1)・α), (n>1) → 1 (収束)
  ここに α = (1/2)log((1+b[1])/|1-b[1]|),

・x=0 のとき a[n] = a[1]/{2^(n-1)}, → 0 (収束)

・x<0 のとき a[n]/√(2|x|) = b[n] とおくと、漸化式は
 b[n+1] = (1/2){b[n] - 1/b[n]},  
 b[n] = 1/tan(2^(n-1)・β),  (非収束)
 ここに β= arctan(1/b[1]),

27 名前:132人目の素数さん:2009/01/12(月) 00:39:22
>>24
列ベクトルで人口を表す。
初期状態は(60,40)
次の年は、(54+8,32+6)=([0.9,0.2],[0.1,0.8])(60,40)
同様に考えるとn年目は、([0.9,0.2],[0.1,0.8])^n(60,40)であり、n→∞での収束点が均衡点である。
計算は勝手にやって下さい。

28 名前:132人目の素数さん:2009/01/12(月) 01:03:55
暇だし計算してみた
a[n]=(1/10)^n * ([9,2],[1,8]) * (60 40)
として
A=([9,2],[1,8])
とすると、
A^n=(1/3) * ([7^n+2*10^n,-2*7^n+2*10^n],[-7^n+10^n,2*7^n+10^n])より
lim[n→∞] a[n]=1/3 * ([2,2],[1,1]) * (60,40) = (200/3,100/3)
よって均衡点では都市におよそ67万人、農村におよそ33万人

29 名前:132人目の素数さん:2009/01/12(月) 02:46:22
もしかしてこの板って行列式の話できないの?(MathMLとかないから)

30 名前:132人目の素数さん:2009/01/12(月) 04:53:01
28だが、行列の固有ベクトルを作る時と逆行列作る時とに使う奴だよね?って程度の認識。
あれって他に何か利用法あるのかな。

ちなみに当方高校生なのでググれば出る程度の語彙の範囲で説明頼みたい

31 名前:132人目の素数さん:2009/01/12(月) 05:21:52
>>30
誰にレスしてるの。29 だったら 29 の文意を大幅に取り違えてるぞ。

32 名前:132人目の素数さん:2009/01/12(月) 11:37:32
>>31
29他がこれから話す事が理解できなかったら結構残念なんで
このスレに来てる人に教えてもらえたらいいなという駄目人間やってました

そろそろセンター対策しないと・・・

33 名前:29:2009/01/13(火) 16:44:03
漏れが言ったのは、
MathMLとかTeXとかが使えれば、
行列とか行列式とかをWEBで表記できるから
話ができるけど、そうでないなら、
わざわざ画像にして貼り付けなければならないから、
行列とか行列式の話がとってもやりにくいね!

って意味だったのだけれど。

34 名前:28:2009/01/13(火) 23:05:30
やっと把握したorz
文字見て理解できるから問題ないんじゃないかな
それより他に作って解く問題があるし

どうでもいいけどクラスにTeXをテックスと発音する人が居るんだが、放っておいてもいいかな

35 名前:132人目の素数さん:2009/01/14(水) 00:17:33
恥をかけばすぐに治るので、そのままにしておくとよい

36 名前:132人目の素数さん:2009/01/18(日) 15:40:03
雑誌でみかけた。

四角形ABCDで角ABD=39度、角DBC=24度、角BCA=18度、角ACD=57度のとき、
角BDA=15度となることを証明せよ

できんのか?

37 名前:132人目の素数さん:2009/01/18(日) 21:58:06
CADとADBとの角度がそれぞれ出てこない。足したら42度なのはすぐ分かるんだが。

他の角度はBAC=99,BDC=81

38 名前:132人目の素数さん:2009/01/19(月) 00:40:35
点A,Bの座標を(0,0),(1,0)としてDの座標を計算すると
D=((tan39°+ αtan42° + β)/(tan39°+tan42°),(tan39°((α-1)tan42° + β))/(tan39°+tan42°))
である。
ただし上の式でαとβは
α=tan63° /(tan63°-tan81°)
β=tan63°tan81°/(tan63°-tan81°)
とする。
これで直線ADの傾きが出るのでtan15°と一致する事を示せばよい。
あとは誰か任せた

39 名前:132人目の素数さん:2009/01/19(月) 00:59:05
良く考えたら一致するかどうか確かめるのは
tan15°じゃなくてtan(15°+∠ABD)だった。

40 名前:132人目の素数さん:2009/01/19(月) 19:28:43
>>38
その方針でやってみたら、機械的に求まったけど、膨大な計算になった

AD の傾き a とすると
a = tan(∠BDA + ∠ABD) = tan(∠BDA + 39°)
∴ tan(∠BDA) = (a - tan(39°)) / (1 + a*tan(39°))
>>38 より
tan(∠BDA) = t39 (t81 - t63) (t39 + t42)
/ (t63(t39 + t42) + t81(t63 - t39) + (t39)^2 t81 (t42 + t63))
  (t39 は tan(39°) の意味、他も同様)

t39 = (4-3√3-2√5+√15 + (-1+2√3+√5)√(5-2√5)) / 2
t42 = (√3+√15 - (3+√5)√(5-2√5)) / 2
t63 = -1 + √5 + √(5-2√5)
t81 = 1 + √5 + (2+√5)√(5-2√5)
より
tan(∠BDA) = (297-170√3-131√5+76√15 + (-94+55√3+44√5-25√15)√(5-2√5))
/ (84-43√3-34√5+21√15 + (-23+16√3+13√5-6√15)√(5-2√5))

分母、分子に
84-43√3-34√5+21√15 - (-23+16√3+13√5-6√15)√(5-2√5)
を掛けて分母の √(5-2√5) を消す

tan(∠BDA)
= (89-53√3-41√5+23√15) / (19-17√3-13√5+5√15)
= 1/(2+√3)
= tan(15°)

41 名前:132人目の素数さん:2009/01/19(月) 20:14:58
>>40に敬礼

42 名前:132人目の素数さん:2009/01/19(月) 20:24:27
では次は初等幾何で。

43 名前:132人目の素数さん:2009/01/19(月) 22:18:47
任意の三角形ABCにおいて、各辺を底辺とする三つの正三角形を△ABCの外部に描く。
三つの正三角形の重心を結んでできる三角形は、やはり正三角形であることを示せ

44 名前:132人目の素数さん:2009/01/19(月) 22:29:33
>>36の証明を初等幾何で。

45 名前:132人目の素数さん:2009/01/20(火) 03:33:59
>>43
ポルナレフの定理でつね

46 名前:132人目の素数さん:2009/01/21(水) 14:39:16
2項係数C(n,k)=n!/{k!(n-k)!}が奇数であるための条件は、
n,kを2進数で表したとき、各位のビットについてそれぞれ
kのビットが1ならばnのビットも1であることである。
これを証明せよ。

47 名前:132人目の素数さん:2009/01/22(木) 19:24:56
a[1]=b[1]=1
a[n+1]=a[n]-b[n]
b[n+1]=a[n]+3*b[n]
を満たす数列a[n],b[n]がある。
(1)a[n]+b[n]を求めよ
(2)a[n],b[n]を求めよ

48 名前:132人目の素数さん:2009/01/22(木) 19:35:17
扉が左右2つあり一方は天国、もう一方は地獄への扉です
扉の前に門番が3人(3人ともどちらが天国への行き先か知っている)がいます
門番の内、
一人はいつでも正直な事を言う正直者
一人はいつでも嘘をつく嘘つき
一人は適当に「はい」か「いいえ」で答えます
3人の門番は見た目では区別できません
また、門番への質問は合わせて2回までしかできません
さらにこの3人の門番は「はい」か「いいえ」の2つの返事しかしません
この門番たちにどの様な質問をすればどちらが天国への扉かが判るでしょうか?

49 名前:132人目の素数さん:2009/01/22(木) 19:59:47
1回でわかる気がするといってみるテスト

50 名前:132人目の素数さん:2009/01/22(木) 20:18:27
京都+大阪=東京
これを証明せよ

51 名前:132人目の素数さん:2009/01/22(木) 20:31:06
質問できる相手は1回にひとりなんじゃないの?

52 名前:132人目の素数さん:2009/01/22(木) 21:33:23
まずランプの精を呼び出して質問の数を増やしてもらう

53 名前:132人目の素数さん:2009/01/23(金) 00:58:35
>>47宿題乙
答えだけ書いとくね。
(1)2^n
(2)an=(2-n)2^(n-1),bn=n*2^(n-1)

54 名前:132人目の素数さん:2009/01/23(金) 02:53:41
>>48
3人の門番をa、b、cと呼ぶ。
【1回目の質問】:aに対し、「あなたは、『bは適当に答える門番(以下Tと呼ぶことにする)
ですか?』と聞かれたら、『はい』と答えますか?」
aがTでないのなら、(仮に嘘つきであっても)「この質問へのaの返事が『はい』」⇔「bはT」
が成り立つことに注意すると、
「この質問へのaの返事が『はい』」ならば、aがTか、さもなくばbがTなので、
cはTでない。
「この質問へのaの返事が『いいえ』」ならば、aがTか、さもなくばbがTでないので、
bはTでない。
【2回目の質問】:1回目の質問で、Tでないと判明した門番に対し、
「あなたは、『右が天国への扉ですか?』と聞かれたら、『はい』と答えますか?」

55 名前:132人目の素数さん:2009/01/23(金) 07:56:09
>>54
それだと、問題文に加えて
「3人の門番は、互いに、誰がどんな性格であるか知っている」
という条件が必要になるのでは。

56 名前:132人目の素数さん:2009/01/24(土) 06:54:44
2つとも天国に行きますか?ー>いいえ、はい、?



57 名前:132人目の素数さん:2009/01/24(土) 07:02:15
みぎは天国に行きますか?ー>いいえ、はい、? ー>左が正解



58 名前:132人目の素数さん:2009/01/24(土) 10:13:04
http://www.sciencedaily.com/news/computers_math/mathematics/

59 名前:132人目の素数さん:2009/01/24(土) 17:39:42
F(x)=x+(1-x)+(1,0)
F(0)=0+1+(1,0) 0+1+0->2=F 0+1+1->0=T
F(1)=1+0+(1,0) 1+0+1->T=1 1+0+0->T=1


60 名前:132人目の素数さん:2009/01/25(日) 04:50:44
>>48の問題をクリアするには2つを満足させる必要がある。

1) どちらか一方の門が天国または地獄への門であることを確定させる。
2) 1)を確定させるための質問をする相手が、適当に答えるひと以外であることがわかっている。

1)はまあ当然として、2)については、質問の答がランダムな「はい/いいえ」でないための保障として必要。
もし尋ねた相手が適当に答えるひとだったら、その答には価値のある情報を含まないので。


正直、嘘吐きの定義からすると、はい、いいえ、で答えられない質問には答えてくれなさそうだ。
しかし、もうひとりは、どんな質問にでも「はい」か「いいえ」という答を返すようである。
たとえば、「あなたの年齢はいくつですか? 」という質問に、正直と嘘吐きは答えないが
適当なひとは、「はい/いいえ」のどちらかを答えるだろう。
先の質問は、質問した相手が適当に答える人なのかそうでない人なのかを特定することができそうだ。

そこから先は、もうよくある問題と同じ。


61 名前:132人目の素数さん:2009/01/25(日) 12:56:25
基本は、1回目で適当に答える人ではない人を1人探すこと。

1回目の質問:正直者は「はい」、嘘つきは「いいえ」と答えるような質問をする。
(「あなたは門番ですか」等)
すると、適当に答える人はそのどちらかと答えが重複するので、重複しなかった人を対象に
2回目の質問をすればOK。

1回目でこの手のパズルでよくあるようなややこしい質問をしようと考えるから
かえって難しく見えるが、少数決に気付けば実はシンプルな問題。

62 名前:132人目の素数さん:2009/01/25(日) 14:39:12
>>61
一度に複数の人に質問できるなら、そんなめんどくさいことしなくても
54の方法で「『右が天国への扉ですか?』と訊かれたら『はい』と答えますか」と訊けば49の言うように1度で済むじゃんよ


63 名前:132人目の素数さん:2009/01/25(日) 15:03:35
>>62
そうか。通りがかりで適当に答えたら失敗したな。すまん。

では改訂版
1回目:「質問した相手が嘘つきか正直者だと仮定すると、残りの2人のうち嘘つきか正直者を特定できる」ような質問をする。
たとえばaに「『bが嘘つきであるか、またはcが正直者である』という命題は正しいですか?」ときく。
答えが「はい」なら、bが嘘つきまたは正直者、「いいえ」なら、cが嘘つきまたは正直者。
その結果嘘つきまたは正直者のどちらかであると判明した1人に2回目の質問をすればいい。

結局>>54と同じだな。(そこで終わってた話題であったか。重ねてすまん)
>>55の条件はやっぱり必須だろう。

64 名前:132人目の素数さん:2009/01/25(日) 15:23:25
> >>55の条件はやっぱり必須だろう。

>>60で否定的に解決してるじゃん。

65 名前:54:2009/01/25(日) 20:55:05
>>60はある意味とんちクイズみたいな答えだと思ったけど、
確かに問題文には、「はいかいいえで必ず答えられる
質問しかしてはいけない」(※)って断りはないね。

ところで>>55の条件が絶対要るって示せるんだろうか?
(正確には、それプラス上の※の条件)

古典的な命題論理みたいな議論でいけるかと思ったけど
頭が混乱してきてわからんくなった・・・。

xに対して真偽が問える命題の集合を、
Hx:xは正直者
Lx:xは嘘つき
Tx:xは適当に答える
R:天国への扉は右
とその論理演算で書けるもの全体として、
命題Pに対する返事が仮に「いいえ」だったら、
(Hx∧¬P)∨(Lx∧P)∨T

・・・みたいな感じで。



66 名前:132人目の素数さん:2009/01/25(日) 23:57:54
>>50
KYOTO+OSAKA=TOKYO
これに数字を当てはめて完成させればいいの?

67 名前:65:2009/01/26(月) 00:45:46
一応きちんと書いてみる。
(この定式化で果たして合っているんだろうか??当方論理学には不慣れなので・・・)
門番x(=a,b,cと名づける)に対して真偽が問える命題(>>65)のことを
P(x)などと書くことにして、
示すべきは、

『・Xa,Xb,Xc(X=H,L,T)の排他的論理和が真
・Hx,Lx,Tx(x=a,b,c)の排他的論理和が真



∀x,∀P(x),(

((Hx∧P)∨(Lx∧¬P)∨Tx

∀y,∀Q(y),(
(¬((Hy∧Q)∨(Ly∧¬Q)∨Ty→R)∧¬(((Hy∧Q)∨(Ly∧¬Q)∨Ty)→¬R)))
∨(¬((Hy∧¬Q)∨(Ly∧Q)∨Ty→R)∧¬(((Hy∧Q)∨(Ly∧¬Q)∨Ty)→¬R))))



((Hx∧¬P)∨(Lx∧P)∨Tx

∀y,∀Q(y),(
(¬((Hy∧Q)∨(Ly∧¬Q)∨Ty→R)∧¬(((Hy∧Q)∨(Ly∧¬Q)∨Ty)→¬R)))
∨(¬((Hy∧¬Q)∨(Ly∧Q)∨Ty→R)∧¬(((Hy∧Q)∨(Ly∧¬Q)∨Ty)→¬R))))

)』


68 名前:132人目の素数さん:2009/01/26(月) 00:54:52
>>67
論理パズルは真理表書いたら瞬殺だよ

69 名前:132人目の素数さん:2009/01/26(月) 01:02:29
あー、括弧やら否定やらがいろいろ狂ってる・・・
心の目で読んでほしいけど根本的に間違ってるかも。

70 名前:132人目の素数さん:2009/01/26(月) 07:28:01
常に「はい」と言う嘘吐き。


71 名前:132人目の素数さん:2009/01/27(火) 16:14:12
http://myhome.cururu.jp/gersdorffite/blog/article/71002515020
http://www.iis.it-hiroshima.ac.jp/~ohkawa/math/math_problem_all.htm#076
w

72 名前:132人目の素数さん:2009/01/27(火) 18:02:34
嘘吐きは、常に嘘を言うひとではなく、嘘も言うひとのことだ。

73 名前:132人目の素数さん:2009/01/27(火) 20:46:27
そのとおり。
そして正直者は、常に正しいことを言うひとではなく、正しいことも言うひとのことだ。


74 名前:132人目の素数さん:2009/01/27(火) 20:47:01
それは違うww

75 名前:132人目の素数さん:2009/01/28(水) 01:16:07
箱が2009個並んでいて、
どれか1つに「当たり」と書かれた紙が入っている。
その箱より右の箱には「左」と書かれた紙が、
左の箱には「右」と書かれた紙が入っている。
(つまり、当たりの箱がある方向を示している。)
これらの箱から1つずつ選んで開けていき、
当たりの箱がどの箱か特定できたら終了とする。
開ける箱の個数の期待値を最小にするには
どうすればよいか。

76 名前:132人目の素数さん:2009/01/28(水) 01:17:05
二分探索でやれば最小だろうな。

77 名前:132人目の素数さん:2009/01/28(水) 01:23:25
最悪ケースの最小値なら二分探索だろうけど、平均もそうなんだろうか。

78 名前:132人目の素数さん:2009/01/28(水) 15:33:05
>>75
頼むから、
「どの箱があたりである確率も1/2009である」ということを
条件として書いてくれ。

79 名前:132人目の素数さん:2009/01/28(水) 21:47:55
>>78
条件:どの箱があたりである確率も1/2009である

80 名前:132人目の素数さん:2009/01/28(水) 22:10:29
>>79
頼むから、
馬鹿にしないでくれ。

81 名前:132人目の素数さん:2009/01/29(木) 01:28:11
どの箱があたりである確率も1/2009である

82 名前:132人目の素数さん:2009/01/29(木) 01:46:43
「当たりの箱がどの箱か特定できたら終了」
ってのがポイントかな?
「当たりの箱を開けたら終了」
ってのと微妙に違う。
箱の個数が少ない場合を計算してみると、
つねに真ん中の箱を開ければいいってわけじゃないみたい。
誰かプログラム組んで計算してほしいんだけど。

83 名前:132人目の素数さん:2009/01/29(木) 02:16:00
986番目くらいがいいな

84 名前:132人目の素数さん:2009/01/29(木) 12:49:22
>>82
> 箱の個数が少ない場合を計算してみると、 
> つねに真ん中の箱を開ければいいってわけじゃないみたい。

kwsk

85 名前:132人目の素数さん:2009/01/29(木) 13:53:49
素朴な二分法が、ベストではない、最も、単純なケースは箱が5個の場合
最初に真ん中(3番目)の箱を開ける場合
1/5の確率で1回で特定可能、4/5の確率で2回で特定可能、従って、9/5回
最初に、2番目の箱を開ける場合
2/5の確率で1回で特定可能、3/5の確率で2回で特定可能、従って、8/5回


箱の数が19個の場合

10−5,15−2,7,12,17−3,8,13,18
(一回目は10番目、2回目は5番目か15番目、...の意)
これは、当たりが3,4,8,9,13,14,18,19の何れかであった場合は、4回目の開封を
行わなければならない方法である。

しかし、次のような方法もある。
8-4,12-2,6,10,16-14,18
これは、当たりが、13,14,15,17,18,19の何れかであった場合だけ、4回目の開封を
行わなければならない方法である。

86 名前:132人目の素数さん:2009/01/31(土) 23:45:58
お金を預けると1年毎に1/2の確率で2倍か2分の1になる銀行があります(端数もきちんと計算する)

(1)この銀行に1万円あずけた時,10年後は平均で何円になってるでしょうか
(2)同じ銀行が10店舗あって、それぞれに1年,1万円ずつ預けたら1年後の期待値は?
(3)同じ銀行が無限店舗あって,10年間で手持ちの1万円をできるだけ増やすのに最適な預け方は?
また,その時の期待値は?

87 名前:132人目の素数さん:2009/02/01(日) 00:21:46
n年後に最初の金が2^(2k-n)倍になっている確率はnCk/2^n
(1)は二項定理で計算して(5/4)^10(万円)
(2)(3)は線形性から預け方によらずn年後に(5/4)^n倍になる

88 名前:132人目の素数さん:2009/02/01(日) 01:31:19
>>86
「最適な」の定義にもよるな。
期待値ではなく「得する可能性」を上げたいなら、分散投資するけど

89 名前:132人目の素数さん:2009/02/03(火) 13:13:32
p1〜pnの中から最大のものを取り出す関数max(p1,p2,p3,….,pn) と
最小のものを取り出す関数min(p1,p2,p3,….,pn)を使って 
5個の要素から中央値(昇順または降順に並べた際の3番目の数値)
を取り出す関数 mid(p1,p2,p3,p4,p5) を作れ。

90 名前:132人目の素数さん:2009/02/03(火) 19:27:10
min(max(pi,pj,pk)[1≦i<j<k≦5])かなあ。

91 名前:132人目の素数さん:2009/02/03(火) 19:41:22
> [1≦i<j<k≦5]
ココの意味がわからん。 繰り返しの指定とかなの? 


92 名前:132人目の素数さん:2009/02/03(火) 19:49:26
ijkが条件を満たす
max(pi,pj,pk)を全部という意味じゃないか?

書き下すなら
max(p1,p2,p3),max(p1,p2,p4),max(p1,p2,p5),max(p1,p3,p4),max(p1,p3,p5),
max(p1,p4,p5),max(p2,p3,p4),max(p2,p3,p5),max(p2,p4,p5),max(p3,p4,p5)
になる。

93 名前:90:2009/02/03(火) 19:57:42
フォローサンクス。min[1≦i<j<k≦5](max(pi,pj,pk))と書くべきだったか。

94 名前:132人目の素数さん:2009/02/03(火) 20:27:56
なるほど、テクニカルで面白いな。
数式処理ソフトで使えそう。

95 名前:132人目の素数さん:2009/02/03(火) 21:07:30
5->3->1

96 名前:132人目の素数さん:2009/02/04(水) 02:09:55
加減乗除と絶対値記号のみを使って
5個の要素から中央値(昇順または降順に並べた際の3番目の数値)
を取り出す関数 mid(p1,p2,p3,p4,p5) を作れ。


97 名前:132人目の素数さん:2009/02/04(水) 02:33:32
max(a,b)を作ればminも、n変数のmax,minも出来る。

max(a,b)=(|a-b|+a+b)/2 以下関数合成を繰り返す

98 名前:132人目の素数さん:2009/02/04(水) 02:49:30
n個の数の中で大きいほうから数えてm(<n)番目の数を与える関数を
f_{n,m} (p1, ........., p_n)とする。
max{x,y}、min{x,y}から関数合成を行うことによって
決して構成できないようなf_{n,m}は存在するか。
無いならそのことを示し、あるならば反例を与え、そのnとmに対して
関数合成によって決してf_{n,m}にならないことを示せ

99 名前:132人目の素数さん:2009/02/04(水) 04:04:08
次の等式を証明せよ。
nHr=Σ[m=1,n]{Σ[l=1,m]…(Σ[j=1,k]j) }
※右辺のΣの数はr-1個

↓例
8H3=Σ[m=1,8](Σ[l=1,m]l)
5H4=Σ[m=1,5]{Σ[l=1,m](Σ[k=1,l]k)}

100 名前:132人目の素数さん:2009/02/04(水) 18:03:40
>>98
任意個の変数のmax、minは容易に作れるから、
あとは>>90と同じやりかたでf_{n,m}も作れる

101 名前:132人目の素数さん:2009/02/04(水) 18:30:55
>>99
直感的には、n個から重複を許してr個とるわけだから、
まず種類1をn-m個、次に種類2をn-m-l個、・・・ととるのだと考えれば成り立つ。

nH1,1Hn=1に対しては成立。(r=1のときはΣ[m=1,n]{Σ[l=1,m]…(Σ[j=1,k]j) }=1
だと解釈)
n+rに関する帰納法。この和がn+r-1のときまで成り立っているなら、
nHr=(n+r-1)C(r-1)=(n+r-2)C(r-1)+(n+r-2)C(r-2)=(n-1)Hr+nH(r-1)
=Σ[m=1,n-1]{Σ[l=1,m]…(Σ[j=1,k]j) } + Σ[l=1,n]…(Σ[j=1,k]j)
=Σ[m=1,n]{Σ[l=1,m]…(Σ[j=1,k]j) }
よりこの和がn+rのときも成り立つ。


102 名前:101:2009/02/04(水) 18:48:56
miss!
>nHr=(n+r-1)C(r-1)=(n+r-2)C(r-1)+(n+r-2)C(r-2)=(n-1)Hr+nH(r-1)
nHr=(n+r-1)Cr=(n+r-2)Cr+(n+r-2)C(r-1)=(n-1)Hr+nH(r-1)

103 名前:101:2009/02/04(水) 20:32:20
あ、
>r=1のときはΣ[m=1,n]{Σ[l=1,m]…(Σ[j=1,k]j) }=1 だと解釈
r=1のときはΣ[m=1,n]{Σ[l=1,m]…(Σ[j=1,k]j) }=n だと解釈
しないとうまくいかないな。
こうしておけば帰納法はうまく進むので問題なし。

104 名前:132人目の素数さん:2009/02/07(土) 11:31:41
>>99
多項式(x+1)^(n+1)-1を考える。二項定理から(x+1)^(n+1)-1=Σ[k=0,n]C[n+1,k+1]x^(k+1) …(1)
(ここでC[n,k]は2項係数を表す。)一方、
(x+1)^(n+1)-1={(x+1)-1}Σ[k=0,n](x+1)^k=xΣ[k=0,n]Σ[l=0,k]C[k,l]x^l
右辺を整理すれば(x+1)^(n+1)-1=Σ[k=0,n]{Σ[l=k,n]C[l,k]}x^(k+1) …(2)
(1),(2)の係数を比較することでC[n+1,k+1]=Σ[l=k,n]C[l,k]を得る。
書き直せばC[n+k,k+1]=Σ[l=1,n]C[l+k-1,k] これより
Σ[j_1=1,n]Σ[j_2=1,j_1]…Σ[j_(r-1)=1,j_(r-2)]j_(r-1)
=Σ[j_1=1,n]Σ[j_2=1,j_1]…Σ[j_(r-1)=1,j_(r-2)]C[j_(r-1),1]
=Σ[j_1=1,n]Σ[j_2=1,j_1]…Σ[j_(r-1)=1,j_(r-3)]C[j_(r-2)+1,2]
=…
=Σ[j_1=1,n]C[j_1+r-2,r-1]
=C[n+r-1,r]
=H[n,r]

105 名前:132人目の素数さん:2009/02/07(土) 11:58:39
>>99
nHr = (r+1)H(n-1) は、次の非負整数解の個数と同じ
x_1 + x_2 + … + x_{r+1} = n-1.
そこで、
y_i = 1 + x_1 + x_2 + … + x_i (i = 1, 2, ..., r+1)
と換えると、
1 ≦ y_1 ≦ y_2 ≦ … ≦ y_r ≦ y_{r+1} = n.
これを数えると、題意の等式を得る。

106 名前:132人目の素数さん:2009/02/07(土) 15:58:51
http://www.sonnyradio.com/leapfrog.htm

107 名前:132人目の素数さん:2009/02/11(水) 21:17:45
自然数nについての不等式
(n^n)/(e^(n-1))≦n!≦(n^(n+1))/(e^(n-1))
を証明せよ。
ただしeはネイピアの数

108 名前:132人目の素数さん:2009/02/11(水) 23:35:25
>>107
n=1 のときは 等号成立。
n>1 のときは log(1+x) < x を使う。
 k・log(k) - (k-1)log(k-1) -1 = k・log(k) - (k-1){log(k) - log(k/(k-1))} -1
 = log(k) + (k-1)log(1 + 1/(k-1)) -1 < log(k) +1 -1 = log(k),
 (k+1)log(k) - k・log(k-1) -1 = (k+1)log(k) - k{log(k) + log((k-1)/k)} -1
 = log(k) - k・log(1 - 1/k) -1 > log(k) +1 -1 = log(k),
k=2,3,・・・,n について たす。
 n・log(n) - (n-1) < log(n!) < (n+1)log(n) - (n-1),

109 名前:132人目の素数さん:2009/02/13(金) 06:16:35
>48
1+1=2?
天国は右?

110 名前:132人目の素数さん:2009/02/13(金) 15:59:43
a[1]=1,a[2]=3,b[1]=1,b[2]=1
a[n+2]=(4n+2)a[n+1]+a[n]
b[n+2]=(4n+2)b[n+1]+b[n]
とするとき
lim[n→∞]a[n]/b[n]を求めよ

111 名前:132人目の素数さん:2009/02/15(日) 04:01:43
(改変転載)

一辺が1の正四面体Tが、厚さの無視できる壁に開いた
半径rの円形の穴を通過する。半径rの最小値をもとむ。

112 名前:132人目の素数さん:2009/02/15(日) 10:06:27
>>111
>もとむワロタ

外接円考えてr=1/√2じゃないの?

113 名前:132人目の素数さん:2009/02/15(日) 10:09:39
よく見たら正四面体か。
r=1/√3かな。

114 名前:132人目の素数さん:2009/02/15(日) 10:20:04
ばかw

115 名前:132人目の素数さん:2009/02/15(日) 10:41:24
(0,0,0),(1/√2,1/√2,0),(1/√2,0,1/√2),(0,1/√2,1/√2)
を結んだ正四面体を考えてr=1/2

116 名前:132人目の素数さん:2009/02/15(日) 20:06:12
転載

◆ わからない問題はここに書いてね 255 ◆
http://science6.2ch.net/test/read.cgi/math/1234188243/

215 名前:132人目の素数さん[sage] 投稿日:2009/02/15(日) 03:59:34
>>213>>214
1/2より小さくできると思う。
正四面体をOABCとし、OBとOC上にそれぞれM,Nを取ったとき、
△AMNの外接円の半径の最小値が答えになると予想。

ちなみにM,NをOB,OCの中点としたときは9/(8√11)≒0.34。
この付近に最小値を与える点があるはずなんだが、
式が爆発して手が着けられなくなった。


225 名前:132人目の素数さん[sage] 投稿日:2009/02/15(日) 15:26:47
>>215
なぁ、本当に0.34まで小さくなるか?

ちょっとやってみたんだが
O(0,0,0) A(Sqrt[3]/3,0,Sqrt[6]/3) B(Sqrt[3]/2,1/2,0) C(Sqrt[3]/2,-1/2,0)
としてOM:MC=s:1-s,ON:NB=t:1-tと置いて内積使って正弦定理を書き直して求めてみた。
http://www2.uploda.org/uporg2019178.jpg
どうもそんなに小さくならなさそうなんだが……俺なんか間違えたかな?

117 名前:116:2009/02/15(日) 20:08:33
上の 225 の外接円の半径 R は
R^2 = (1-s+s^2)(1-t+t^2)(s^2-st+t^2)/(3s^2-2st+3t^2 - 2st(s+t) + 3s^2t^2)
で、これは自分(≠225)も確認

ただし、△AMN が円に納まる条件は
min(R, (1/2)max(AM, AN, MN)) ≦ r
max のほうは鈍角三角形の場合

118 名前:132人目の素数さん:2009/02/15(日) 20:35:00
確か「幾何学の散歩道」か何かに、
一辺1の立方体に穴を開けて別の立方体を通したい。
通る立方体の一辺の長さの上限を求めよ、みたいな問題があって
1よりも何%か大きくなるとかなんとかで、結構意外な問題だったと思う。
手元にたぶん無いんでアレだが。

119 名前:132人目の素数さん:2009/02/15(日) 21:27:18
>>111
 r = 0.44780569665327156030029247262839…
らしいよ。
>>116-117
 漏れも そんなに小さくならなさそうだとオモタ.

http://science6.2ch.net/test/read.cgi/math/1158198794/284-286
数学セミナー

120 名前:132人目の素数さん:2009/02/15(日) 21:50:44
>>117
鈍角三角形ってなりえるかな?

121 名前:132人目の素数さん:2009/02/15(日) 22:30:53
(i)
xy平面上に4個の点O,A1,A2,A3がある。
Oから出発し、Oではない全ての点A1,A2,A3を一度だけ通りOに帰ってくるルートを考える。
最も長い距離を移動するルートをα、最も短い距離を移動するルートをβとしたときの
α/βの最大値を求めよ。


O(0,0) A1(0,1) A2(1,0) A3(1,1)とすると
α=2+2√2、β=4よりα/β=(1+√2)/2

(ii)
3次元ユークリッド空間上にn個の点O,A1,A2,A3,…,A[n-1]がある。(ただしn≧4)
Oから出発し、Oではない全ての点A1,A2,A3,…A[n-1]を一度だけ通りOに帰ってくるルートを考える。
最も長い距離を移動するルートをα、最も短い距離を移動するルートをβとしたときの
α/βの最大値はnの式で書けるか?書けないならその事をを示せ。

122 名前:132人目の素数さん:2009/02/15(日) 22:47:19

「面白い」の定義を数式を用いて述べなさい。


【配点 5 点】


123 名前:132人目の素数さん:2009/02/15(日) 22:59:27
面白い⊆興味深い

124 名前:132人目の素数さん:2009/02/15(日) 22:59:55
interesting と funny で区別とかするよね

125 名前:132人目の素数さん:2009/02/15(日) 23:02:51
数学でfunnyってどういうのだろう?
文体なんかでfunnyに見せるのであれば芸がないな

126 名前:132人目の素数さん:2009/02/16(月) 00:04:42
>>120
なる
自分は t=0 で s→0 にすると R が変な値になって気がついた

127 名前:132人目の素数さん:2009/02/16(月) 00:18:13
>>125
「点」「直線」「平面」を「机」「椅子」「ジョッキ」と言い換える

128 名前:132人目の素数さん:2009/02/16(月) 01:53:41
n枚のコインを全て裏向きにして円形に並べ、
そのうちの1枚をスタート地点としてそこにコマを置く。
この状態から、
(1)表向きになっているコインの枚数だけ時計回りにコマを進め、
(2)コマの位置にあるコインをひっくり返す
という操作を繰り返しおこなう。
ただし、最初の状態では表向きのコインが0枚なのでコマは動かさない。

全部のコインが表向きになるのは、nがどのような数のときか?

129 名前:132人目の素数さん:2009/02/16(月) 02:23:24
>>122
limit[white→tail](dog)

130 名前:132人目の素数さん:2009/02/16(月) 10:35:13
(俺用メモ)

>>117 の R が最小値を取るのは
s = t,
3s^3 - 6s^2 + 7s - 2 = 0
のとき

これを解いて
s = t
= (2 + (√43-4)^(1/3) - (√43+4)^(1/3)) / 3
= 0.39125971
のとき最小

このとき最小値は
R
= √(1 + (12+√43)(√43-4)^(1/3) - (12-√43)(√43+4)^(1/3)) / (6√2)
= 0.447805697

131 名前:132人目の素数さん:2009/02/16(月) 22:55:09
>>128

裏=○,表=●,コマの位置=☆,★
[○]→[★]
[○○]→[★○]→[●★]
[○○○]→[★○○]→[●★○]→[☆●○]→[○☆○]
[○○○○]→[★○○○]→[●★○○]→[●●○★]→[●●★●]
[○○○○○]→[★○○○○]→[●★○○○]→[●●○★○]→[●☆○●○]→[●○○☆○]
→[●○○○★]→[●★○○●]→[●●○○☆]→[●☆○○○]→[●○★○○]→[●○●○★]
→[●○☆○●]→[●○○○☆]→[☆○○○○]
以下、"→・・・→"(n個)でn手先を表すことにする
[○○○○○○]→→→[●●○★○○]→→[○●★●○○]→→[○●●☆○●]→→[●●●○★●]→[●●●★●●]
[○○○○○○○]→→→→[●●○●○○★]→→[●●○○○○☆]→→→[●○●○★○○]→→→[○○○★●○○]
→→[○★○●●●○]→→→→[○○○○☆○○]

以下同様に20枚まで調べていくと、全部表向きになった枚数は(カッコ内は手数)
1(1),2(2),4(4),6(10),8(8),12(44),16(16),20(3116)
また、それ以外の枚数で、全部裏にもどってしまったときの手数は
3(4),5(14),7(18),9(14),10(24),11(58),13(34),14(12),15(158),17(1430),18(792),19(2216)

もしやとおもって、32枚、64枚のときを調べてみるとそれぞれ32手、64手で全部表になった。
ここから「あるnがあってコインの枚数が2^nならば2^n手で全部表になる」と予想。
それ以上は分からないし証明も出来ない。

132 名前:132人目の素数さん:2009/02/17(火) 00:42:13
>>110
勘だけどeだと思う

133 名前:132人目の素数さん:2009/02/17(火) 04:46:53
>ここから「あるnがあってコインの枚数が2^nならば2^n手で全部表になる」と予想。
これは簡単。mod 2^nにおいてk(k+1)/2 (k=0,1,2,…,2^n−1)は全て異なる。

134 名前:132人目の素数さん:2009/02/20(金) 20:21:12
>>121の(i)って2になる気がする。
でもどうやって証明しよう。

135 名前:132人目の素数さん:2009/02/21(土) 09:51:02
問題というよりは質問にちかいのだけれども…

平面上にいて見える景色について考える。

球面上(曲率が正の平面)にいる場合、地平線は水平よりも少し下に周囲一周円を描いて見えるはず。
曲率が0の平面にいる場合、地平線は水平に無限のかなたに周囲一周見えるはず。
では曲率が負の平面に立っている場合、地平線はどういう形に見えるんだろう?


136 名前:132人目の素数さん:2009/02/21(土) 11:13:26
曲率が負なら球面の中みたいになるでしょ。

137 名前:132人目の素数さん:2009/02/21(土) 20:08:25
>>136
曲率が負というのはそういう意味じゃない。
いわゆる馬の鞍のような曲面が負の曲率を持つもの。

球面はどこをとっても同じ正の曲率だが、
どこをとっても一定の負の曲率の曲面は、そもそも3次元空間の中に
きれいに収めることはできない。

曲率は、その面が空間内でどういう形に収められているかとは
関係なく決まっているもので、たとえば紙を丸めてロール状にしても
その曲率は0で変わらない。なので、>>135の考えていることも
曲率を持ち出して議論すること自体ナンセンス。

138 名前:132人目の素数さん:2009/02/23(月) 08:31:02
なるほど

139 名前:132人目の素数さん:2009/02/23(月) 15:32:28
ナ、ナ、ナ、ナンセンス!

140 名前:132人目の素数さん:2009/02/23(月) 15:32:59
>>137
> どこをとっても一定の負の曲率の曲面は、そもそも3次元空間の中に
> きれいに収めることはできない。
> どこをとっても一定の負の曲率の曲面は、そもそも3次元空間の中に
> きれいに収めることはできない。
> どこをとっても一定の負の曲率の曲面は、そもそも3次元空間の中に
> きれいに収めることはできない。
> どこをとっても一定の負の曲率の曲面は、そもそも3次元空間の中に
> きれいに収めることはできない。
> どこをとっても一定の負の曲率の曲面は、そもそも3次元空間の中に
> きれいに収めることはできない。

141 名前:132人目の素数さん:2009/02/23(月) 17:10:16
>>140
できるの?

142 名前:132人目の素数さん:2009/02/23(月) 21:48:31
だまされるな、>>140>>137に恋心を抱いているだけだ

143 名前:132人目の素数さん:2009/02/23(月) 22:53:50
空間が無限なら収まるってはなしじゃないの

144 名前:132人目の素数さん:2009/02/24(火) 13:25:42
無限なら収まるの?

145 名前:132人目の素数さん:2009/02/27(金) 02:11:29
F(0)=0, F(1)=1, F(n)=F(n-1)+F(n-2)
で定まるフィボナッチ数列を考える。
正整数mに対し、F(n)がmの倍数となるような最小の正整数nをg(m)と定義する。
例えばg(1)=1, g(2)=3, g(3)=4である。
g(n)=nを満たすnはどのような数か?

146 名前:132人目の素数さん:2009/02/27(金) 02:55:08
面白いなー
なんで5と12なんだ

147 名前:132人目の素数さん:2009/02/28(土) 00:25:12
>>144
"擬球"でぐぐるよろし

148 名前:132人目の素数さん:2009/03/07(土) 19:55:11
なんか問題文自体は簡単に書けるけど中身は面白い問題キヴォンヌ

149 名前:132人目の素数さん:2009/03/07(土) 22:42:59
(1) 任意の実正方行列Aが高々4つの直交行列(Aに依存してよい)の線型結合で書けることを示せ。
(2) (1) の主張の4という数字はこれ以上小さくできないことを示せ。

150 名前:132人目の素数さん:2009/03/07(土) 23:23:07
はっ!まさか4色問題がらみ?
と直感だけで言ってみる。


151 名前:132人目の素数さん:2009/03/07(土) 23:28:21
>>145
n=5^kまたはn=12*5^kのときにg(n)=nになることは証明できた。
めんどいから書かんけど。

152 名前:132人目の素数さん:2009/03/08(日) 03:17:55
>>150
例えるなら,4色問題よりラグランジュの4平方和定理(全ての自然数は高々4つの平方数の和で表せる)の方が近いだろう。

153 名前:132人目の素数さん:2009/03/09(月) 16:12:53
>>99の等式って
Σ[k=1,n]kHr=nH(r+1)
と同値なのかな?

154 名前:132人目の素数さん:2009/03/11(水) 08:24:46
教養のない人間=獣を証明せよ

155 名前:132人目の素数さん:2009/03/11(水) 08:34:53
まずは教養のない人間と獣の定義を聞かせてもらおうか?


156 名前:132人目の素数さん:2009/03/11(水) 08:42:13
∀x{(x∈教養のない人間)→(x∈獣)}は納得できるが
∀x{(x∈獣)→(x∈教養のない人間)}はかなり無理のある定義しないと証明できないんじゃなかろか

157 名前:132人目の素数さん:2009/03/14(土) 15:16:09
>50 京都+大阪=東京 ,これを証明せよ

京都で買ったおたべと大阪で買ったたこやきは、東京のバナナだった。




158 名前:132人目の素数さん:2009/03/14(土) 22:36:51
>>157

「おたべ」は (株)おたべ〔京都〕 の
「§京都銘菓\おたべ 」 は (有)あど・おたべ〔京都〕 の  (4484724号)
「大阪新名物\たこ焼き\ようかん」は(有)黒須製餡所〔栃木・今市〕の  (4699568号)
「東京ばな奈」は (株)グレープストーン〔東京〕 の 登録商標でつ。。。

159 名前:132人目の素数さん:2009/03/20(金) 01:49:57
面白い解法があることを期待して転載
http://science6.2ch.net/test/read.cgi/math/1234113574/209

実数a,b,c,x,y,zが

ax+by+cz=1
ax^2+by^2+cz^2=2
ax^3+by^3+cz^3=6
ax^4+by^4+cz^4=24
ax^5+by^5+cz^5=120
ax^6+by^6+cz^6=720

を満たすとき、ax^7+by^7+cz^7の値を求めよ

160 名前:132人目の素数さん:2009/03/20(金) 03:28:42
ワクワク…、ワクワク…

161 名前:132人目の素数さん:2009/03/20(金) 06:06:47
ガウス-ラゲールの積分公式を求めるのと同じようにしてできる
(ax^6+by^6+cz^6=6! の代わりに a+b+c=1 とするとガウス-ラゲールそのもの)

関数 g(t), h(t) の内積を
(g(t), h(t)) ≡ ∫[0,∞] g(t) h(t) t e^(-t) dt
で定義する
(f(t), 1) = (f(t), t) = (f(t), t^2) = 0  …(1)
となる t の3次式 f(t) を求めると、定数倍を除いて
f(t) = t^3 - 12t^2 + 36t - 24
f(t) = 0 は相異なる3実根を持ち、それを x,y,z とする
({x,y,z} = {0.935822, 3.305407, 7.758770})

ax^n + by^n + cz^n = (t^(n-1), 1) (n = 1,2,3)  …(2)
となるように a,b,c を定めると、a,b,c,x,y,z は与条件を満たす

∵)
(t^(n-1), 1) = n!  …(3)
なので (2) より
ax^n + by^n + cz^n = n! (n = 1,2,3)
あとは ax^n + by^n + cz^n = n! (n = 4,5,6) を言えばよい
例えば n=5 のとき x^4 を f(x) で割った商を q(x) とすると
x,y,z は f(t) = 0 の根なので
ax^5 + by^5 + cz^5
= ax(x^4 - f(x)q(x)) + by(y^4 - f(y)q(y)) + cz(z^4 - f(z)q(z))
  x(x^4 - f(x)q(x)) は x,x^2,x^3 の線形結合(y,z についても同様)
  なので (2) を使って、
= (t^4 - f(t)q(t), 1) = (t^4,1) - (f(t), q(t))
  第1項に (3) を使い、q(t) は1次なので第2項に (1) を使って、
= 5!
n=4,6 のときも同様■

162 名前:132人目の素数さん:2009/03/20(金) 06:07:58
(続き)
同じようにして
ax^7 + bx^7 + cz^7
= ax^4(x^3-f(x)) + by^4(y^3-f(y)) + cz^4(z^3-f(z))
= 12a(x^6-3x^5+2x^4) + 12b(y^6-3y^5+2y^4) + 12c(z^6-3z^5+2z^4)
= 12 (t^5 - 3t^4 + 2t^3, 1)
= 12(6! - 3*5! + 2*4!)
= 4896

# a,b,c,x,y,z の一意性は言えてないけど

163 名前:132人目の素数さん:2009/03/20(金) 18:51:06
>>159
高校数学の範囲内の問題?

164 名前:132人目の素数さん:2009/03/20(金) 20:05:00
>>163
p[n] = ax^n+by^n+cz^n について、漸化式
p[n] = A p[n-1] + B p[n-2] + C p[n-3]
の問題に帰着できる。

165 名前:132人目の素数さん:2009/03/20(金) 21:26:51
>>161
蛇足だが・・・

 f(t) = t^3 - 12t^2 + 36t - 24 = (t-4)^3 -12(t-4) -8 = 16{4T^3 -3T -(1/2)},
ここに T = (t-4)/4,
 x = 4 + 4・cos( 7π/9) = 0.93582222752408785919042939777833・・・
 y = 4 + 4・cos(13π/9) = 3.3054072893322786045931334929227・・・
 z = 4 + 4・cos( π/9) = 7.7587704831436335362164371092989・・・

{a,b,c} は 次の多項式の根。
 g(u) = u^3 - (3/4)u^2 + (11/12^2)u - {1/(3・12^3)} = (u -1/4)^3 - (1/9)(u -1/4) -(1/81) = (2/81√3){4U^3 - 3U - (√3)/2},
 ここに U = {(3√3)/2}(u -1/4)
よって
 a = (1/4) + {2/(3√3)}cos( π/18) = 0.62905268086775253761255598397337・・・
 b = (1/4) + {2/(3√3)}cos(-11π/18) = 0.11835638545510051414429421693642・・・
 c = (1/4) + {2/(3√3)}cos( 13π/18) = 0.002590933677146948243149799090212・・・

166 名前:132人目の素数さん:2009/03/20(金) 21:42:04
>>164
特性多項式
 f(t) = t^3 -At^2 -Bt -C,   >>161
から出まつね。

167 名前:132人目の素数さん:2009/03/20(金) 22:33:05
中学三年の問題らしいよ

168 名前:132人目の素数さん:2009/03/21(土) 03:03:23
解法はありきたりだが結果が面白い問題ということで一つ。

数列I_nと関数列f_n(x)を次のように定義する。
I_n=∫[0,π/2]cos^(2n)(t)dt
f_n(x)=∫[0,π/2]cos(xt)cos^(2n)(t)dt (xは任意の実数)
(1)I_n,f_n(x)を計算せよ。
(2)任意の実数xについて lim[n→∞]f_n(x)/I_n=1
 が成り立つことを示せ。

169 名前:132人目の素数さん:2009/03/21(土) 03:44:35
>>165
その a,b,c,x,y,z が与えられた方程式を満たすのはいいとして、
逆に、与えられた方程式を満たす a,b,c,x,y,z が(並べ替えを除いて)
>>165 のものだけに限ることは言えるんだろうか

170 名前:132人目の素数さん:2009/03/21(土) 20:13:57
一意性もOKみたい

171 名前:132人目の素数さん:2009/03/22(日) 09:50:28
>>170
考えてみたけど、ごちゃごちゃした証明しか思いつかない
簡単に証明できたんなら教えて

172 名前:132人目の素数さん:2009/03/22(日) 14:46:10
>>159, >>171 (>>164にあるp[n]の母関数を使いました)
F(t):=-(a+b+c)+ae^(xt)+be^(yt)+ce^(zt) をマクローリン展開すると仮定により
F(t) = t + t^2 + t^3 + t^4 + t^5 + t^6 + (7次以上の項)
となる。この6次までの項からなる多項式を G(t) とおく:
G(t) = t + t^2 + t^3 + t^4 + t^5 + t^6。

f(t):=F'(t) は(A=x+y+z, B=yz+zx+xy, C=xyz とおくと)
f'''-Af''+Bf'-Cf=0 を満たすので
g(t):=G'(t) に対して g'''-Ag''+Bg'-Cg の2次までの項は無い。(※)-->>173

実際に計算すると(h:=g'''-Ag''+Bg'-Cg とおくと)
h(t) = 24-6A+2B-C + (-2C+6B-24A+120)t + (-3C+12B-60A+360)t^2 + (3次以上の項)
となるので A,B,C は連立方程式
24-6A+2B-C=0, -2C+6B-24A+120=0, -3C+12B-60A+360=0
の解で、これを解くと A=12, B=36, C=24 が得られる。

173 名前:132人目の素数さん:2009/03/22(日) 14:47:02
注:一般に二つの関数f(t),g(t)のマクローリン展開がn次の項まで一致すれば
二つの関数 f'''-Af''+Bf'-Cf と g'''-Ag''+Bg'-C のマクローリン展開は
n-3次の項まで一致します。>>172では(※)でそれを使ってます。

手で計算するのが面倒ならMaximaで↓これを1行ずつ実行させればいいです。
F(t):=-(a+b+c)+a*exp(x*t)+b*exp(y*t)+c*exp(z*t); taylor(F(t),t,0,6);
G(t):=t+t^2+t^3+t^4+t^5+t^6; define(g(t), diff(G(t),t,1));
define(h(t), diff(g(t),t,3)-A*diff(g(t),t,2)+B*diff(g(t),t,1)-C*g(t))$ rat(h(t),t);
eq0:h(0)=0;
define(h1(t), diff(h(t),t,1))$ eq1:h1(0)=0;
define(h2(t), diff(h(t),t,2))$ eq2:h2(0)=0;
linsolve([eq0,eq1,eq2], [A,B,C]);

結局これもゴチャゴチャしとるな・・・

174 名前:132人目の素数さん:2009/03/22(日) 14:54:39
訂正:>>173の二行目

(誤) g'''-Ag''+Bg'-C
(正) g'''-Ag''+Bg'-Cg

175 名前:132人目の素数さん:2009/03/22(日) 15:25:07
マクローリン展開のn-3次の項がうんぬんとかやるくらいなら
もう行列式の値を x、y、z の対称式として愚直に
計算したほうがすっきりしてるような

176 名前:132人目の素数さん:2009/03/22(日) 16:10:59
A=x+y+z, B=yz+zx+xy, C=xyz とおくと

x^4 = Ax^3 - Bx^2 + Cx
y^4 = Ay^3 - By^2 + Cy
z^4 = Az^3 - Bz^2 + Cz より 24 = ax^4 + by^4 + cz^4 = 6A-2B+C

x^5 = Ax^4 - Bx^3 + Cx^2
y^5 = Ay^4 - By^3 + Cy^2
z^5 = Az^4 - Bz^3 + Cz^2 より 120 = ax^5 + by^5 + cz^5 = 24A-6B+2C

x^6 = Ax^5 - Bx^4 + Cx^3
y^6 = Ay^5 - By^4 + Cy^3
z^6 = Az^5 - Bz^4 + Cz^3 より 720 = ax^6 + by^6 + cz^6 = 120A-24B+6C

これで終りだった・・・orz

177 名前:132人目の素数さん:2009/03/22(日) 23:50:03
>>176
神キタ━(゚∀゚)━!!!

178 名前:171:2009/03/22(日) 23:59:37
>>172-176
サンクス
これから読ませてもらう

179 名前:132人目の素数さん:2009/03/24(火) 11:05:25
充分に大きい白い容器と黒い容器が無限個ある。
最初、ひとつの白い容器に純水が 1kg、
ひとつの黒い容器に 100% のアルコールが 1kg 入っている。
以下の操作を好きなだけ行って、最終的にひとつの白い容器になるべく
アルコール濃度の高い 1kg の液体を作りたい。
このアルコール濃度の上限はいくらか?

可能な操作
・ひとつの容器から同じ色の別の容器(空でなくてもよい)に好きなだけ液体を移してよく混ぜる
・白い容器と黒い容器をひとつずつ取り、両方の容器の液体を一緒にしてよく混ぜて、
もとの質量と同じだけ両方の容器に分ける
(混ぜる前に、白と黒の容器にそれぞれ m, M の質量の液体が入っていたら、
混ぜたあとも、白と黒の容器にそれぞれ m, M の質量の液体を入れるということ)

180 名前:132人目の素数さん:2009/03/24(火) 11:24:01
濃度は重量濃度です
アルコール濃度 = 液体中のアルコールの質量 / 液体の質量

181 名前:132人目の素数さん:2009/03/24(火) 14:14:48
白い容器のほうの濃度を a、黒い容器のほうの濃度を b 、
操作後の濃度を a1 及び b1 とすると
a < a1 < b1 < b が分かる。
また a1 ≦ 1/2(そうでないとするとアルコールの総量が増えたことになる)。
二番目の操作を繰り返すことによって白い容器内の濃度は 1/2 に
限りなく近づけることが出来る。よって50%。

182 名前:132人目の素数さん:2009/03/24(火) 18:53:59
1 - exp(-1)  ≒  0.632 まで濃度をあげられるんじゃないか?



183 名前:132人目の素数さん:2009/03/24(火) 20:14:35
体積モル濃度を調べよう。

184 名前:132人目の素数さん:2009/03/24(火) 20:46:50
>>181
> a1 ≦ 1/2(そうでないとするとアルコールの総量が増えたことになる)。

ここの理屈がわからん。
なぜa1>1/2だと、アルコールの量が増えたことになるんだ?

185 名前:132人目の素数さん:2009/03/24(火) 22:01:03
1) 最初に白い容器の純水をn個の白い容器に等分する。
2) 次に、n個全ての白い容器に対して、その白い容器をひとつと黒い容器を混ぜ、戻す。
3) 最後にn個の白い容器のをすべてひとつの白い容器に集める。

たとえば、n が 2の場合
1) 純水を白い容器2つに1/2kgづつに分ける。
2-1) ひとつめの白い容器と黒い容器を混ぜ、戻す。 ここで黒い容器に残るアルコールは2/3kg 
2-1) ふたつめの白い容器と黒い容器を混ぜ、戻す。 ここで黒い容器に残るアルコールは(2/3)^2 = 4/9 kg
3) 白い容器をすべて集めると、アルコールは1-4/9 = 5/9 kg

ここで >>181の 
> a1 ≦ 1/2(そうでないとするとアルコールの総量が増えたことになる)。 
は、正しくないことがわかる。

この方法だと、白い容器に入るアルコールの量は、 1-(n/(n+1))^n なので、使用する白い容器の数を増やせば 
最大で lim_{n->∞}(1-(n/(n+1))^n) = 1 - exp(-1) のアルコールを白い容器に入れることができる。

これが最大かどうかは知らん。


186 名前:132人目の素数さん:2009/03/24(火) 22:03:20
× 2-1) ふたつめの
○ 2-2) ふたつめの

187 名前:132人目の素数さん:2009/03/24(火) 22:05:08
瑣末なことだが、白い容器は3個あれば事足りる。

188 名前:132人目の素数さん:2009/03/25(水) 06:54:29
n等分ではなく
1番目の白い容器には1/n、2番目の容器には1/n × (n/(n+1)) … と等比になるように分配する
つまり n番目の容器には 1/n × (n/(n+1))^(n-1) の水。 # この数列の和はもちろん1
(2)以降の操作は同じ。

きちんと計算はしていないが、
nを大きくとれば、白い容器のアルコールを、いくらでも1に近づけることができると思うが、どうだろうか。


189 名前:132人目の素数さん:2009/03/25(水) 07:07:32
訂正:
× 1番目の白い容器には1/n 
○ 1番目の白い容器には1/(n+1)


190 名前:132人目の素数さん:2009/03/25(水) 07:16:12
あ、ダメか 
1 - exp(-1) は超えられないや。

191 名前:132人目の素数さん:2009/03/26(木) 02:03:09
某サイトからの引用。

2人組の手品師AとBが、観客に対して次のようなマジックを行なう。
問: このマジックのタネ(phase0 の内容)を考案せよ。

(phase0)
事前にAとBは綿密に打ち合わせをしておく。

(phase1)
Bには目隠しと耳栓をさせる。Aは1組52枚のトランプカードを
全て観客の一人に渡し、その中から好きな5枚を選んでもらう。
余った47枚はその場で廃棄する。

(phase2)
Aは、観客が選んだ5枚の内容を確認した上で、その中の1枚を指定する。
観客はAが指定した1枚を手に残して隠し持ち、その他の4枚をAに返却する。
Aはその4枚を表向きにして机の上に並べ※、舞台から退場する。

(phase3)
Bが目隠しを外し、机上の4枚を見て観客の手にある1枚を当てる。(終了)

※4枚のカードを机の上に並べる際は、あらかじめ固定された
「同じ向き」「等間隔」「一列」のポジションに置かなければならない。
Aのアレンジが許されるのは、4枚の「並び順」のみであるとする。

192 名前:132人目の素数さん:2009/03/26(木) 02:16:34
コマ大のやつね

193 名前:191:2009/03/26(木) 02:56:14
あらら。有名だったのかな。
一応、引用元はここの19番なんだけど。
http://www.qbyte.org/puzzles/

他にもいろいろ面白いのがありそうでマジお勧め。
皆さんも気に入ったのがあったら翻訳転載よろ。

194 名前:179:2009/03/26(木) 07:29:29
>>190
その方法はまだ計算してないから、上限がいくらかは分からないけど、
別のやり方で 1-(1/e) 以上にできます

>>193
そのサイトの方法だと52枚までだけど、もっと枚数増やせないんだろうか

195 名前:132人目の素数さん:2009/03/26(木) 13:42:27
>>193
http://gascon.cocolog-nifty.com/blog/2008/11/112-3c56.html
この後半に紹介されているが、そこから採ったのかも。番組では実演して失敗してた。

196 名前:132人目の素数さん:2009/03/26(木) 17:30:35
a


197 名前:132人目の素数さん:2009/03/27(金) 00:24:19
>>194
直後の第20問で、124枚まで増やした場合が採り上げられてるよ。

198 名前:132人目の素数さん:2009/03/27(金) 03:08:48
以下の条件を満たす四角形は存在するか?
存在するなら例示し、存在しないならその事を証明せよ。

(i)
三辺の長さと対角線の長さが全て整数
(ii)
四辺の長さと対角線の長さが全て整数

199 名前:198:2009/03/27(金) 03:14:11
書き損ねた。
(iii) (ii)を満たし、かつどの2辺をとっても長さが等しくならない。

200 名前:132人目の素数さん:2009/03/27(金) 13:22:54
(iii)とは?

201 名前:132人目の素数さん:2009/03/27(金) 15:28:19
四辺の長さと対角線の長さが全て整数
かつ
どの2辺をとっても長さが等しくならない。

すると(ii)は円か何かを使うのかな。

202 名前:132人目の素数さん:2009/03/27(金) 15:40:57
(i)(ii)は同じ長さの辺があってもいいの?
もしそうなら、例えば縦横比3:4の長方形(対角線は5)があるけど。

203 名前:132人目の素数さん:2009/03/27(金) 23:31:05
くそ、卑猥な記号ばかり並んでやがる

204 名前:132人目の素数さん:2009/03/28(土) 01:19:56
問題 1.
a,b,c は正の実数で、a+b+c=1 を満たすとき
 a^(1-a) * b^(1-b) * c^(1-c) ≦ 1/9,

問題 2.
(a) 2008のすべての約数d >0 に対して P(d) = 2008/d,
  となるような 整数係数の多項式P(x)は存在するか?

(b) nのすべての約数d >0 に対して P(d) = n/d,
  となる整数係数の多項式P(x)が存在するような自然数nを求めよ。

問題 4.
fは正整数から非負整数への写像とする。次の条件を満たすfをすべて定めよ。
 (1) f(mn) = f(m) + f(n),
 (2) f(2008) = 0,
 (3) f(n) = 0, for all n≡39 (mod 2008).

問題 5.
nを自然数とするとき、数列 n + [√n] + [ n^(1/3) ] に含まれない自然数をすべて挙げよ。
ここに [ x ] はx以下の最大の整数である。

http://www.math.ust.hk/excalibur/v13_n5.pdf
Austrian M.O. 2008, Final round (part 2)
2008/06/07〜08

205 名前:132人目の素数さん:2009/03/28(土) 18:10:26
問2への答え
(a)2008=251*8より約数は1,2,4,8,251,502,1004,2008の8つ。
今f(1)=2008,f(2)=1004,f(4)=502,f(8)=251,f(251)=8,f(502)=4,f(1004)=2,f(2008)=1
よってf(x)=(x-1)A(x)+2008=(x-2)B(x)+1004=(x-4)C(x)+502=(x-8)D(x)+251
=(x-251)E(x)+8=(x-502)F(x)+4=(x-1004)G(x)+2=(x-2008)H(x)+1

一般にA(x)とD(x)の次数が同じならば、A(x)〜H(x)を整数係数多項式として
f(x)=A(x)B(x)+C(x)=D(x)E(x)+F(x)と書ける時、f(x)=A(x)D(x)G(x)+H(x)と書ける為
与式を満たす整数係数多項式P(x)は存在する。

(b){n|nの平方根が整数にならない}

206 名前:132人目の素数さん:2009/03/28(土) 19:04:00
>>205
(b)なんだけど、おれが考えた答えと違う。

x = (nの約数) のとき、 xP(x)-n = 0 である。
よって xP(x)-n = Q(x) (x-d_1) (x-d_2) ... (x-d_m) (ただし、d_i はすべての n の約数を渡る。Q(x) は適当な整数係数多項式)
両辺の定数項を比較して n は d_1 * d_2 * ... * d_m の倍数である。
nの約数はn自身も含むので、nが素数でなければ、n < d_1 * d_2 ... * d_m となるので
nが素数であることが必要条件。

逆に n が素数なら、xP(x) - n = -(x-1)(x-n) = -x^2 + (n+1)x - n として
P(x) = x - (n+1) をとればいい。

よって n が素数であることが必要十分。

207 名前:132人目の素数さん:2009/03/29(日) 03:15:42
nが素数のときはP(x)=x−(n+1)とすればよい。

nが合成数のときは、ある素数p,qについてn=pqmとなる。
P(x)の定数項をaとする。

p=qのとき:仮定よりP(n)=1だから、a≡P(n)≡1 (mod p)が成り立つ。
次に、x=pとして、P(p)=n/p=pm となるからP(p)≡0 (mod p)
一方、P(p)≡a≡1 (mod p)だから、矛盾。

p≠qのとき:P(n)=1だから、a≡P(n)≡1 (mod p)が成り立つ。また、
P(pm)=n/(pm)=qだから、a≡P(pm)≡q (mod p)となり、よってq≡1 (mod p)が
成り立つ。つまりp|(q−1)が成り立つ。これとq−1>0より、q−1>pとなる。
pとqの役割を入れ替えても同様の議論が成り立ち、そのときp−1>qが得られる。
よってq−1>p>q+1となり、矛盾。

208 名前:132人目の素数さん:2009/03/29(日) 06:45:31
>>205
f(x) = (x-1)A(x) +2008,
A(x) = -1004 + (x/2 -1)[502 -(x/4 -1){251 - (x/8 -1)[8 -(x/251 -1){4 - (x/502 -1)[2 - (x/1004 -1)]}]}] + r(x)
   = -1004 + 502(x/2 -1) -251(x/2 -1)(x/4 -1) +8(x/2 -1)(x/4 -1)(x/8 -1) -4(x/2 -1)(x/4 -1)(x/8 -1)(x/251 -1) +2(x/2 -1)(x/4 -1)(x/8 -1)(x/251 -1)(x/502 -1) -(x/2 -1)(x/4 -1)(x/8 -1)(x/251 -1)(x/502 -1)(x/1004 -1) +r(x)
r(x) = (x-2)(x-4)(x-8)(x-251)(x-502)(x-1004)(x-2008)g(x),

209 名前:132人目の素数さん:2009/04/01(水) 01:19:15
数日前に質問スレで、以下の趣旨の問題が投下され、解決することなく流れていた。

「1〜nの番号がついた玉を無作為に一列に並べたとき、連続するどの2つの番号も、
その順番通りに隣接して配置されない順列のパターン数は?」
(もとの問題は楽曲のシャッフル演奏が題材だった)

n=3の場合、12も23も現れない配置ということで、132、213、321の3通りとなる。
帰納的な考察により、そのようなパターン数をa(n)とおいたとき、
a(1)=a(2)=1として、a(n)=(n-1)*a(n-1)+(n-2)*a(n-2)となることがわかった。

これって解けるのかな。
閉じた式じゃなくても、再帰構造が排除できればいいとして。

210 名前:132人目の素数さん:2009/04/01(水) 02:06:54
>>209
不完全ガンマ関数Γ(m+1,x) := ∫[x,∞] t^m exp(-t) dt を使うと
 a(n) = Γ(n+2,-1) / (n e) になる.
不完全ガンマの展開公式
 exp(x) Γ(n+2,x) = Γ(n+2) Σ[k=0,n+1] x^k/k!
を使えば
 a(n) = ( (n+1)! Σ[k=0,n+1] (-1)/k! ) / n
になる.例えば n = 3 だと 4! (1 - 1/2 + 1/3! - 1/4!) / 3 = 3.

211 名前:132人目の素数さん:2009/04/01(水) 16:35:36
>>210
なんか違わない?計算が合わないんだけど。

>>209
y = x + y' (x^2+x^3) ていう微分方程式を解けば、各係数がその数列のなっているはず。


212 名前:132人目の素数さん:2009/04/01(水) 20:22:10
>>211
ん、あわない?具体的に指摘頼む。
漸化式と一致してることは、小さい n に対しては確認したつもりだけど。

213 名前:132人目の素数さん:2009/04/01(水) 20:24:07
>>212
> 漸化式と一致してることは、小さい n に対しては確認したつもりだけど。
> 漸化式と一致してることは、小さい n に対しては確認したつもりだけど。
> 漸化式と一致してることは、小さい n に対しては確認したつもりだけど。
> 漸化式と一致してることは、小さい n に対しては確認したつもりだけど。
> 漸化式と一致してることは、小さい n に対しては確認したつもりだけど。
> 漸化式と一致してることは、小さい n に対しては確認したつもりだけど。
> 漸化式と一致してることは、小さい n に対しては確認したつもりだけど。


214 名前:132人目の素数さん:2009/04/01(水) 23:31:42
>>209
b[n]=n*a[n]とおけば、その漸化式はb[n]=n*b[n-1]+n*b[n-2]と表せる。
b[n]-(n+1)*b[n-1]=-(b[n-1]-n*b[n-2]) と書けるので、n≧3とすれば
b[n]-(n+1)*b[n-1]=(-1)^(n-2)*(2*1-3*1*1)=(-1)^(n-1) となる。
これはn=2でも正しいので、以下n≧2とする。
b[n]=(n+1)b[n-1]+(-1)^(n-1) の両辺を(n+1)!で割れば
b[n]/(n+1)!=b[n-1]/n!+(-1)^(n-1)/(n+1)!
b[n]=(n+1)!*{Σ[k=2,n](-1)^(k-1)/(k+1)!+b[1]/2!}
   =(n+1)!*Σ[k=1,n](-1)^(k-1)/(k+1)!
よってa[n]=(n+1)!/n*Σ[k=1,n](-1)^(k-1)/(k+1)!=
これはn=1でも成り立つ。

215 名前:132人目の素数さん:2009/04/02(木) 00:33:16
事故解決しました

216 名前:209:2009/04/24(金) 00:15:31
>>210-214
遅くなったが、ありがとう。

217 名前:132人目の素数さん:2009/04/24(金) 21:42:07
A君はn枚、B君はn+1枚の公正なコインを持っている(n≧1)。
両者ともに全てのコインを投げたとき、A君の表の枚数よりも
B君の表の枚数の方が真に大きくなる確率を求めよ。

218 名前:132人目の素数さん:2009/04/24(金) 22:48:53
age

219 名前:現場の職人:2009/04/24(金) 23:50:17

切り出した木の側面を切って

最も無駄のない柱を作るには

曲尺をどのようにして使うのであろうかっ。

【 配点 1 点 】


220 名前:132人目の素数さん:2009/04/25(土) 03:31:21
最も無駄のない とは どういう意味なのか

221 名前:ユビー ◆6wmx.B3qBE :2009/04/25(土) 07:47:17
>>217
nも乱数で確率なの?

222 名前:132人目の素数さん:2009/04/25(土) 10:28:49
>>217
nの値にかかわらず、求める確率は 1/2

223 名前:132人目の素数さん:2009/04/25(土) 10:43:31
>>219
糞スレ立てんなウンコ虫が

224 名前:132人目の素数さん:2009/04/25(土) 12:38:27
>>223
用語の間違いに注意

225 名前:キノコ狩りが趣味 ◆ghclfYsc82 :2009/04/25(土) 12:51:17
あの〜
ワラビが右巻きでゼンマイが左巻きだって、どうやって証明したらいいのでしょうか?



226 名前:132人目の素数さん:2009/04/25(土) 12:56:00
>>225
まずワラビとゼンマイの定義がなければ話になりません
それらの定義を述べてください

227 名前:132人目の素数さん:2009/04/25(土) 13:13:59
転載
http://science6.2ch.net/test/read.cgi/math/1240495780/38
解答案は75。

相異なる9個の整数からなる集合Sがあり、各元の正の素因数はすべて3以下である。
Sからうまく相異なる3個の元をとれば、それらの積がある整数の3乗になることを示せ。

228 名前:キノコ狩りが趣味 ◆ghclfYsc82 :2009/04/25(土) 17:09:22
>>226
ワラビは京浜東北線の駅にありますが、ゼンマイは昔の時計で使いました。
どっちが美味しいんでしょうか、それだけでも知りたくて・・・



229 名前:132人目の素数さん:2009/04/25(土) 17:27:57
>>228
まずはワラビやゼンマイの定義を述べよ

230 名前:ぺれるまん ◆ghclfYsc82 :2009/04/25(土) 18:52:17
>>229
調べたんですが、ゼンマイの学名はOsmunda japonicaで山野に生えてて水気が多いところを
好むという特徴だけなんですね。それでワラビは確定した分類体系さえ無いんだそうで、食べ過ぎ
たらアカンそうですが、色んな食べ方があるそうですねぇ。どうやら山でなくても畑でも出るそうで、
おひたしと天麩羅がおススメだそうです。

何方か定義を御存じではないでしょうか?



231 名前:132人目の素数さん:2009/04/25(土) 22:25:36
今月の日経サイエンスのパズルがわからん。
問題の概要はこんな感じ。

4名の死刑囚(A,B,C,D)が一人ずつ部屋に入って運命のくじ引きをする。
部屋には4つの箱があって、それぞれの箱に1枚ずつ
A,B,C,D誰か一人を助ける免罪符が入っている。

4つの内3つの箱を開けて、自分の免罪符を引き当てれば勝ち。
ただし、4人は一心同体なので、誰かが失敗すれば全員処刑される。

部屋には一人ずつ順番に入り、別の出口から出るので、
どの箱にどの免罪符が入っていたかを教えることは出来ない。

単純に勝率を計算すると、(3/4)^4=81/256で、勝率は1/3以下。
しかしAには3/4の勝算があり、B,C,Dにそれを伝えた。
・・・ここまで。

232 名前:231:2009/04/25(土) 22:29:45
事前の相談が許されてるから、勝率0%のやり方を避けることは出来る。
(全員同じ開け方をすれば確実に死ねる)
完全に無作為に開けるんじゃなくて、それを避けるという相談をするだけで、
ちょっぴり勝率が上がるのはわかる。

また、一人目が失敗したときの事は考えなくて良いから、
一人目がどこを開けたか聞いておけば(事前に決めておけば)、
二人目以降の勝率が若干上がることはわかる。

しかし、どういう戦略をとっても、最初の一人の勝率は3/4だろ。
そうすると、後の3人はその後100%成功しなくちゃいけない。
が、100%にはなりそうにないんだが。。。

一人目が箱を開けっ放しにするとか、
ガンのための傷を付けるとかのズルしか思いつかん。
福本伸行の読み過ぎ?

233 名前:132人目の素数さん:2009/04/26(日) 00:39:22
箱の中身を入れ替えればいいじゃん

234 名前:132人目の素数さん:2009/04/26(日) 03:34:23
中身の入替えを許したら簡単すぎじゃね?

もっとも、箱の配置が指定されてないから、部屋の中で
箱がいかように配置されていたとしても、そのうちの1個を
特定できるようなルールをあらかじめ策定するってのは、
それはそれで面白いかもしれない。

しかしそういう意図の問題なんだろうか。

235 名前:231:2009/04/26(日) 04:18:58
>>233
最初の人が、箱の中身を入れ替えるか・・・
(1枚は入れ替えられないが、次の人がなんとかする?)
いっそ箱の並びを変えて、左からABCDにしておけば、
後の人は簡単だわな。

もう一回問題を読み直してみたが、
免罪符の箱の部屋に見張りが居るかどうかは書いてなかった。
しかし・・・見張りが居なかったら、3つじゃなくて4つ全部開けるのも可能だろう。

ヤンジャンや近代麻雀じゃなくて、日経サイエンスだから・・・
ルールの穴じゃなくて、場合分けとかで解くと思うんだが。

236 名前:132人目の素数さん:2009/04/26(日) 06:00:00
こんな確率もとめてみたい その1/3
http://science4.2ch.net/test/read.cgi/math/1109546954/
430-485


237 名前:132人目の素数さん:2009/04/26(日) 09:14:14
>>231
事前相談のみで
後の人に情報が残せない場合
(前の人が開けた結果に応じた作戦変更ができない)
最大で 9/24 にしかならないので (すべての組み合わせを試した)
なんらかの方法で情報を残すことをしないと、それ以上にはできない。

238 名前:132人目の素数さん:2009/04/26(日) 14:06:37
各箱にA,B,C,Dと名前をつける。
囚人Xは、最初に自分と同じ名前のついた箱Xをあける。
箱Xの中に、Yの免罪符があったら、次に箱Yをあける。
箱Yの中に、Zの免罪符があったら、次に箱Zをあける。

239 名前:132人目の素数さん:2009/04/26(日) 17:42:14
なるほど

240 名前:231:2009/04/26(日) 21:34:54
>>238
A箱からスタートして、Aカードが3つ目だった場合、その次はまたA箱に来る。
つまり、3の輪っかが出来ているので、全員セーフ。

A箱からスタートして、Aカードが2つめだった場合、
他の箱は最悪でも2輪っかだからセーフ。

A箱からスタートして、Aカードがいきなりあった場合、
他の箱は最悪でも3輪っかだからセーフ。

A箱からスタートしてAカードが4の距離だった場合、
4輪っかが出来ているので、全員死亡。

最初の人が失敗した場合は必ず全員失敗し、
最初の人が成功した場合には全員成功するアルゴリズムってことか…

いや…すごすぎる!!! 俺も、成功の場合を裏返して、
「最初の人が失敗した場合に他の失敗も集めてしまう戦略にするんじゃないか」
とは思ったけど…

類似問題知らないで解けたとしたら、IQ150-160くらいありそうだ。

241 名前:132人目の素数さん:2009/04/26(日) 22:11:25
上手いね。
出題者は巡回置換からこの問題を発想したんだろうか。

1 2 3 4
2 3 4 1

みたいな配置だったら失敗。そうでなければ成功。

いやー、こんな応用があったとは。

242 名前:132人目の素数さん:2009/04/28(火) 17:46:06
コインランドリーを並べ替えてできる言葉はコインランドリーを含めて何通りあるか。
ただし、ンおよびーを頭に持ってきてはいけません。また、ンが連続してもいけませんし、ンの直後にーがきてもいけません。

243 名前:132人目の素数さん:2009/04/28(火) 18:08:43
全文字使うのか?

244 名前:132人目の素数さん:2009/04/28(火) 18:23:28
>>243
もちろん

245 名前:132人目の素数さん:2009/04/28(火) 19:16:58
ちゃんとした言葉になってなくてもいいの?

246 名前:132人目の素数さん:2009/04/28(火) 19:22:09
インリンドコラー

247 名前:132人目の素数さん:2009/04/28(火) 19:28:51
ただ数えるだけ。4500個。

248 名前:132人目の素数さん:2009/04/28(火) 19:52:54
意味のない言葉でももちろんかまいません。
4500個ではないと思いますよ。

249 名前:132人目の素数さん:2009/04/28(火) 19:57:11
プログラム組んでみたら12000個だった。
でも一つ目と二つ目の「ン」が入れ替わっても違うって判定されてた。
もっかい直してみる。

250 名前:132人目の素数さん:2009/04/28(火) 20:04:30
> でも一つ目と二つ目の「ン」が入れ替わっても違うって判定されてた。 

てことは半分ってことではないのか?

251 名前:132人目の素数さん:2009/04/28(火) 20:11:23
理屈の上ではそうなんだけど一応修正したらやっぱり6000個になった。

252 名前:132人目の素数さん:2009/04/28(火) 20:30:46
「ン」「ー」が先頭に来ない場合の数は全部で
 5*7!=12600通り
「ンン」が並ぶ場合の数は、「ンン」を1組として
 5*6!=3600通り
「ンー」が並ぶ場合の数は、「ンー」を1組として
 5*6!=3600通り
「ンンー」が並ぶ場合の数は、「ンンー」を1組として
 5*5!=600通り
よって、12600-3600-3600+600=6000通り

間違ってたら悲しむ

253 名前:132人目の素数さん:2009/04/28(火) 20:40:17
6000通りで正解です。出題者の私は、最初に、コイラドリの並べ方120通りに、ン2つ、ーを組み込むという考え方で計算しましたが。

254 名前:132人目の素数さん:2009/04/28(火) 20:42:17
正八面体が存在することを示せ。

京大の入試だったような。

255 名前:132人目の素数さん:2009/04/28(火) 20:45:00
場合の数だから賢い小学生なら解けちゃう

256 名前:132人目の素数さん:2009/04/28(火) 21:15:15
条件数え落としてた。6000通りだね。
ただ、どこが面白い問題かはわからんなあ。
どこを面白いと思って出題したの?

257 名前:132人目の素数さん:2009/04/29(水) 07:36:32
単なる重複順列じゃなくて、いろいろ条件が付くとこかな。

258 名前:132人目の素数さん:2009/04/29(水) 23:05:48
>>254
模範解答よろ〜

259 名前:132人目の素数さん:2009/04/30(木) 00:09:00
具体的に作ればいいんじゃないかな

260 名前:132人目の素数さん:2009/04/30(木) 00:13:49
>>254
|x| + |y| + |z| ≦ 1

261 名前:132人目の素数さん:2009/04/30(木) 00:33:57
正八面体のサイコロ買ってきて見せれば示した事にならないかな。

262 名前:132人目の素数さん:2009/04/30(木) 00:44:10
>>261
その模型が正八面対であることを証明せねばならない

263 名前:132人目の素数さん:2009/04/30(木) 15:48:46
正三角形4つと正方形1つで四角錐を構成して
それをふたつ用意し底面の正方形同士をくっつけたら
正8面体になることを利用すればいいんじゃないか?

四角錐の側面が互いに同相なのは自明として使っていいのかな?

上下にくっつけて(正8面体を作って)それを縦に切ってできた
四角錐が元の四角錐と合同だということを示すのはどうだろう?





264 名前:132人目の素数さん:2009/04/30(木) 15:50:52
正6面体が存在することを仮定していいのなら
正8面体の構成はたいして難しくないが…

265 名前:132人目の素数さん:2009/04/30(木) 15:54:08
>>263
>正三角形4つと正方形1つで四角錐を構成して
>それをふたつ用意し底面の正方形同士をくっつけたら

最初の正四角錐の頂点以外の頂点のまわりの面のなす角が等しいことを証明する必要がある。

266 名前:132人目の素数さん:2009/04/30(木) 15:58:38
>>265
まさにそれを証明するのに、そこ以下が書かれているのだ。

267 名前:132人目の素数さん:2009/04/30(木) 16:02:02
>>264
正6面体の存在は、高さが任意の正四角柱の存在を仮定すれば
そんなに難しくない。

268 名前:132人目の素数さん:2009/04/30(木) 16:04:28
>>267
このあたりになると、何をどこまで仮定していいのか難しいな。
元の問題に、○○は仮定していいなどの記述がないと何もできん。

269 名前:132人目の素数さん:2009/04/30(木) 16:06:33
空間中に任意の角度で任意の長さの直線が引けること
空間中に任意の平面が用意できること

さすがにこのくらいは仮定してよいだろう。

270 名前:132人目の素数さん:2009/04/30(木) 16:09:05
正三角形や正方形の存在から証明かよ…

271 名前:132人目の素数さん:2009/04/30(木) 16:44:29
二つの面が作る角度が180度未満になることも証明しないとな。

272 名前:132人目の素数さん:2009/04/30(木) 17:03:57
正八面体は、普通にxyz空間上に座標をとって構成すればいいんジャマイカ

273 名前:132人目の素数さん:2009/04/30(木) 22:47:28
ところで俺、正八面体って頂点の角度とか形が一定じゃないから、
正多面体の仲間に入ってるのに抵抗感じるんだけど。

274 名前:132人目の素数さん:2009/04/30(木) 22:49:48
>>273
どういう意味ですか?

275 名前:132人目の素数さん:2009/04/30(木) 22:52:27
カーボンナノチューブは正何面体の角を削るとできるか?


276 名前:132人目の素数さん:2009/04/30(木) 23:33:34
>>254
1)答案用紙に展開図と投影図をいくつか書いておく。
 「こうやれば貴方にも正八面体が作れます!」とか。
2)三次元空間には正八面体が存在しえないと仮定して、なんらかの矛盾を導く。
3)正多面体の一般論を展開して、ハルヒにも出てきた多面体定理かなんかから、
 正八面体の存在を示す。l

・・・やっぱ1が一番簡単そうじゃね?

>>273
確かにそう見えるけど、目の錯覚だろ。
正六面体に内接するんだから…

>>275
チューブは無理だろ。鉛筆キャップみたいのもあるし。
C60 フラーレンのこと?

277 名前:132人目の素数さん:2009/05/01(金) 03:00:28
>>276
2)が一番かっこよさそうなので、それでお願いします。

278 名前:132人目の素数さん:2009/05/01(金) 11:31:55
>>273
なんかい見直しても一定だが。

279 名前:132人目の素数さん:2009/05/01(金) 13:33:19
ここに縦が a、横が bの長方形の紙があります。この紙にハサミを入れて、
できる範囲の中で面積最大となる正三角形を切り取ります。
この切り取った正三角形の面積を Sとするとき、
この Sを a、bを使って表してください。

面積最大となる理由も併せてお答えください。

280 名前:132人目の素数さん:2009/05/01(金) 14:21:40
面白い解法があるの?

281 名前:132人目の素数さん:2009/05/01(金) 15:01:52
>>279
「面積最大となる理由」の部分を真面目にやると、結構面倒だな。

いくつかの補題を示して、可能性を絞り込む。
以下、正三角形は長方形の内部(周を含む)に含まれるものとする。

1)正三角形の3つの頂点のうち2つ以上が長方形の周上にないとき、その正三角形は面積最大ではない。
→それより大きい正三角形の存在を示す

2)正三角形の3つの頂点のうち2つのみが長方形の周上にあり、その2つは長方形の同一の辺上にはなく、互いに向かい合う辺上にもないとき、その正三角形は面積最大ではない。
→平行移動すると、3頂点とも周上にない正三角形になる

3)正三角形の3つの頂点のうち2つのみが長方形の周上にあり、その2つは長方形の互いに向かい合う辺上にあり、さらにそのうち少なくとも1つは長方形の頂点と一致するとき、その正三角形は面積最大ではない。
→長方形の頂点と一致する頂点を軸に回転すると、2頂点が周上にない正三角形となる

4)正三角形の3つの頂点のうち2つのみが長方形の周上にあり、その2つが長方形の互いに向かい合う辺上にあるとき、その正三角形は面積最大ではない。
→平行移動すると、3)の状態に

5)正三角形の3つの頂点のうち2つのみが長方形の周上にあり、その2つは長方形の同一の辺上にあり、さらにその2つのうちの少なくとも1つは長方形の頂点と一致しないとき、その正三角形は面積最大ではない。
→それより大きい正三角形の存在を示す

6)正三角形の3つの頂点がいずれも長方形の周上にあり、なおかつ長方形の頂点とは一致せず、さらにそのうちどの2つをとっても長方形の同一の辺上にはないとき、その正三角形は面積最大ではない。
→平行移動すると、3)の状態に

282 名前:132人目の素数さん:2009/05/01(金) 15:44:23
abのどっちが長いかわからんが、
a>bなら
s = a * a√(3) / 2
a<bなら
s = b * b√(3) / 2
a=bならどっちでも可

じゃないの?

283 名前:132人目の素数さん:2009/05/01(金) 15:55:25
>>280>>282
そんなに簡単な問題ではありませんよ

284 名前:132人目の素数さん:2009/05/01(金) 16:36:06
結構昔に、どっかの国立大学の推薦入試で長方形の紙を渡されて、「折り目で出来るだけ大きな正三角形を作ってみて」と言われたヤツがいたのを思い出した。

285 名前:132人目の素数さん:2009/05/01(金) 19:39:26
切り出した図形を組み合わせて作るとかはアリかな?
それなら
S = a * b
でもおk。

286 名前:132人目の素数さん:2009/05/01(金) 19:49:38
はさみを入れて、だからそれもアリなのか?!

287 名前:132人目の素数さん:2009/05/01(金) 19:53:07
>>285
具体的にどう鋏をいれるの?

288 名前:132人目の素数さん:2009/05/01(金) 20:05:56
>>285
切った断片を張り合わせるのは無しです。

289 名前:132人目の素数さん:2009/05/01(金) 20:14:29
普通に、縦が a、横が bの長方形の中に入る正三角形の最大面積 Sを求めてください。
>>282の回答は不正解です。

290 名前:132人目の素数さん:2009/05/01(金) 20:34:31
場合わけ面倒なんで、縦a、横1,a>=1としてもいいかい?




291 名前:132人目の素数さん:2009/05/01(金) 20:45:28
>>290
いいですよ

292 名前:132人目の素数さん:2009/05/01(金) 21:07:21
>>282は条件入れ替えであってない?

293 名前:132人目の素数さん:2009/05/01(金) 21:08:53
いや、あってないな
すまない

294 名前:132人目の素数さん:2009/05/01(金) 21:10:50
>>292
どっちにしろ間違ってます

295 名前:132人目の素数さん:2009/05/01(金) 21:26:58
なんか大学受験おもいだした。
とりあえず途中経過。

(1)a>=2*sqrt(3) の時
S=1/sqrt(3)

(2)1<=a<2*sqrt(3)の時、
a=tan(x)+cos(π/6+x)/cos(x)とおくと
S=sqrt(3)/(2*(cos(x)^2)

あとは式変形してSをaで表せればおkかのう。


296 名前:295:2009/05/01(金) 21:29:24
あ〜最大になることはどうやって証明しよう…
意外と迷うな。

297 名前:ルイス・キャラメル:2009/05/01(金) 21:58:34

ある日、花咲く森の中で、熊さんに出会ったときのこと。

そのとき、熊さんが「お逃げなさい」と言ったのに、

なぜだか、熊さんが あとから追いかけてくる。

というのは、数学的論理から考えれば

どのように説明できるのであろうかっ ?


298 名前:132人目の素数さん:2009/05/01(金) 22:16:36
>>295
場合分けの境目は 2*√(3)ではないですよ
あと(2)のところはどうやって出したんでしょうか?
最終的に aを使った式で表してくれますか?

aと bを使うと対称性が見えて、よりきれいな式になるんですけど…

299 名前:295:2009/05/01(金) 22:25:54
境目は2/√3かな。
まず、正三角形の一つの頂点が長方形の一つの頂点と重なってるとしても良い(と思う。証明はちょっと勘弁)
その重なった頂点で、正三角形の辺と長方形の辺のなす角度をxと置くと>>295のaとxの式が出てくる。

300 名前:132人目の素数さん:2009/05/01(金) 22:27:34
>>297
熊さん「早く逃げるんだお嬢! 俺の理性が残っている内に・・・」

熊さんZ(理性崩壊)「うがあああぁぁぁあ!! 待てやこのアマァァアア!!!」

301 名前:132人目の素数さん:2009/05/01(金) 22:45:08
黙れ

302 名前:ルイス・キャラメル:2009/05/01(金) 22:57:44

熊さんが「お逃げなさい」と、言ったのを「正」とすれば

熊さんが追いかけてくるのは、どのように例えるかを

もっと、ふかく考えようよっ。


303 名前:132人目の素数さん:2009/05/01(金) 22:59:32
>>299
その考え方で合ってます。境目もそれでOKです。
ただ、>>295のaとxの式がどうやって立てたのか…
たぶん僕のやり方と違う方法でやってるんですね。

一応、aと bを使った場合の答えを書いておきます。

bを固定して aの範囲を動かす
(1)a =< (√(3)/2)*b のとき、S = (1/√(3))*a^2
(2)(√(3)/2)*b =< a =< (2/√(3))*b のとき、S = √(3)*a^2 - 3ab +√(3)*b^2
(3)(2/√(3))*b =< a のとき、S = (1/√(3))*b^2

あとは(2)でそのような正三角形が必ず取れることと面積最大の証明です。

304 名前:295:2009/05/01(金) 23:24:08
出来れば解きたかったが、明日早い&旅行いってしまうんでギブアップ。
他の人が解いてくれることを祈ってまつ。


305 名前:132人目の素数さん:2009/05/02(土) 03:38:43
>>297
きっと、逃げ惑う少女を追い詰めて襲うシチュエーションが好きなんだよ、クマーさんは

306 名前:132人目の素数さん:2009/05/03(日) 16:50:13
age

307 名前:132人目の素数さん:2009/05/04(月) 02:01:39
ある任意の角を三等分する線を、
定規とコンパスだけをつかって作図する方法を示せ。

308 名前:132人目の素数さん:2009/05/04(月) 02:10:27
最近このスレひどいな。悲しい。

309 名前:132人目の素数さん:2009/05/04(月) 02:25:05
悲しむばかりじゃ能がないので、ちょっと気になった問題を転載。
http://science6.2ch.net/test/read.cgi/math/1240495780/810

問:ふたつの三角形がある。
  それらの外接円の半径、内接円の半径 面積がそれぞれ等しい。
  このふたつの三角形は必ず合同といえるだろうか?

310 名前:132人目の素数さん:2009/05/04(月) 03:17:49
>>309
おもしろそうですね、だが分からんぜ!

311 名前:132人目の素数さん:2009/05/04(月) 08:29:48
>>309
確か三角形の三辺を a, b, c, 内接円・外接円の半径を r, R とすると
三角形の面積 S = (a + b + c) r / 2 = a b c / 4 R

ところで>>279で面積最大の証明はどうなったの?

312 名前:132人目の素数さん:2009/05/04(月) 14:00:00
いえる。


313 名前:132人目の素数さん:2009/05/04(月) 14:42:28
a_{n+1} = 1/2(a_n + 1/a_n) みたいな漸化式の問題知らん?
なんかあったよなぁと思いつつ、初項や係数とか間違ってたら解く過程で気づくだろうとか思ってたら
ぜんぜん手が進まん。

314 名前:132人目の素数さん:2009/05/04(月) 14:49:51
>>313
とりあえずその漸化式はa_n=1/tanh(x_n)と置けば解ける。

315 名前:132人目の素数さん:2009/05/04(月) 14:53:22
>>309
>>311さんのを借りれば
a+b+c=2S/r
ab+bc+ca=4Rr+r^2+(S/r)^2
abc=4RS
で三辺相等

316 名前:132人目の素数さん:2009/05/04(月) 15:10:32
最小公倍数が720である相異なる3つの自然数の組は何通りあるか?

317 名前:132人目の素数さん:2009/05/04(月) 15:35:11
>>315
>ab+bc+ca=4Rr+r^2+(S/r)^2
どうしてこうなるのか教えて下さい

318 名前:132人目の素数さん:2009/05/04(月) 15:39:21
>>314
ありがとう。よく思いつくね。
でも想定していた解きかたと違うんだよなぁ。式が間違ってるのかなぁ。

>>317
ヘロンの公式を整理。

319 名前:132人目の素数さん:2009/05/04(月) 16:08:09
>>313
 a_n = 1/tanh((2^n)x_0), n>0,
ここに、x_0 = (1/2)log|(1-a_0)/(1+a_0)|,
 |a_0| >1 のとき 1/tanh(x_0) = a_0,
 |a_0| <1 のとき 1/tanh(x_0) = 1/a_0,

320 名前:319:2009/05/04(月) 16:20:06
>>313,318
 a_0 = ±1 のとき a_n = a_0,

321 名前:132人目の素数さん:2009/05/04(月) 18:07:27
>>316

自信ないし、日本語変だけど・・・

720 = 2^4 * 5 * 9

なので、
相異なる数を、a, b, c,とすると、

(a,b,c ともに1以上720以下であり)
a は 「2^4」の倍数、・・・(1)
b は 5 の倍数・・・(2)
c は 9 の倍数・・・(3)
である。

また
「3つの数字が全て約数として『9^2』を持つ(・・・(5)とおく)」ことはない。
「3つの数字が全て約数として『 (2^4)^2 』を持つ(・・・(6)とおく)」ことはない。
「3つの数字が全て約数として『 5^2 』を持つ(・・・(7)とおく)」ことはない。

(1)を満たす a は、(2^4) * (5*9) = 720 なので、5*9 個存在
(2)を満たす b は、5 * { (2^4)*9 } =720 なので、{ (2^4)*9 } 個存在
(3)を満たす c は、9 * { (2^4)*5 } =720 なので、{ (2^4)*5 } 個存在
==
===========
よって、5*9と{ (2^4)*9 }と{ (2^4)*5 }をかけた値 マイナス
『(5)成立(6)不成立(7)不成立」+「(5)不成立(6)成立(7)不成立」+「(5)不成立(6)不成立(7)成立」』
===========

計算ギブ・・・。
でもスマートじゃないなー。てか、どっか見過ごしてる。

322 名前:321:2009/05/04(月) 18:12:30
訂正

誤:
「3つの数字が全て約数として『9^2』を持つ(・・・(5)とおく)」ことはない。
「3つの数字が全て約数として『 (2^4)^2 』を持つ(・・・(6)とおく)」ことはない。
「3つの数字が全て約数として『 5^2 』を持つ(・・・(7)とおく)」ことはない。

正:
「3つの数字が全て約数として『9*2』を持つ(・・・(5)とおく)」ことはない。
「3つの数字が全て約数として『 (2^4)*2 』を持つ(・・・(6)とおく)」ことはない。
「3つの数字が全て約数として『 5*2 』を持つ(・・・(7)とおく)」ことはない。

---------------------------------------------------------------------

誤:
『(5)成立(6)不成立(7)不成立」+「(5)不成立(6)成立(7)不成立」+「(5)不成立(6)不成立(7)成立」』

正:『(5)不成立(6)成立(7)成立」+「(5)成立(6)不成立(7)成立」+「(5)成立(6)成立(7)不成立」』・・・(A)


323 名前:132人目の素数さん:2009/05/04(月) 21:00:00
 (5^3−4^3)(3^3−2^3)(2^3−1^3)
−3(5^2−4^2)(3^2−2^2)(2^2−1^2)
+3(5^1−4^1)(3^1−2^1)(2^1−1^1)
−(5^1−4^1)(3^1−2^1)(2^1−1^1)
=7710。


324 名前:132人目の素数さん:2009/05/04(月) 22:48:19
>>323
この愚か者めに日本語で解説おながいします!

325 名前:132人目の素数さん:2009/05/05(火) 10:54:16
>>316
720=2^4*3^2*5
まず、最小公倍数が720である3つの自然数x,y,zの組(同じ物があってもよく、x,y,zは区別する)の個数Jを考える。
x,y,zをそれぞれ
x = 2^a_x * 3^b_x * 5^c_x
y = 2^a_y * 3^b_y * 5^c_y
z = 2^a_z * 3^b_z * 5^c_z
とおくと、
a_x,a_y,a_zはいずれも0〜4の整数で、Max(a_x,a_y,a_z)=4
b_x,b_y,b_zはいずれも0〜2の整数で、Max(b_x,b_y,b_z)=2
c_x,c_y,c_zはいずれも0〜1の整数で、Max(c_x,c_y,c_z)=1
なので、
(a_x,a_y,a_z)の個数が6*6+4*6+1=61
(b_x,b_y,b_z)の個数が1*6+2*6+1=19
(c_x,c_y,c_z)の個数が0*6+1*6+1=7
J=61*19*7=8113

次に、最小公倍数が720である2つの自然数x,yの組(x=yでもよく、x,yは区別する)の個数Kを考えると、同様にして、
(a_x,a_y)の個数が4*2+1=9
(b_x,b_y)の個数が2*2+1=5
(c_x,c_y)の個数が1*2+1=3
K=9*5*3=135

Jの中で、3つのうち2つが一致するものの数は、(K-1)*3
Jの中で、3つとも一致するものの数は、1

よって、最小公倍数が720である3つの異なる自然数x,y,zの組(x,y,zは区別する)の個数は、J-(K-1)*3-1=7710
求める答えは、ここでx,y,zを区別するのをやめればよいので、7710/6=1285通り


326 名前:132人目の素数さん:2009/05/05(火) 10:58:58
問題忘れ去られそうなのでもう一度再掲

縦が a、横が bの長方形の中に入る正三角形の最大面積 Sを求めてください。

面積最大となる理由も併せてお答えください。

327 名前:132人目の素数さん:2009/05/05(火) 11:35:40
>>326
え、それってもうほとんど終わった話じゃないの?

最終的な答えは>>303で合ってんでしょ。
なぜそれが最大かを含めて示すには、
まず>>281の議論を丁寧にやれば、

・正三角形の2頂点が長方形の1辺上にあり、もう1つの頂点が長方形の対辺上にある場合
・正三角形の1つの頂点が長方形の1つの頂点(X)と一致し、あと2つの頂点は、長方形のXを含まない2辺上に1点ずつ存在する場合

の2通りの場合以外は、面積最大にはなりえないことが言えるから、そこを出発点とすれば、無理なく>>303が面積最大だということは示せる。


328 名前:326:2009/05/05(火) 11:49:33
>>327
それをきちっと証明してくださいと言っているのです
口だけですか?

329 名前:132人目の素数さん:2009/05/05(火) 11:54:02
>>327
>>304の人がまだ証明できていないようだったので…

>>328
勝手に僕になりすまさないでください。

330 名前:132人目の素数さん:2009/05/05(火) 11:55:33
後やる事は手を動かすだけ。
各自手元で計算して確かめればいい程度で、ここにいちいち書き込む暇人はそうは居まい。
だいいち、そんなに難しい問題だとでも思っているのか?

331 名前:132人目の素数さん:2009/05/05(火) 12:01:56
>>330
いえ、ぼくはただ>>304が問題を解けないまま忘れ去られては可哀そうだと思っただけです。
>>328は僕ではないので無視してもらって構いません。

332 名前:330:2009/05/05(火) 12:06:15
失礼した。陳謝。
>>330はスルーしてくれ。

333 名前:132人目の素数さん:2009/05/05(火) 12:11:53
匿名掲示板を利用した高度なやり取りに感心した。
なるほどなー。

334 名前:132人目の素数さん:2009/05/05(火) 12:22:45
問題【改】
縦が a、横が bの楕円の中に入る三角形の最大面積 Sを求めてください。


335 名前:132人目の素数さん:2009/05/05(火) 12:32:35
>>334
「正」三角形
じゃなくていいの?
なら、かなり簡単な気が...

336 名前:132人目の素数さん:2009/05/05(火) 12:38:04
それより縦と横って

337 名前:132人目の素数さん:2009/05/05(火) 21:23:26
>>334 長半径a短半径bの楕円に内接する三角形の面積の最大値Sを考えるとすれば、
円に内接する三角形で面積最大の正三角形の変換と考えれば S=(3√3)ab/4 になると思います。
また、この楕円に内接する正三角形の面積の最大値Sは、S=(6√3) a^4 b^2 / (3a^2 + b^2)^2と思いますが、
どちらも一辺aの正方形に内接する正三角形の面積の最大値((2√3)-3)a^2のような形の解がないか心配です。

338 名前:132人目の素数さん:2009/05/06(水) 01:17:02
>>318
そのタイプの問題のまとめは,以前
http://image02.wiki.livedoor.jp/l/y/loveinequality/0ee2036199b33804.pdf
に書いておいたよ。

339 名前:318:2009/05/06(水) 07:11:09
>>338
おお、すごい。でも答えが違った気がするので、やはり式がうろ覚えなんだと思う。
問題が載ってた本は大体見当がついているんだけど、引越ししたときに捨ててしまったようなので
今度調べてくる。

340 名前:132人目の素数さん:2009/05/06(水) 08:33:10
>>339
タイトル期盆濡!

341 名前:132人目の素数さん:2009/05/06(水) 20:16:52
>>340
本屋行ってきて、今帰ってきた。
問題は 「S_n = (a_n + 1/a_n)/2 で a_n > 0 のとき、a_nを求めよ」だった。

本のタイトルは、「入試数学 伝説の良問100」(講談社ブルーバックス)でした。

342 名前:132人目の素数さん:2009/05/06(水) 20:33:05
>>341
ttp://science6.2ch.net/test/read.cgi/math/1241232605/200

かな?

343 名前:132人目の素数さん:2009/05/06(水) 20:34:32
問題の写しまちがいがあって、最終的には341と同じだったはず。
http://science6.2ch.net/test/read.cgi/math/1241232605/215

344 名前:341:2009/05/06(水) 20:58:27
>>342-343
まんまですねぇ。なんとな〜くがっかりした。

345 名前:132人目の素数さん:2009/05/09(土) 20:14:27
===
平面上で、
半径1の円の内部には、半径nの円は最大いくつ入るか?
(どの円同士も、接点以外の交点を持たない)
===
・・・って、ふと考えたんだけど、まともな解き方ってないですかね?

346 名前:132人目の素数さん:2009/05/09(土) 20:19:21
算額みたいな問題ですな

347 名前:345:2009/05/09(土) 20:34:56
発展問題だと、
「半径1の球に半径nの球は最大いくつ入るか?」

348 名前:132人目の素数さん:2009/05/09(土) 20:53:46
その手の詰め込み問題は一般には2次元でもかなり難しい
http://www.stetson.edu/~efriedma/cirincir/

349 名前:345:2009/05/09(土) 22:28:28
>>348
ひええ・・・・

12個以上の場合は、provedでなくfoundなのですね。。。

これ、21以上のときは、未解決なのかな・・・

350 名前:132人目の素数さん:2009/05/10(日) 14:42:38
問題:以下の命題を反証し、その反例を示せ。

命題:3次元空間上に4つの点ABCDがあり、そのx座標、y座標及びz座標は整数である。
この4つの点について3字のベジェ曲線を引いた時、その長さをf(A,B,C,D)とする。
f(A,B,C,D)が整数となるような自明でない点の組は存在しない。

351 名前:132人目の素数さん:2009/05/10(日) 14:43:28
3字の→3次の

352 名前:132人目の素数さん:2009/05/11(月) 04:49:58
>>350
その場合の、「自明でない」とは、どういう事を指すのか?

353 名前:132人目の素数さん:2009/05/11(月) 04:51:20
>>349
数学的には12個も未解決だろう。

354 名前:132人目の素数さん:2009/05/13(水) 23:07:33
有名かな・・・?

536870912人が集まり、テニスのトーナメント戦を行おうとしたが、
そのうちの1人が病欠し、その結果1人が不戦勝となった。

優勝者が確定するまでに、何回試合が行われるか。

ただし、等差数列や等比数列の総和の公式を用いずに求めること。

なお、536870912 = 2^29 である。

(シングルスです。引き分けはあり得ないとします。不戦勝のぶんはカウントしません。
 3位決定戦などは不要で、単に、優勝者を決めるだけのトーナメント戦です)

=====
答えがわかった人、解くのにどのくらい時間かかったか教えてくださいまし。

355 名前:132人目の素数さん:2009/05/14(木) 04:43:05
問題を読むのに3分くらいかな。
中学で習ったよ。

356 名前:132人目の素数さん:2009/05/14(木) 17:04:55
トーナメントの試合数は最初中学で数学ではなく保険体育の時間に習った。


357 名前:132人目の素数さん:2009/05/14(木) 18:27:47
最初の二行読み終わったら分かったな

358 名前:132人目の素数さん:2009/05/14(木) 18:37:57
有名かな?
既出かな?

の人が書くのは全部有名で既出と決まっているので一行目でスルー
俺を含めてもう4人も釣れたな

359 名前:132人目の素数さん:2009/05/14(木) 23:34:21
>>358
> 有名かな?
> 既出かな?
> の人が書くのは全部有名で既出と決まっているので一行目でスルー

あー確かに
なんていうか、算数にロマンを感じて、背伸びして受験数学を理解しようと
必死こいてるモッサンって感じがプンプンしますな〜

360 名前:132人目の素数さん:2009/05/15(金) 02:17:22
>>357
問題文を最後まで読まないせいで間違えたことは何度くらいある?

361 名前:132人目の素数さん:2009/05/15(金) 03:12:32
この人数でテニスのトーナメント戦をして生きてるうちに優勝が決まるのを想像したら感動で涙が止まらなくなった

362 名前:132人目の素数さん:2009/05/15(金) 03:12:55
>>360
お前は今までに食べたパンの枚数を覚えているのか?

363 名前:132人目の素数さん:2009/05/15(金) 03:16:40
>>361
会場さえたくさんあれば、そんなに時間はかからないのだが

364 名前:132人目の素数さん:2009/05/15(金) 03:26:12
会場が2^28もあれば楽勝

365 名前:132人目の素数さん:2009/05/15(金) 03:27:59
>>364
審判の数は足りるのか?

366 名前:132人目の素数さん:2009/05/15(金) 03:30:07
中国かインドから連れてくるか

367 名前:132人目の素数さん:2009/05/15(金) 04:41:40
会場が10000もあれば、2セット1時間で1試合終了として寝ず休まずで6年〜7年もあれば終わるか。会場数が1000だと辛いな。

368 名前:132人目の素数さん:2009/05/15(金) 13:08:04
それよりも世界のテニスの競技人口が1億もいないような

369 名前:132人目の素数さん:2009/05/15(金) 18:35:33
>>354
この人数なら「勝ったものの負傷等で次戦以降棄権」が相当数出そうだから総試合数は上限しか算出できまい。

370 名前:132人目の素数さん:2009/05/16(土) 01:27:33
世界のスポーツの競技人口(自称も含む)

1チェス 5億
2象 棋 5億
3バスケ 4億5000万
4蹴 球 2億4000万
5クリケット 不明
6卓 球 不明

(12は逆の説あり)
以下は順位不明

ビリヤード 1億
ゴルフ 6000万
テニス 5000万
バドミントン 5000万
ソフトボール 5000万
テコンド 5000万
囲 碁 4200万
ボウリング 4000万
ハンドボール 3100万
アメフト 2300万
トライアスロン 2000万
スカッシュ 1500万
野 球 1200万
柔 道 1000万
ビーチバレー 1000万
ラグビー 424万
ラクロス 60万
近代五種  3万

室内のバレーはビーチよりは多そう

371 名前:132人目の素数さん:2009/05/16(土) 01:56:21
>>362
すまない、そんなにだとは思っていなかったんだ。
ほんとにすまない。

372 名前:132人目の素数さん:2009/05/16(土) 02:15:56
>>371
半斤8枚きりとして、一日2枚がおれの朝食。
小学校に入る前、幼稚園の年中組が終ったあの日から、いつも朝はそれだった。
自慢じゃないが、朝食を抜いたことは一度もない。旅行で家を離れたこともない。
だから、食べた枚数は確実に計算できる。閏年も勘定に入れてよい。
一枚の誤差も無い。高校に入った時には、3枚にして欲しいとは思ったが、家計が許さなかった。
だから、今はそれが幸いして、これまでに食べた食パンの枚数を答えることができる。



373 名前:132人目の素数さん:2009/05/16(土) 03:49:33
象 棋
をやってみたいのだが、どういう競技なんだろう。
簡単に説明してくれ。

374 名前:132人目の素数さん:2009/05/16(土) 04:14:58
象象象象象象象象象
 角         飛
香桂銀金王金銀桂香車


375 名前:132人目の素数さん:2009/05/16(土) 06:25:25
>>373
象棋(シャンチー)については中国人に聞くと良い。俗に日本では中国将棋と言ったりするから、どんなもんかは大体想像がつくだろう。競技人口が多いのは中国人がやってるから。
ちなみに他のチェスライクゲームでの競技人口は日本の将棋が1500万、韓国のチャンギは700万、タイのマックルックは500万だそうだ。この五つが世界の五大チェスになるらしい。

376 名前:132人目の素数さん:2009/05/16(土) 06:31:09
一応参考に

http://www.snowdolphin.net/play_room/xiangqi/xiangqi_index.htm

377 名前:132人目の素数さん:2009/05/16(土) 11:58:25
>>354

普通に答えちゃっていいのかな・・・?
間違ってるかもしれんけど。

536870912人から一人抜けて536870911人。

トーナメントは一試合で一人負けて減るから、
最後の一人が決まるまでの試合数は536870911-1で
536870910試合 以上。

378 名前:132人目の素数さん:2009/05/16(土) 13:01:03
ダブルスでやれば1試合でふたり負けて
倍の効率

379 名前:132人目の素数さん:2009/05/16(土) 13:03:34
5億人近くに教えるのも手間かかるな。テニスでまだ良かった。

380 名前:132人目の素数さん:2009/05/16(土) 13:03:42
>>372
給食は食べさせて貰えなかったのか?

381 名前:132人目の素数さん:2009/05/16(土) 13:29:25
パンは危険な食べ物です
のコピペが頭に浮かんだ

382 名前:132人目の素数さん:2009/05/16(土) 16:33:20
千円乞食のごとく週末に湧いてくるなこいつら

383 名前:132人目の素数さん:2009/05/16(土) 20:06:17
昔吸う蝉で見た問題


自然対数eを予備知識を持たない人(小学生など)に出来る限り分かりやすく説明しなさい。

384 名前:132人目の素数さん:2009/05/16(土) 20:14:07
数学の問題になっていない

385 名前:132人目の素数さん:2009/05/16(土) 20:19:28
xの多項式f(x)について、その項数をQ(f(x))とおく。
例) Q(x^3 - 5) = 2、 Q(x^10 + x^8 - 7x + 2) = 4、など

このとき、Q( f(x)^2 ) < Q(f(x)) を満たすf(x)を一つ求めよ。

386 名前:132人目の素数さん:2009/05/17(日) 04:12:25
>>385
そんなもんあるの?
あったら素直にスゴイと思うけど。

387 名前:354:2009/05/17(日) 04:43:19
>>377

そうそうそれ。お見事。

なお、マイクロソフトの面接問題(筆記試験ではない)です。
(桁数はもっと小さいけど)

でも、面接でストレスになってるときに、これ言われたら、
すぐに答えられないかもなー・・・・・と、ある本に書いてました。

388 名前:132人目の素数さん:2009/05/17(日) 04:56:33
>>385

複素数使ってもいいの?

389 名前:132人目の素数さん:2009/05/17(日) 04:56:34
>>385
 f(x) = 1 ±(√2)x -x^2 ±(√2)x^3 + x^4,
 f(x)^2 = 1 ±(2√2)x +7x^4 ±(2√2)x^7 + x^8,
ぢゃだめぽ ・・・・・ orz

390 名前:132人目の素数さん:2009/05/17(日) 05:09:54
ゲーセンにあるAnswer×Answer(全国のゲーセンをオンラインで結んだクイズゲーム
・・・QMAことクイズマジックアカデミーのほうが有名だけどね・・・)で、
どーも、おなじ人となんどもあたったり、
同じ問題が何回も出題されるから、
「プレイ人数少ない&問題数少ない」のか、ゲーム作ってるセガのプログラミングが悪いのか・・・と思って、
Answer×Answerの仕様通りに、

総問題数をQとして
1日にn回ゲームをしたときに、
「今日、さっきみた」っていう問題が出現する回数の期待値とか、だぶった問題の種類の数の期待値とかを、求めようと思ったら、
こんがらがった。
(仕様だと、4人がいっしょに戦って、予選と決勝があって、予選の問題数が約問(<=経験則。正確には違うけど)、
予選の上位二人が決勝に進んで、決勝の問題数が6問(<=経験則で約6問)
(いやほんとは間違えると-10点とか30点とると優勝とかこまかいことあるんだけど)


391 名前:390:2009/05/17(日) 05:11:42
誤:予選の問題数が約問
正:予選の問題数が8問

392 名前:354:2009/05/17(日) 05:29:21
じゃあまあ、

数学だか数学じゃないんだかわからないところで・・・


==============================================================
10進法における1から10までの整数を、マイナス2進法で数えなさい。
ただし、「マイナス2進法」は自分でなるべく論理的に定義すること。
==============================================================

393 名前:132人目の素数さん:2009/05/17(日) 07:15:28
00000001
00000110
00000111
00000100
00000101
00011010
00011011
00011000
00011001
00011110

394 名前:132人目の素数さん:2009/05/17(日) 10:05:03
>>385
ttp://mathworld.wolfram.com/RenyisPolynomial.html

395 名前:132人目の素数さん:2009/05/17(日) 18:46:49
>>387
ストレス下での回答を見るためのテストは2chに向かないと思うが……
面接時に回答できなかった人も、2chでは回答できるような問題だろ?
それって、ここで聞く内容じゃないべ

396 名前:132人目の素数さん:2009/05/17(日) 19:27:02
>>394
どうやって見つけたんだこれ……
いや、ぐぐったとかはなしね。

397 名前:132人目の素数さん:2009/05/17(日) 19:43:47
>>394
dクス
ttp://mathworld.wolfram.com/SparsePolynomialSquare.html
もあった・・・

398 名前:132人目の素数さん:2009/05/17(日) 19:50:34
少し変えて

xの多項式f(x)について、その項数をQ(f(x))とおく。
例) Q(x^3 - 5) = 2、 Q(x^10 + x^8 - 7x + 2) = 4、など

f(x) が Q( f(x)^2 ) < Q(f(x)) を満たす時、Q(f(x))の最小値を求めよ。



----

うん、思い付きだ。俺も答え知らない……

399 名前:132人目の素数さん:2009/05/17(日) 19:56:24
おろ? 新たなネタがきてる
>>397
読ませてもらおうか!

400 名前:132人目の素数さん:2009/05/18(月) 05:06:58
>>398
13と >>397 に書いてあるように見えるが

401 名前:132人目の素数さん:2009/05/18(月) 05:17:38
>>396
いくつかの例はErdosとかの時代に(試行錯誤的に)知られてたんだと。
現在では計算代数の手法を用いて具体的に構成できる。

402 名前:132人目の素数さん:2009/05/18(月) 14:00:48
解けるわけじゃないので、偉そうなこと言えないが釈然としない解決だ。

403 名前:132人目の素数さん:2009/05/18(月) 14:24:19
>>402
なんか綺麗な構造を発見できたら現在でも論文にできるよ。

404 名前:132人目の素数さん:2009/05/18(月) 14:53:40
要するに未解決問題とほぼ同レベルなのね

405 名前:132人目の素数さん:2009/05/19(火) 01:13:26
これの3乗版とかないの?

406 名前:132人目の素数さん:2009/05/19(火) 01:31:02
・3乗だと存在しないけど4乗だと存在する。
・むしろ2^n乗のときのみ存在する。

……みたいになってたら面白いだろうな。

407 名前:132人目の素数さん:2009/05/19(火) 01:57:20
今のところ、どうなってるか分からん。
マジで分からんとしか、言いようがないわけだが……
誰か類似の結果知ってる奴いないか?

408 名前:132人目の素数さん:2009/05/19(火) 02:06:09
つーか、3乗くらいならmathematicaに計算させてみたらどうか

409 名前:132人目の素数さん:2009/05/19(火) 05:08:24
本気かネタ振りかわからんな

410 名前:132人目の素数さん:2009/05/19(火) 06:32:29
麻雀のテンホーとチーホーは、どちらがあがりやすいか

411 名前:132人目の素数さん:2009/05/19(火) 07:41:57
一巡目はジハイが出やすい。

412 名前:132人目の素数さん:2009/05/19(火) 07:48:55
おまえは雀鬼会を敵に回した。


413 名前:132人目の素数さん:2009/05/19(火) 14:39:52
>410 天和

414 名前:132人目の素数さん:2009/05/19(火) 17:39:18
有界で凸な立体Tは、どの平面に正射影しても、その面積が一定である。
このとき立体Tは球であると言えるか。

415 名前:132人目の素数さん:2009/05/19(火) 21:21:39
テンホーとチーホーが同時に入った場合、あがれるのはテンホーのほうだからな。
しかしそれがどれほどの差というのかwww


416 名前:132人目の素数さん:2009/05/19(火) 22:53:05
>>414
言えるが、どう証明したものか…

417 名前:132人目の素数さん:2009/05/19(火) 23:11:45
>>415
親はランダムに切るわけではなく、あくまでの自分の不要牌しか切らないので
そんな単純な話じゃない

418 名前:132人目の素数さん:2009/05/19(火) 23:18:52
>>416
本当に言えるんかな?
正三角形をふくらませたような図形で高さが一定なのがあるけど、
あれって正射影させると長さが一定だろ?
あれの立体版みたいなのってないのかな?

419 名前:132人目の素数さん:2009/05/19(火) 23:51:37
定幅曲線ね

420 名前:132人目の素数さん:2009/05/20(水) 00:07:03
>>389 の根を求む。


421 名前:132人目の素数さん:2009/05/20(水) 00:17:12
>>418
「正射影の面積一定」ではなく、
「幅一定」ということであれば、
ルーローの三角形に似た四面体っぽい図形が作れる。
(ただし、単に4点を中心とした球の交わりではなく、エッジ部分を削って調整する必要あり)
面積一定だと、制約がきつ過ぎるから、球しかなさそうだが、証明は知らん。

422 名前:132人目の素数さん:2009/05/20(水) 00:17:21
>>420

x + 1/x = t とおくと、x^2 -1 + 1/x^2 = t^2 -3,

1 +(√2)x -x^2 +(√2)x^3 + x^4 = (x^2){t^2 +(√2)t -3},
 t = (-1±√7)/√2,
 x = -(1+√7)/2 ±(1/2)7^(1/4),

 1 -(√2)x -x^2 -(√2)x^3 + x^4 = (x^2){t^2 -(√2)t -3},
 t = (1±√7)/√2,
 x = (1+√7)/2 ±(1/2)7^(1/4),

近似値 0.475679456437866… と 2.10225601813565…

423 名前:132人目の素数さん:2009/05/20(水) 01:24:20
>>415
本来のチーホーか今風かによるかも

424 名前:132人目の素数さん:2009/05/20(水) 03:44:45
>>415
地和が成立しないのは、なにも親が天和したときだけに限らない。


425 名前:132人目の素数さん:2009/05/20(水) 04:03:58
どうでもいいが、役満がらみのローカルルールって、洒落にならんよな。
事前に全部確認なんてほぼ不可能だし。

「ロン!人和!」
「そんな役ねーよ。なんだそれ役なしじゃん。チョンボね」

426 名前:132人目の素数さん:2009/05/20(水) 04:07:33
>>425
雀荘なら事前に確認できるようになってるだろ。
個人でやる場合は知らん。 

実施に打つと
役満がらみじゃないローカルルールのほうが
はるかに洒落にならん。

427 名前:132人目の素数さん:2009/05/20(水) 18:08:11
>>414
未解決問題?

428 名前:132人目の素数さん:2009/05/20(水) 18:36:42
>>421
> (ただし、単に4点を中心とした球の交わりではなく、エッジ部分を削って調整する必要あり)

( ゚∀゚)<詳しく聞こうか!

429 名前:132人目の素数さん:2009/05/20(水) 19:59:15
>>428
このへんで発見
ttp://www.geocities.co.jp/Technopolis-Mars/4319/quasi_Rouleau/quasirurotetragon.html

430 名前:132人目の素数さん:2009/05/21(木) 14:57:29
ある有界で凸な立体Tは、どの平面で断面図をとっても、断面が相似である。(ただし、点になる場合や、断面が存在しないケースは除いて考える)
このとき、立体Tは球であると言えるか。

431 名前:132人目の素数さん:2009/05/21(木) 22:57:33
>>429
( ゚∀゚) <かたじけのうござる!

432 名前:132人目の素数さん:2009/05/24(日) 16:51:28
>>309
1963年度京大に似たような問題が出されてたな。

433 名前:132人目の素数さん:2009/05/24(日) 22:38:32
随分昔の話だな。

いろんな意味で。

434 名前:132人目の素数さん:2009/05/25(月) 00:35:44
>>432->>433
詳しく!

その昔、「大学への数学」の東京出版が、CD-ROMで、
東大と京大の数学の過去問を出してたけど、また出してくれんかな〜

435 名前:132人目の素数さん:2009/05/25(月) 03:46:14
ずっと昔に作ったままお蔵入りにしてたやつ。


W氏は自然数を1から順に次のルールに沿って読み上げていく。

(ルール) 3の倍数と十進法表記で3のつく数字を飛ばす。

すなわち、1、2、4、5、7、…、11、14、…である。このときW氏がn個目に読み上げることになる自然数x(n)をnを用いて表せ。

436 名前:132人目の素数さん:2009/05/25(月) 04:38:33
>>434

こんな問題

「△ABCと△DEFにおいて、AB=DEとし、それぞれの外接円の半径は等しく、また内接円の半径は等しいとする。
そのとき二つの三角形は合同になるか。理由をつけて答えよ」


聖文社より京大50年問題集が出てたので、その本より引用した。

437 名前:132人目の素数さん:2009/05/25(月) 06:39:07
>>436
ありあとん!

438 名前:132人目の素数さん:2009/05/25(月) 06:40:18
>>437
アリアハン

439 名前:132人目の素数さん:2009/05/25(月) 13:02:08
>>435
それなんてナベアツwww


440 名前:132人目の素数さん:2009/05/25(月) 16:03:00
k桁の自然数Nを10進数表記したとき、0でない桁の個数をf(N)とする。
例)N=105502,f(N)=4  、 N=100,f(N)=1など
このとき、f(N^2)<f(N)なるNを一つ求めよ。存在しないならその理由を示せ。

----

あらかじめ言っておくと、解答は用意してない。

441 名前:132人目の素数さん:2009/05/25(月) 17:37:26
>>440
404 そのような問題は存在しません

404^2=163216

442 名前:132人目の素数さん:2009/05/25(月) 18:29:30
k桁の自然数Nを10進数表記したとき

表記する前のk桁って何だろう


443 名前:132人目の素数さん:2009/05/25(月) 18:40:00
24495^2=600005025.


444 名前:132人目の素数さん:2009/05/25(月) 19:05:39
計算機って便利ね

445 名前:132人目の素数さん:2009/05/25(月) 19:30:02
316227766017^2=100000000000102500044289.


446 名前:132人目の素数さん:2009/05/25(月) 19:43:59
14142135623731^2=200000000000001400410360361

447 名前:132人目の素数さん:2009/05/25(月) 20:34:31
>>443-446
うれしそうに書いてるところ申し訳ないのだが
・・・逆なんだけど

448 名前:132人目の素数さん:2009/05/25(月) 20:40:29
何が逆なんだ?

449 名前:132人目の素数さん:2009/05/25(月) 20:42:47
>>447
N=24495
f(N)=5
f(N^2)=4
f(N^2)<f(N)
だけど何が逆?


450 名前:132人目の素数さん:2009/05/25(月) 20:47:29
そういえば問題中のk使ってないな

451 名前:132人目の素数さん:2009/05/25(月) 21:38:49
2^24495=600005025
2^316227766017=100000000000102500044289
2^14142135623731=200000000000001400410360361


452 名前:132人目の素数さん:2009/05/25(月) 21:45:27
なんだその記法

453 名前:132人目の素数さん:2009/05/25(月) 22:04:06
自然数Nを10進数表記したとき、0でない桁の個数をf(N)とする。
例)N=105502,f(N)=4  、 N=100,f(N)=1など
このとき、f(N^3)<f(N^2)<f(N)なるNを一つ求めよ。存在しないならその理由を示せ。


454 名前:132人目の素数さん:2009/05/25(月) 22:12:03
いやです。

455 名前:132人目の素数さん:2009/05/25(月) 22:15:11
>>414
結局、これどうやんの?

456 名前:132人目の素数さん:2009/05/25(月) 22:31:28
f(N^3)<f(N)
これが絶望的

457 名前:132人目の素数さん:2009/05/26(火) 00:28:49
>>453
直感だが、sqrt(10^(6k+1) + 1) - [sqrt(10^(6k+1) + 1)] が十分小さくなるようなkの値をとってくれば良いんじゃないだろうか……
いや、スマン適当。

458 名前:132人目の素数さん:2009/05/26(火) 01:16:27
十進数って書いたほうがいい

459 名前:132人目の素数さん:2009/05/26(火) 06:55:30
>>451みたいな数を研究してたサイトがあったな

460 名前:132人目の素数さん:2009/05/26(火) 06:57:15
ここに書ききれないぐらいでかい数になりそう

461 名前:132人目の素数さん:2009/05/26(火) 09:34:55
>>459
kwsk!

462 名前:132人目の素数さん:2009/05/26(火) 13:08:17
>>461
十進表示で0を含まない自然数Nを二乗した数N^2が0を出来るだけ長く含む(Nの桁数に対する割合で)ものを一覧にしていて、上記の意味で一定以上の長さで0を含むものが有限であるとか示して、リストもあったと思った。ただ、どこにあったか思い出せん。

463 名前:132人目の素数さん:2009/05/26(火) 15:40:00
使う数字が少ない平方数などならここにある。
http://www.asahi-net.or.jp/~KC2H-MSM/mathland/math02/index.htm


464 名前:132人目の素数さん:2009/05/27(水) 01:18:15
追いかけるだけで精一杯だね
てか、追いかけることすらできない…

465 名前:132人目の素数さん:2009/05/27(水) 02:04:41
それは精一杯と表現していいのか?

466 名前:132人目の素数さん:2009/05/27(水) 17:18:04
4×4の正方形のマスにおいて、左下の頂点からスタートし、線上を通り、右上の頂点まで辿り着く道順は何通りあるか。
ただし、最短か否かは問わず、同じ線上を通ってはならないものとする。

467 名前:132人目の素数さん:2009/05/27(水) 17:45:37
>>466
無理

468 名前:132人目の素数さん:2009/05/27(水) 19:44:29
ウィンドウズのフリーセル。並べ替えに失敗すると手詰まりか、
同じカードペアを永遠に並べる事しか出来なくなる。後者の場合、
1ペアで繰り返しになる時と2ペアでの場合がある。さて、このパターンで
「3ペア以上は存在しない」これを証明せよ。証明できるとは限らない。

469 名前:132人目の素数さん:2009/05/28(木) 01:52:40
>同じカードペアを永遠に並べる
意味がわからん

470 名前:132人目の素数さん:2009/05/28(木) 02:21:46
>>469
フリーセルの必殺技

471 名前:132人目の素数さん:2009/05/28(木) 06:51:02
A君がB君をあるゲームに誘った。
A君“B君、ゲームをしよう。二人で1から交互に順番に数字を数えていって100と言った方が負け。一回につき三つまで数を言うことができる。僕が先攻ね”
B君“わかったよ”

何回かやったがA君の全勝
B君“たまには僕の先攻にしてよ”
A君“いいよ、でもちょっとルールを変えて101と言った方が負けにしよう”
B君“いいよ”
やっぱりA君の全勝

A君の戦略を簡単に説明せよ

472 名前:132人目の素数さん:2009/05/28(木) 08:55:51
>>471
自然数という制限もないのに100や101を言ったBがマヌケなだけ

473 名前:132人目の素数さん:2009/05/28(木) 11:45:08
>>471
小学生のときにこれ研究したなー。
↓そのときの解答。

言ったら負けな数字を M (この場合100や101)
増やせる数の下限 a (この場合1)
増やせる数の上限 b (この場合3)
としたとき、
先攻は (M-1)mod(a+b) を宣言。
ただしこれが0の場合は後攻になる。

あとは(a+b)-(相手の増やした数) を増やせばよい。


474 名前:132人目の素数さん:2009/05/28(木) 11:56:53
>>471
100を言えば負けは、すなわち99を言えば勝ち。
先攻を選び99を4で割った余り3から以後7,11,1519,……,99を言えば勝ち。
101を言えば負けの場合は100を言えば勝ち。
100を4で割った余りは0だから、後攻で4,8,12,……100を言えば勝ち。
常識だね。

475 名前:132人目の素数さん:2009/05/28(木) 13:10:38
【先手必勝なゲームを探せ!!】
http://science6.2ch.net/test/read.cgi/math/1234151632

476 名前:132人目の素数さん:2009/05/28(木) 15:13:03
「二人で1から交互に順番に数字を数えていって100と言った方が負け。」なんだから
口には出さず100や101を心の中で数えればよい。負けなど無くなる!

477 名前:132人目の素数さん:2009/05/28(木) 15:18:02
>>476
意味が分かりません。

478 名前:132人目の素数さん:2009/05/28(木) 22:42:32
>>468
http://www.dotup.org/uploda/www.dotup.org72587.bmp.html
3ペアってこうですか?わかりません

479 名前:132人目の素数さん:2009/05/29(金) 21:52:49
>>466
こんな問題、解けるやついるの?

480 名前:132人目の素数さん:2009/05/29(金) 22:22:44
>>466
色々と計算式を考えてみてもどこかで同じルートだったり抜けがあったりで上手く行かない。
難しいからプログラムでも組んでみようかと思ったがどういうアルゴリズムにすべきか・・・
虱潰しで行くにしても判定が難しいし一回判定したルートを保存するのも大変。

481 名前:132人目の素数さん:2009/05/29(金) 23:01:52
定石で行けば頂点に適当に順番をつけて、幅優先探索か深さ優先探索だなぁ。


482 名前:480:2009/05/29(金) 23:45:49
通る道と通らない道の組み合わせを全通りチェック。
一筆書きの要領で開始点と終了点以外で交点が奇数の部分があれば除外。

というプログラムを組んだが異様なほど時間がかかる。
しかも何か間違えてそうで怖い。

483 名前:480:2009/05/29(金) 23:51:19
まあ3流プログラマなんでお世辞にも効率の良いアルゴリズムとは言えないんだが・・・
このペースだと終わるまで3時間くらいか。

組み合わせ自体が(2の20乗)×(2の20乗)だし。

・・・これ間違ってないよね?

484 名前:132人目の素数さん:2009/05/29(金) 23:55:13
>>483
対称性から、始めの一歩を指定しておけば、半分になるぞ!

485 名前:480:2009/05/29(金) 23:58:23
>>484
おお、そうだった!
サンクス!

処理が終わったらまた書き込む。
合ってるって確信できないのがアレだけどw

486 名前:480:2009/05/30(土) 00:18:13
たった今欠陥に気付いた。

組みやすくするために左上から右下というルートで考えてたんだが、

┯━┯━┯━┯━┓
│  │  │  │  ┃
├─┼─┼─┼─┨
│  │  │  │  ┃
├─┼─┼─┼─┨
│  │  │  │  ┃
┏━┓─┼─┼─┨
┃  ┃  │  │  ┃
┗━┛─┴─┴─┨

これでもカウントされてしまう。

つながってるかも判定入れないと・・・

もうギブアップ。

487 名前:132人目の素数さん:2009/05/30(土) 00:36:42
諦めたらそこでry

488 名前:132人目の素数さん:2009/05/30(土) 00:42:01
∠ABC=24°の菱形ABCDがあって、
線分BCのC側の延長上に点Eを、∠CDE=30°となるようにとるとき、
∠DAE=30°となることを証明してちょ。

489 名前:132人目の素数さん:2009/05/30(土) 01:39:48
>>480
漏れのプログラムでは184通りとでた。
計算時間は0.2sec

490 名前:489:2009/05/30(土) 01:41:57
あ、しもた。
4x4の解釈間違ってるしw
3x3でやっちゃった。

491 名前:489:2009/05/30(土) 01:44:56
4x4でやり直したら8512通り、計算時間は6.1secになった。


492 名前:132人目の素数さん:2009/05/30(土) 01:59:07
>>491
プログラムの素人に分かるように解説期盆濡!

493 名前:132人目の素数さん:2009/05/30(土) 02:07:00
>>480
同じ組合せでも道順は複数あることはわかってるよな?
...って、もうギブしてたかw

494 名前:132人目の素数さん:2009/05/30(土) 02:07:52
オンライン整数大辞典様にはばっちり収録されてた

495 名前:132人目の素数さん:2009/05/30(土) 02:08:01
単純な幅優先探索しか使ってないっす。


496 名前:491:2009/05/30(土) 02:22:17
ヒントだけ言うと用意するクラスは
頂点、ルート、ルートの集合の3つ。

それぞれのクラスに用意したメソッドは

頂点

その頂点に隣接する頂点の集合を返す関数
その頂点が4x4のマスの中にあるかどうかを返す関数


ルート

自分自身から一つ頂点を増やしたルートの集合を返す関数
同じ頂点を2度、通ってないかを返す関数
ゴールにたどり着いたかを返す関数


ルートの集合

自分が持っているそれぞれのルートに対して一つ頂点を増やしたルートを集めたルートの集合を返す関数


後は自分でアルゴリズムを組み立てて見れ。

497 名前:132人目の素数さん:2009/05/30(土) 02:40:00
それは同じ頂点を通らないもの。


498 名前:132人目の素数さん:2009/05/30(土) 02:42:11
>>496
>同じ頂点を2度、通ってないかを返す関数
とあるが、今回は同じ頂点は2度通ってもいいというルールと認識しているぞ。
2度通ってはいけないのは「同じ道」
「辺の集合」を持たないといけないから結構面倒。


499 名前:491:2009/05/30(土) 02:43:20
あ、なるほど〜
やけに簡単な問題に悩んでるなと思ったら、漏れが問題勘違いしてたか。



500 名前:491:2009/05/30(土) 03:00:32
一応、プログラム書き直して3x3でやったら800通りになったけどみんなの結果と一致してる?
4x4は結構計算時間が掛かりそうだ。



501 名前:132人目の素数さん:2009/05/30(土) 03:19:37
通る道は同じでも道順が違うのも考慮しなきゃいけないのか・・・
ℓとΩみたいなので。

わかりづれえorz

502 名前:491:2009/05/30(土) 03:20:36
幅優先探索なもんだからメモリ食い尽くしちまったぜw
ヌルポきたこれwww




503 名前:132人目の素数さん:2009/05/30(土) 03:22:36
NP完全・・・だと・・・・????

504 名前:132人目の素数さん:2009/05/30(土) 03:30:00
数え終わったのを記憶しておく必要がないから深さ優先で。


505 名前:491:2009/05/30(土) 04:03:02
ヌるぽに切れてC++で書き直してやった。
漏れのプログラムでは4x4の結果は323632通りになったよ。
計算時間は9秒。
アルゴリズムは同じく幅優先。
あってるかどうかは知らね。





506 名前:132人目の素数さん:2009/05/30(土) 04:06:04
オンラ(ry
によると合ってる
素辺な経路っていう用語があるのね

507 名前:491:2009/05/30(土) 04:15:23
>>506
さんくす。コレで気持ちよく寝れるw
ま、>>492は幅優先探索とか深さ優先探索は勉強しといて損はないぞ。



508 名前:132人目の素数さん:2009/05/30(土) 04:35:57
>>466 の問題について

準備:
2×2の正方形の連結 (田の字) で考える。
頂点に名を付ける 。 左下から 右へ ABC、中段左からDEF 、上段左からGHI
Aがスタート、Iがゴールである。
スタートからゴールまで同じ道を2度以上通らずに辿る道順を正規のルートと呼ぶ。
問題は、そのような道順が何種類あるのかを数えることである。

GHI
DEF
ABC

道順 ABEHGDEFI は 正規のルートである
道順 ABEDGHEFI は 正規のルートである

このふたつのルートは、道「順」である以上、区別されるべきか?
それとも、通る道は同じなので、同一視するべきか?

509 名前:132人目の素数さん:2009/05/30(土) 04:37:08
あ、すまん。501が先に書いてた。

510 名前:132人目の素数さん:2009/05/30(土) 05:58:30
面倒なのでperlでw
しらみつぶしに樹型図を描くようなイメージの素朴なアルゴリズムで探すと、
(なんせperlなんで)数分かかったが、同じく323632通りという結果が出た。

基本的には、現在見ている経路と、その経路上でまだ試していない分岐だけ
覚えておけばいいので、メモリーは全然食わないし、真面目にcとかで書けば
時間もそんなにかからないとは思うが。

ちなみに、2×2は16、3×3は800。
これらは経路リストも出したが、やたら長い経路をたどってみると
なんかPipeDreamみたいになってて笑える。

511 名前:132人目の素数さん:2009/05/30(土) 06:03:15
4*4の計算が1913年後に終わるorz
おやすみなさい

512 名前:132人目の素数さん:2009/05/30(土) 06:40:55
んじゃ、n×nの素辺な経路の
経路の長さの最大値は2n^2であること、および、
n≧3の場合は最長経路の描く図形が対角線について対称な2種類しかないこと
を示せ

ってのはどう?

513 名前:132人目の素数さん:2009/05/30(土) 06:52:12
>>512はちょっと間違ったので書き直し。

n×nの素辺な経路の長さの最大値は2n^2であること、および、
n≧3の場合は、最長経路の描く図形が、向きの違うものを区別すると
ちょうど4種類あることを示せ。

514 名前:132人目の素数さん:2009/05/30(土) 10:26:23
>>510
Oops!!
Well done.

515 名前:132人目の素数さん:2009/05/30(土) 10:52:24
>n×nの素辺な経路の長さの最大値は2n^2であること
nが偶数の場合は出来たけど、奇数の場合は評価が少し面倒そうだな。

516 名前:132人目の素数さん:2009/05/30(土) 11:44:22
http://www.research.att.com/~njas/sequences/A013990

517 名前:132人目の素数さん:2009/05/30(土) 12:11:56
http://www.research.att.com/~njas/sequences/A134847

このサイト便利だな

518 名前:132人目の素数さん:2009/05/31(日) 04:34:51
n×mができるやつ。 ただし5×5くらいが限界。
それ以上は、経路数が32ビットを超えるのでlong long int をつかうとかして。
4×4がPemM 1.6GHzで5秒くらい。5×5はたぶん半日以上かかると思う。

#include <stdio.h>
#define DX 4
#define DY 4
void main(void)
{
int r[DX*(DX+1)+DY*(DY+1)],m[2][DX+1][DY+1],c=0,x,y,p,n;
memset(r,0,sizeof(r));
for(;;){
memset(m,0,sizeof(m));
for(x=y=p=0;p<DX*(DX+1)+DY*(DY+1);p++){
if(r[p]&1){
 if(r[p] 2){
  if(--y<0||!(m[1][x][y]^=1))break;
 }else if(!(m[1][x][y]^=1)||++y>DY)break;
}else if(r[p]&2){
 if(--x<0||!(m[0][x][y]^=1))break;
}else if(!(m[0][x][y]^=1)||++x>DX)break;
if(x==DX&&y==DY){
 c++;
 break;
}
}
for(n=p;n>=0;n--)if((r[n]=((r[n]+1)&3))!=0)break;
if(r[0]==(DX==DY?1:2))break;
}
printf("%d",c*(DX==DY?2:1));
}


519 名前:132人目の素数さん:2009/05/31(日) 08:04:36
× if(r[p] 2){

○ if(r[p]&2){



520 名前:132人目の素数さん:2009/05/31(日) 13:05:45
1x1 2
1x2 4
1x3 8
1x4 16
1x5 32
2x1 4
2x2 16
2x3 72
2x4 335
2x5 1562
3x1 8
3x2 72
3x3 800
3x4 9754
3x5 121130
4x4 323632
4x1 16
4x2 335
4x3 9754
4x5 11171466
5x1 32
5x2 1562
5x3 121130
5x4 11171466
5x5 1086297184
計算の結果

521 名前:132人目の素数さん:2009/05/31(日) 13:50:00
http://groups.google.co.jp/group/rec.puzzles/browse_thread/thread/b17ee576f4040e95/3e0fdb6a4a6d8379

522 名前:468:2009/06/01(月) 03:16:34
>>478
消されてて分からないけど、実際にプレイして3ペアに到達したのなら反例を
見つけた事になるね。次の問題は当然、特定カード内での操作しか出来なくなる
、その最大枚数は? という事だけど。現在値は6ですか。(未確認情報)
個人的には4 しか見てない。かなりやってるけど見ないね。

523 名前:132人目の素数さん:2009/06/01(月) 05:15:21
>>522 
ゲームを終わらせようとしてると3ペアにはまずならない
わざと3ペア以上を作ろうとしないと

524 名前:468:2009/06/01(月) 09:27:05
>>523
それでも作ろうとして作れるのならそれが法則なんで。ただし、フリーセル
を基準にした場合を想定してるのでこの証明は相当に厄介だと思われ。
最大で64000位のゲームが存在するけど、トランプの順列からすれば
ほんの一部でしか無いからね。

525 名前:132人目の素数さん:2009/06/01(月) 10:28:13
>>524
何が言いたいのかよくわからない。

>>523は522の 「かなりやってるけど見ない」というのに返しただけなんだが
522がわざと作ろうとしてやっていてできないというのなら申し訳ないが





526 名前:468:2009/06/01(月) 10:35:54
>>525
まず、面白い問題とは思うが自分はその答えを知らない。ココ重要。
だから経験則から言える事を言ってるだけ。証明はしたい人もしくは出来る人が
してくれ。そういう事。

527 名前:132人目の素数さん:2009/06/01(月) 11:19:00
「実際にプレイして」3ペアに到達する必要はあるのか?

528 名前:468:2009/06/01(月) 11:41:28
>>527
さあ・・。でもフリーセルはごく一部のゲームを除いてほとんど全てが
クリア可能になってる。ここで問題なのはクリアできずに終わってしまう
手順。しかし実際にはクリア出来ない初期設定の方が膨大と思われ。
しかし、それはこの証明には関らない。何故ならそれはフリーセルの仕様に
反するから。

529 名前:468:2009/06/01(月) 11:49:14
それにわざと3ペアを残そうとしても相当に苦労すると思うよ。やってみれば
分かるがクリアする方がむしろ簡単かも。将棋みたいなゲームもそう。
わざと負けるのが簡単なのは相手が勝とうとしてる場合だけ。

530 名前:132人目の素数さん:2009/06/01(月) 11:49:56
>  しかし実際にはクリア出来ない初期設定の方が膨大と思われ。
?
なぜそう思う?

> しかし、それはこの証明には関らない。何故ならそれはフリーセルの仕様に 
反するから。 

仕様に反するの意味がよくわからないのだが
フリーセルのルールに
カードの乱数(のようにみえるもの)による初期配置に関するルールも
加えようと言うことなのか?

トランプよく切って、フリーセルの初期状態のようにテーブルに並べたものは
フリーセルのようなものであって、フリーセルのルールに反すると?

531 名前:132人目の素数さん:2009/06/01(月) 12:01:27
> それにわざと3ペアを残そうとしても相当に苦労すると思うよ。

何故そう思う? 
http://hp.jpdo.com/ff02/148/img/35.jpg

532 名前:468:2009/06/01(月) 12:11:03
>>530
例えばクリアできないのはゲームNO.−1、−2
普通にカードを順序良く並べただけだが、ルール通りにやっても絶対に無理。
という事は、ここから発生する順列は全てアウトという事。それは別の見方を
するとクリアされた状態から逆操作をして52!通りの初期値を作ることが
出来ない事を意味してる。
>トランプよく切って
それはダメ。あくまでも正規のフリーセルゲーム(NO.を指定できる)で
ないと。

533 名前:468:2009/06/01(月) 12:18:21
それともう一つ条件を付ける必要があった。「フリーセル」は
4つとも埋まっていて空かないものとする。

534 名前:132人目の素数さん:2009/06/01(月) 12:18:53
>>532
> ここから発生する順列は全てアウトという事。

ここから発生する順列ってなんのこと? 

http://hp.jpdo.com/ff02/148/img/36.jpg

535 名前:132人目の素数さん:2009/06/01(月) 12:21:42
> それはダメ。あくまでも正規のフリーセルゲーム(NO.を指定できる)でないと。

ではフリーセルが指定したナンバーを元にトランプの並びをどう決定しているのかを教えてくれないかな。
でないと、「フリーセルが並べるパターン全て」という証明はできないよ。

http://hp.jpdo.com/ff02/148/img/37.jpg



536 名前:132人目の素数さん:2009/06/01(月) 12:23:01
>>533
フリーセルがひとつでも開いていれば手詰まりではないのだから
その条件は要らないと思うが?

537 名前:132人目の素数さん:2009/06/01(月) 12:23:53
4ペア ワロス。

538 名前:468:2009/06/01(月) 12:33:41
>>535
君の思考が理解出来ないが、それを知る必要はないでしょ。知ったところで
ゲーム内容は変化しないよ。
>>536
その条件は問題を単純化する為に敢えて必要。そうしないとセルが空いていても
クリア不能な状態を定義する必要があるからね。

539 名前:132人目の素数さん:2009/06/01(月) 12:36:30
3ペアやその変形や4ペアを作ってみたが
3ペアが作れないパターンのほうが少ないように感じるよ。

何度もやってるのにできない、というのは作る気がないのか
よほど操作が下手なのかのどちらかだと思う。

540 名前:132人目の素数さん:2009/06/01(月) 12:37:24
>>538
> それを知る必要はないでしょ

それがわからない状態で、どうやって全てのパターンについての証明をするのか?

541 名前:132人目の素数さん:2009/06/01(月) 12:39:29
フリーセルには幾つかのバージョンがあるようだけど
全てのフリーセルで、同じナンバーを入力すると同じパターンで始まるの?

542 名前:132人目の素数さん:2009/06/01(月) 12:40:54
>>538
>>536の言うように「ひとつでも空きがあるなら手詰まりでない」の定義で十分だと思うが。

543 名前:132人目の素数さん:2009/06/01(月) 12:43:04
これだけ反例を貼られてもなっとくしない468ってなんなの?

544 名前:468:2009/06/01(月) 12:50:18
>>542
何故そういう風に考えるのか分からないけど、どんな状態、組み合わせであろうと
そこからクリア出来ないのならそれは手詰まり(の一種)ですよ。

545 名前:132人目の素数さん:2009/06/01(月) 12:52:19
>>524
> 最大で64000位のゲームが存在するけど

うちのフリーセルは1〜1000000の数が指定できるんだけど
結構かぶってるの?


546 名前:132人目の素数さん:2009/06/01(月) 12:54:23
>>544
だからそれはそう定義するからでしょ。 よくわからん人だね。 
定義の問題だと言ってるのに別の定義で反論されても意味ないでしょ。

547 名前:132人目の素数さん:2009/06/01(月) 12:58:53
で、反例はもう納得できたのか?
それとも自分でやったわけじゃないから未確認か?

548 名前:468:2009/06/01(月) 13:05:32
>>545
そうなんだ。一応ですね、M$版のものを想定してますんで。Vista
とか7ではそうなってるのかな。その辺の事情は知らないんでスマン。
>>546
分かりました。では538の条件は忘れて下さい。
3ペアが作れるという人、暇ならその手順を書いてくれれば有り難いです。
例えば S9(スペード)up,HA + (home cell),D3→C4
みたいな感じで。ゲームNO.は32000以下頼みます。

549 名前:545:2009/06/01(月) 13:13:11
>>548
うちはXPで、それについていたフリーセルが100000まで指定できるんですが。



550 名前:468:2009/06/01(月) 13:17:48
>>549
うちは2000なんで32000までです。でもNO.を指定して
同じNO.で同じゲームになるなら基本的には同じだと思います。
勘違いを誘ってるようですが自分は面白そうな問題を出しただけで
自分でそれを解決出来るなどとは言ってませんのでよろしく。

551 名前:132人目の素数さん:2009/06/01(月) 13:30:28
ゲーム#3644で以下のキーを入力
6523121380808080468400400804606060


552 名前:468:2009/06/01(月) 13:34:59
>>551
納得しました。私の仮定は完全に誤りでした。
連打スマンこってしたorz

553 名前:132人目の素数さん:2009/06/01(月) 14:08:24
一段落したところで、暇な人は>>488でも考えてくれ

554 名前:468:2009/06/01(月) 19:39:48
>>539
負け惜しみで言うけど551みたいなあり得ない操作は下手ですよええ。
でも普通にクリアするスピードはここいらの住人ではトップクラスと思ふ。
数学とは関係無いですよはい。

555 名前:132人目の素数さん:2009/06/01(月) 19:54:44
>551みたいなあり得ない操作

普通にゲームをクリアするときとは目的が違うのだから
その目的のためには>>551の操作はあり得ないどころか
大変的確な操作でかつ特にトリッキーな操作でもないと思うがどうか。

手詰まりペアを多く作るという目的のときに
どこに着眼してどのような思考をすればいいかすら
考えていないのではないか?

556 名前:132人目の素数さん:2009/06/01(月) 20:04:01
>>554
998連勝中。


557 名前:468:2009/06/01(月) 20:04:12
>>555
ま、通常必要無い思考なので。クリアを目的としても的確な思考は要る。
それでも全ゲームを一発クリアできる人間がいるとは思えないし。
それも、思えないというだけの話でいないという証明は当然無いね。

558 名前:468:2009/06/01(月) 20:11:01
>>556
強いですね。NO.1から順番にといて30連勝までしてやめますた。
そういうコツコツ作業が嫌いなので。

559 名前:132人目の素数さん:2009/06/01(月) 20:55:32
>>557 
554や550あたりでおまえの言っていることを要約すると
「自分で考える気もない問題を、面白い問題だと思って出した」と言っているんだが
そういうのは読んでいるほうからするとどう感じると思う?



560 名前:468:2009/06/01(月) 21:12:28
>>559
別にあんたに説教される筋合いは無い。この問題も、もしかしたら面白いかも知れない。
考えるのは好きでしょ? 自分は得意ではないので。自分が難しいと感じる
事を瞬時に理解する人間もいるっしょ。だいたいこのスレとか、そういう
趣向が無ければ意味無いジャン。

561 名前:132人目の素数さん:2009/06/01(月) 21:33:40
こんなアホにかまってないで次行こうぜ

562 名前:132人目の素数さん:2009/06/01(月) 21:39:23
何か面白い趣向でもあるのかと思った人が突っ込みを入れてるのに
まるで理解していない>>468

何もないことがわかったので>>559は怒っているわけだが
気づくの遅すぎだよ

563 名前:468:2009/06/01(月) 21:39:47
そうそう、次へ行かないと。

564 名前:132人目の素数さん:2009/06/01(月) 21:41:48
普通は何か準備をして出題するよなあ

565 名前:132人目の素数さん:2009/06/01(月) 21:46:32
最初の日本語が変な点でスルーすべきだったな

566 名前:132人目の素数さん:2009/06/01(月) 21:50:31
これに懲りたらアホな問題出すなよ。

567 名前:132人目の素数さん:2009/06/01(月) 22:18:14
468からは、フリーセルに対する妙なプライドが垣間見られる。
>>557なんて、>>555への返答になってないし。

フリーセルを「クリア」する能力と、具体的な問題に対して
「反例」を構成する能力は全くの別物でございます。

568 名前:132人目の素数さん:2009/06/01(月) 22:23:42
今更なんだがローカルルールで
・出題者は回答を用意してから書き込むこと
とか付け足しちゃどうだろう。

569 名前:132人目の素数さん:2009/06/01(月) 22:26:09
過去ログまとめにスレ上での未解決問題もちらほらあるのを考えると
兼ね合いが難しいね

570 名前:132人目の素数さん:2009/06/01(月) 23:30:43
・出題者は自分で面白いと思っていない考える気もないような問題は出さないこと

これだけで馬鹿よけには十分だろ。

571 名前:132人目の素数さん:2009/06/01(月) 23:40:05
>>567
> >>557なんて、>>555への返答になってないし。 

してるよ。

> ま、通常必要無い思考なので。

「考えていないのではないか? 」 きかれて、そう答えたら
考える気もないという返事だろう。


572 名前:559:2009/06/02(火) 00:23:28
>>562
> 何もないことがわかったので>>559は怒っているわけだが 

いや何もないことで怒ったりしたわけではなくてな。
そこに書いてあるとおり
「自分で考える気もない問題を、面白い問題だと思って出した」
と平気書いてしまうことを、どう思っているのかな? と聞いただけなんだ。
そうしたら説教だと思ったらしい。
なるほど自分でも後ろめたい気持ちはもってるんだな。

573 名前:468:2009/06/02(火) 00:40:23
おれって人気者だな。君達ね、いつまでもくだらない事でネチネチ言わない!
そんなんだから就職先無いとかなるんですよ。じゃあ考える気も無い、
というか、考えても理解出来ないからそうなるんだが、必殺の問題を出そう。
フリーセルがクリア出来る初期値は52!の内のどれだけ?
その理由まで言えたらすごいね!

574 名前:132人目の素数さん:2009/06/02(火) 00:48:27
まあもう誰も相手にしないだろう。

575 名前:132人目の素数さん:2009/06/02(火) 00:49:24
573は
>・出題者は自分で面白いと思っていない考える気もないような問題は出さないこと
これが読めないらしい。

576 名前:468:2009/06/02(火) 00:50:36
>>574
そういう態度は感心しない。人をコキ降ろすのは熱心でいざ挑戦されると
ハァ? ですか。少なくとも、お前のためにそこまでの労力は払えないくらいの
言葉は欲しいね。

577 名前:132人目の素数さん:2009/06/02(火) 00:50:37
>>488 は 中学範囲で解かないといかんのか?
三角関数もありなの?

578 名前:132人目の素数さん:2009/06/02(火) 00:51:44
>>576
>>570

579 名前:132人目の素数さん:2009/06/02(火) 00:51:46
自分が考える気も起こらない問題で挑戦とはすごいな。

580 名前:132人目の素数さん:2009/06/02(火) 00:52:06
ここはトリビアの種ではありません

581 名前:468:2009/06/02(火) 00:53:09
まぁ、おれが思うに君らがその超優秀な頭脳を働かせるまでもなく、
世界中の誰かがもうその答えを出してるとは思う。今考えた。

582 名前:468:2009/06/02(火) 00:56:29
>>579
言うまでも無いが、この種の挑戦とは誰かに対する挑戦ではなく、
存在する問題に対する挑戦。興味が無いのなら取り組む必要が無いのは当然。

583 名前:132人目の素数さん:2009/06/02(火) 00:57:22
お前、ポアンカレ予想の証明わかってんの? 谷村志村予想の証明は? 
これは挑戦なんだからちゃんと解いてよ。
オレは知らないし興味もないから解かないけどね。 

とでも言っとけば相手にしてもらえるんだろうか?

584 名前:468:2009/06/02(火) 01:02:01
おれはプロフェッショナル・フリーセラーとしてこのゲームの
内奥にある法則に興味がある。君達のうちの何人がどのNO.でもいい、
60秒以内にクリア出来ると言うのか。まぁ君らの学者じみた脳味噌じゃ
キビシイだろうな。

585 名前:132人目の素数さん:2009/06/02(火) 01:02:42
みんな、付き合いいいな。

586 名前:468:2009/06/02(火) 01:05:04
>>583
いったい誰がそんな高尚な話題を振ってるんですか(笑
たかがトランプの順列組み合わせですよ。もういいです。
後は自分で考えます。

587 名前:132人目の素数さん:2009/06/02(火) 01:09:28
>>585
ばかはいらうとおもしろい

588 名前:132人目の素数さん:2009/06/02(火) 01:10:00
やはりフリーセルに対して「ヘンチクリンな」プライドを持っていた468www
よほど悔しいのかね。

>>584
フリーセルの問題を「クリア」する能力は、フリーセルで成り立ちそうな性質を
証明(もしくは成り立たないことの証明)するのに役に立たない。むしろ、
種々の問題を考えたときに反例を見つけ出す能力の方がずっと役に立つ。

>君達のうちの何人がどのNO.でもいい、60秒以内にクリア出来ると言うのか。
>>555で指摘されてるのに、まだ分かってないのか?
「クリア」に拘るのはナンセンス。いくらクリアできたって、
定理は証明できない。反例を見つけ出すことも出来ない。

>まぁ君らの学者じみた脳味噌じゃキビシイだろうな。
その「学者じみた」脳味噌でないと、定理を証明することも
反例を見つけ出すことも出来ない。

589 名前:132人目の素数さん:2009/06/02(火) 01:11:33
プロフェッショナルフリーセラーというのは誰が給料をくれるんですか?

590 名前:468:2009/06/02(火) 01:18:02
>>589
いい質問だ。金の問題ではない。自称という事だな。無論自分がトップとか
そういう事を意味しない。世の中上には上がいるからな。例えば
一番有名なナンバー1941。 君達も黙ってこれをプレイしてみろ。
このゲームの単純ながら奥深いその魅力を感じられるだろう。
>>588
いいから君は論文の続きでも書き給へ。

591 名前:132人目の素数さん:2009/06/02(火) 01:30:05
>>468はマインスイーパーやフリーセルに興味あるらしいから聞きたいんだが、
これらのプログラムって必ず解決出来るような初期状態を定めるように設計されているのか?
途中で解決不能な状態に陥ることがないように設計されているのか?
マインスイーパーやってて思うんだが、何故いつも最初に自爆することがないのか不思議でならない。


592 名前:468:2009/06/02(火) 01:35:18
>>591
まぁそれほど詳しいわけでも無いが、フリーセルで今のところ
解決不能と言われてるのは一つしか無い。おれが挙げたー1とー2は除いて。
ググればすぐに出る。それとマインスイーパーで最初に爆発するかどうかは
OSのバージョンによる。

593 名前:132人目の素数さん:2009/06/02(火) 01:37:43
> 1941
そんなものもう10年も前の話題なのだが。

594 名前:468:2009/06/02(火) 01:41:22
>>593
名作は時間が経っても色褪せない。だいたいおまえはサクッとクリアしてから
言ってるのかと。

595 名前:132人目の素数さん:2009/06/02(火) 01:44:53
10年前に解いたおぼえがあるよ。

596 名前:468:2009/06/02(火) 01:48:39
因みに1941というのは太平洋戦争の開戦の年だ。理系には分かり難いだろw

597 名前:132人目の素数さん:2009/06/02(火) 01:54:17
どうして理系だと解りにくいんだろう?


598 名前:468:2009/06/02(火) 01:57:48
>>597
そりゃキミィ! 歴史の年号とか覚えなくても合格できるからだろ。
科目に無いところもあるし、傾斜配点でどうでもよくね的な扱いだし。

599 名前:132人目の素数さん:2009/06/02(火) 03:28:18
態態覚えんでもそのくらい日本に暮らしてれば知ってるだろ

600 名前:132人目の素数さん:2009/06/02(火) 03:41:40
くまくま〜
たいたい〜
わざわざ〜

601 名前:132人目の素数さん:2009/06/02(火) 06:36:07
理系にコンプレックスでもあるのかね

602 名前:468:2009/06/02(火) 08:28:27
>>601
そうやっていちいち人をプロファイリングするのを止めんかい気持ち悪い。

603 名前:132人目の素数さん:2009/06/02(火) 09:11:11
>>601
カーッ(゚Д゚≡゚д゚)、ペッ

604 名前:132人目の素数さん:2009/06/02(火) 09:33:05
理系以外の方とか良スレ荒らさずに雑談スレやVIPでやってくれませんか?見るに堪えませんよ

605 名前:132人目の素数さん:2009/06/02(火) 15:39:40
理系の人間は誰でも知ってるが、文系の人間はほとんど知らないことは、いろいろある(ネピア数とかシュレーディンガーの猫とか)けど
文系の人間は誰でも知っていて、理系の人間はほとんど知らないことって何?
そんなのないような気がするんだけど。

606 名前:132人目の素数さん:2009/06/02(火) 16:55:45
>>605
流石に文系を馬鹿にしすぎでね?
重商主義学説とか全体主義体制とか構造依存性とか。
俺も言葉だけで正確に内容把握してない。

政治経済にそういうの多い。

607 名前:132人目の素数さん:2009/06/02(火) 16:58:29
>>605
会計関連で、たとえば漢字の読み方があるかもしれない。

上代・下代とか、前渡金、前受金とか。
あともいっこくらい、この手の話のタネになりそうなの単語があったが、忘れた。

608 名前:132人目の素数さん:2009/06/02(火) 17:04:13
>>607
「しゅつのう」じゃない?

ところでそろそろ話題を数学の面白い問題に戻そうよ。

609 名前:132人目の素数さん:2009/06/02(火) 17:05:03
>>605
理系でもシュレーディンガーの猫をちゃんと理解できてるのなんか半分もいないだろ。
聞いたことがあるくらいなら文系にもいくらでもいるしな。

610 名前:132人目の素数さん:2009/06/02(火) 17:06:09
>>488 は?

611 名前:132人目の素数さん:2009/06/02(火) 17:34:05
そのくらい理系と言う学問は幅が狭く
文系と言う学問は幅が広いと言うことだよ

612 名前:132人目の素数さん:2009/06/02(火) 17:53:46
そもそも理系や文系などとカテゴリ分けするのはナンセンスだがな

613 名前:132人目の素数さん:2009/06/02(火) 17:55:42
科学かどうかで分けるのがよいと思う。

614 名前:132人目の素数さん:2009/06/02(火) 18:43:31
スルーされっぱなしで、たまに(多分出題者から)推薦レスが来る>>488
そろそろ解いてみようか?

問題文転載

∠ABC=24°の菱形ABCDがあって、
線分BCのC側の延長上に点Eを、∠CDE=30°となるようにとるとき、
∠DAE=30°となることを証明してちょ。

615 名前:132人目の素数さん:2009/06/02(火) 21:41:19
>>611
『そのくらい』が何を指してるのかわからないw
三流大の文学部出身者で重商主義やら減価償却やらを全く説明できない者もいるw
要するに、文系では誰もが知っていて、理系はほとんど知らないことなど存在しないんだよ。
理系卒の人間が金融や保険やマスコミに就職することはあっても、
文系卒の人間がメーカーの研究開発職に就くことは無理。
文系には限界があるんだよ。

616 名前:132人目の素数さん:2009/06/02(火) 21:51:39
>>614
えっと、出題者ですがw
一応初等幾何での証明は用意はしてありますが、やたら面倒でカタルシスも得られず
決して「面白い問題」とは言い難い気がしてきたので、スルーしてもらっても...。
もちろん、もっとカッコイイ証明が見つかるかもしれないので、
挑戦したいかたはどうぞ。
用意した解答は明日にでも提出します。

617 名前:132人目の素数さん:2009/06/02(火) 22:47:43
パズル板でも出題たんだが、人が少ないのでこっちにも。
どっかのスレで(i)パターンは見たけど、この問題の真骨頂であると個人的に思う
(ii)パターンはあまり見ないので投下
問題の不備とかあったら教えてくれるとありがたいです

【海賊の多数決】
・メンバー全員に順序があらかじめ決められている
・自分の番が来た者は、宝の分配法を全員に提案する
・提案者を含めた全員でその案を採択するかどうかをyes/noで決をとる
・yesが半数以上なら、その案に従い宝を分配し、終了とする
・noが過半数なら、提案者を処刑し、次の順番の者が新しい案を提案する
・以上を分配法が決定するまで繰り返す
・メンバー全員は各人とも、次の優先順位に基づき提案・選択をする
(1)自分の命(自分が死なないような行動をとる)
(2)物欲(自分が死なないなら、自分の取り分を少しでも多くしようとする)
(3)他人を殺す快楽(自分が死なず自分の取り分が同じなら、他者が死ぬ方を選ぶ)
・メンバーは全員、賢い(論理的思考力と計算能力が十分ある)ことが仮定されている
・メンバー全員の前で同時に「メンバー全員が賢いこと」と
「メンバー全員が上の優先順位に基づき行動すること」が教えられる
・金貨は分割できない

問。次の時、どのようなことがおこるか?
(i)海賊のメンバーが10人で金貨が10000枚の時
(ii)金貨が10枚で海賊が10000人の時


618 名前:132人目の素数さん:2009/06/02(火) 23:00:53
普通に、自分以外の全員を殺して金貨独り占めじゃね
相手に殺られるより先に殺れば自分は死なない

619 名前:132人目の素数さん:2009/06/02(火) 23:15:48
>>607
> 上代・下代とか、前渡金、前受金とか。

読めません!(>_<;)

620 名前:132人目の素数さん:2009/06/02(火) 23:18:09
うえだい
しもしろ
ぜんとかね
さきじゅきん

621 名前:132人目の素数さん:2009/06/02(火) 23:33:11
>>618
計算高くねえし論理的思考力もねえw

622 名前:132人目の素数さん:2009/06/02(火) 23:38:05
いや、自分より多人数の敵を殺すのは腕っぷしだけじゃ叶わないぞ
計算高さや論理的思考力も必要なはずだ

623 名前:132人目の素数さん:2009/06/02(火) 23:50:40
あのなあ、仲間皆殺しにしてそっから先の海賊稼業どうするんだ。
喜びを分かち合う仲間も大切だろ。

とツッコミを入れつつ、そういう問題じゃねえとマジレスもしてみる。

624 名前:132人目の素数さん:2009/06/03(水) 00:47:45
海賊の話、だいぶ前にコマ大で出題されてたな。
同じかどうか詳細はわからんが。

625 名前:132人目の素数さん:2009/06/03(水) 01:26:20
金貨10枚で大人数を納得させるのは無理
→トップ5000人は無条件で死亡
→死にたくないからなんでもいい
→1「全部俺のもの」

626 名前:132人目の素数さん:2009/06/03(水) 02:12:06
考え方としては、最終局面から逆算していくんだよね。
(i)の10人で考えると、もし、最後の2人だけが生き残ったとしたら、
9人目の提案は必ず可決されるので、9人目の提案は「自分の総取り」
これを踏まえると、8人目は「自分が9999枚で10人目が1枚」と提案することになる。
すると、7人目は、「自分が9999枚で9人目が1枚」でも9人目は賛同してくれる。
6人目は「自分が9998枚で8人目10人目が1枚ずつ」
5人目は「自分が9998枚で7人目9人目が1枚ずつ」
…と考えると、
1人目は「自分が9996枚で3人目・5人目・7人目・9人目が1枚ずつ」で、奇数番目が全員賛成して可決。
(ii)は面倒くさそうだな...

627 名前:132人目の素数さん:2009/06/03(水) 02:32:19
n番目に順番が来るメンバーを[n]と表す。
また、生き残っている者で順番が若い者からA枚,B枚,・・・と分け前を提案する事を
{A,B,・・・}で表す

(@):
もし[10]に順番がまわったら、[10]は{10000}を提案する。
もし[9]に順番がまわったら、[9]は{10000,0}を提案する。([9]自身が賛成すれば必ず可決できる)
ここから遡って、[8]の提案に対する[9]と[10]の対応を考察してみると、
・[9]は必ずnoと言う。[8]を処刑して金貨10000枚得る方が得だから。
・[10]は自分に1枚以上分け前があれば賛成する。何故なら[9]に順番がまわると分け前が0になるから。
従って、[8]は{9999,0,1}を提案し、[8]と[10]のyes票が入って可決される。

これを踏まえると、[7]が提案する時点では
・[10]は2枚以上でyes
・[9]は1枚以上でyes
・[8]は10000以上でyes
となるので、yes票二つを確保しつつ[7]の分け前を最大にする{9999,0,1,0}が提案される。

以下同様に、[n]は[n+1]に順番がまわったときの分け前から各人にyesを選ばせる金貨の枚数が分かり、
結局[1]が{9996,0,1,0,1,0,1,0,1,0}を提案して可決される。

628 名前:132人目の素数さん:2009/06/03(水) 02:33:09
(A):
[9980]まで順番が来れば、(@)と同様に考え
{0,1,0,1,0,1,0,1,0,1,0,1,0,1,0,1,0,1,0,1,0}で可決される。
すると[9979]は、[9980]案で0枚なる人物誰か10人と自分自身で11票のyesを得られるが、
その選び方は一意的ではない。
すると[9978]が案を出す時点で、
・奇数番の者は「[9979]の番になれば必ず1枚もらえる」と考える
・偶数番の者は「[9979]の番になれば0枚か1枚もらえる」と考える
すると、奇数番の者からyesをもらうには2枚以上を割り当てねばならず論外。
偶数番の者に1枚ずつ割り振っても一人は分け前0なので、結局[9979]は処刑される。

従って、[9979]は[9978]の提案には必ず賛成する。
従って、[9978]は自分と[9979]と後誰か10人の12票で可決させられる。
従って、[9977]は自分の順番が来たら処刑される。
従って、[9977]は[9976]の提案には賛成するが、[9979]はギリギリまで処刑を楽しむため反対する。
結局これより前の番号について、奇数番の者は自分の番の来る直前までは反対し続けるので
[1]〜[9977]は処刑され、[9978]が例えば
{0,0,1,0,1,0,1,0,1,0,1,0,1,0,1,0,1,0,1,0,1,0,0}
(分け前1は[9980]以降の偶数番の誰か10人)
と案を出してこれが可決


629 名前:132人目の素数さん:2009/06/03(水) 02:42:24
>>628
ちょっと違うとこが。
[9980]の提案は
{0, 0,1,0,1,0,1,0,1,0,1,0,1,0,1,0,1,0,1,0,1}
でしょう。
あと、何を提案しても処刑されるのは、[9979]じゃなくて[9978]だよね。
その後もちょっとずつずれてる。

まあ、考え方はそんな感じだよね。
「絶対1枚も貰えない」よりは「後の人のさじ加減1つではあるが貰える可能性がある」
方を選ぶってのは、条件として明記しとくべきかな。

630 名前:132人目の素数さん:2009/06/03(水) 02:42:53
1788人目までは無条件で死亡。
1789人目の海賊が、
5885〜7932、8957〜9568、9725〜9852、9917〜9948、9965〜9972、9977〜9978、
9980、9982、9984、9986、9988、9990、9992、9994、9996、9998、10000人目の海賊
のうちの10人に金貨を渡せばいいんじゃないかな。
1789〜5884番目までの4096人と金貨をもらった10人の合計4106人の賛成、残り4106人の反対で
可決されると思う。


631 名前:132人目の素数さん:2009/06/03(水) 03:13:32
>>628だが、よくみたら全然違うな。
後ろからさかのぼって考えると、こんな感じ

[9981]:9981,9983,9985,9987,9989,9991,9993,9995,9997,9999に1枚ずつ 10/20で可決
[9980]:9982,9984,9986,9988,9990,9992,9994,9996,9998,10000に1枚ずつ 11/21で可決
[9979]:9980の提案で1枚も貰えない11人のうち10人に1枚ずつ 11/22で可決
[9978]:何を提案してもダメ 処刑
[9977]:9979の提案で確実に1枚も貰えない11人のうち10人に1枚ずつ 9978も賛成で12/24で可決
[9976]-[9974]:何を提案してもダメ 処刑
[9973]:9977の提案で確実に1枚も貰えない13人のうち10人に1枚ずつ 9976-9974も賛成で14/28で可決
[9972]-[9966]:何を提案してもダメ 処刑
[9965]:9973の提案で確実に1枚も貰えない15人のうち10人に1枚ずつ 9972-9966も賛成で18/36で可決

規則性があるから、あとは地道に考えれば...と思ったら、>>630が答えを書いてたw

632 名前:132人目の素数さん:2009/06/03(水) 03:16:28
(ii)については、たとえば最初のもの[1]が
「金貨10枚は捨てて誰のものにもならない」などと言えば
> (1)自分の命(自分が死なないような行動をとる) 
から、>>628の案では2〜9977の死んでしまうものたちが
賛成してはくれないだろうか?


633 名前:630:2009/06/03(水) 03:17:18
間違えた。
金貨を渡す相手は
5885〜7932、8957〜9568、9725〜9852、9917〜9948、9965〜9972、9977〜9978、
9980、9981、9983、9985、9987、9989、9991、9993、9995、9997、9999人目のうちの10人だ。

634 名前:132人目の素数さん:2009/06/03(水) 03:19:14
まあ、この番組自体が、きちんと理解した上での正答と
何だかわかんないけど偶然答だけはあっていたことを
きちんと区別しようとはしていないしな


635 名前:132人目の素数さん:2009/06/03(水) 03:21:28
誤爆すまん

636 名前:132人目の素数さん:2009/06/03(水) 03:24:20
>>634-635
コマ大生乙

637 名前:132人目の素数さん:2009/06/03(水) 04:19:07
まあ、普通に考えたら、最初の1788人が粛々と死を選ぶとは思えないので、
その「生存本能」「物欲」「残虐性」以外の第4の欲望に訴えるような提案を
必死でするわけだな。
つまり、>>1-1788>>1789-10000の性奴隷と化すという提案で、可決。

あるいは、現実的な解として、>>1-1788が協力して>>1789を袋だたきにして、
半死半生の微妙な状態で人質にとると、>>1の提案が過半数をとるかもしれない。

638 名前:132人目の素数さん:2009/06/03(水) 10:00:08
>>634
実際の試験やら受験やらでも、
きっちり考えて解こうがサイコロ振って適当に答えようが
正解なら同じ点数なんだからいいんじゃない?

639 名前:132人目の素数さん:2009/06/03(水) 15:38:20
試験や受験の空欄を埋めるためだけに
勉強をしているのならまったくかまわんよ


640 名前:132人目の素数さん:2009/06/03(水) 18:09:33
視聴者は解けるまでの仮定とか理論を楽しむんだろうけど、
バラエティ的には正解か否かで騒いでるんだから細かいこと気にしなくていいだろって意味。

641 名前:132人目の素数さん:2009/06/03(水) 18:33:49
>>614
一応、>>488の解答例を。
> ∠ABC=24°の菱形ABCDがあって、
> 線分BCのC側の延長上に点Eを、∠CDE=30°となるようにとるとき、
> ∠DAE=30°となることを証明してちょ。

線分CE上に、∠FAC=24°となるように点Fをとり、
Fを通る直線ACと平行な直線と直線BAとの交点をGとする。
さらに、線分AFを1辺とする正三角形AFHを、AFからみてCと反対側に作り、
直線GHと直線BDの交点をXとする。

菱形の対称性より、∠ABD=∠DBC=∠CDB=∠BDA=12°、∠CAB=∠DAC=78°
∠FAB=78°+24°=102°、∠GAF=78°
AC//GFより、∠AFG=∠FAC=24°なので、∠FGA=78°=∠GAFとなり、AF=FG
AF=FH=HAなので、FG=FHとなり、
∠GFH=60°-24°=36°より、∠FHG=∠HGF=72°

2点G、Fは直線BDに対して対称の位置にあり、Xは直線BD上にあるので、
XG=XFであり、∠XGF=∠HGF=72°なので、∠GFX=72°、∠FXH=∠FXG=36°
∠HFX=72°-36°=36°=FXHなので、HX=HF=HA
∠AHG=72°-60°=12°より、∠HAX=∠AXH=6°
∠BXG=36°/2=18°なので、∠BXA=12°
ここで、∠BDA=∠BXAより、点Xと点Dは同一である。

対称性より、∠CDF=∠ADG=∠ADH(=∠AXH)=6°
∠FDA=6°+24°=30°
ここで、∠DAF=78°-24°=54°、∠EDA=30°+24°=54°=∠DAFで、
AD//FEなので、四角形EDAFはED=FAの等脚台形となり、
その対称性より、∠DAE=∠FDA=30°

642 名前:617:2009/06/03(水) 20:31:16
こちらで用意した(ii)の答え↓
1〜1788番は何を提案しても処刑
1789番(残り8212人)の案は
1790〜5884番と1789番自身の4096人は死にたくないから無条件で賛成
5885〜10000番の中の10人に金貨を与えれば、その10人は賛成
以上4106人(ちょうど半数)の賛成により可決される

一般に、海賊の数が金貨の総数aの2倍以上の時、残りの海賊が2a+2^n 人(n=0,1,..)
で可決される(今回は残りが8212=2*10+2^13 人)

9979番(残り22人)の案は[9980番の案で1枚も貰えない11人のうち10人に1枚ずつ与える]
だから配り方は一意に定まらず、[確実に貰えない者]と[一枚貰える可能性がある者]に分かれ
全ての者に[金貨が貰えない可能性がある]ことになる
9978番(残り23人)は何を提案しても処刑
9977番(残り24人)は[9979〜10000番の中の10人に1枚ずつ与える]提案をすれば
その10人は、[金貨が貰えない可能性がある]よりも[確実に金貨が貰える]方を優先させる・・・(*)
ので賛成し、9978番と9979番自身も賛成するので可決される
このような推論をすると
1789番が金貨を与えるのは5885〜10000番の中の10人なら誰でもよいこととなる

問題の条件で(*)は成立してると思うのだが
(*)が不成立or成立してるかどうかがわからない場合は
次に可決する提案で[確実に貰えない者]に与える提案をすれば、必ず賛成してくれる
よって1789番は5885番の提案で[確実に貰えない者]↓
5885〜7932,8957〜9468,9725〜9852,9917〜9948,9965〜9972,9977,9978,
9980,9981,9983,9985,9987,9989,9991,9993,9995,9997,9999
の中の誰か10人に1枚ずつ与えればよい

643 名前:630:2009/06/04(木) 00:34:08
>>642
なるほどねぇ。
結果が不確実な場合にどうしたらいいのかの判断に困ったので、確実性重視の回答に
したんだけど。
[9978]は、金貨を与えれば確実に賛成を得られる者たちがいるにもかかわらず、
既に一定の確率で金貨が手に入り、自分を殺したがっている者たちのうちの誰かに
金貨を渡すようなまねをするだろうか、また、それより前の者も[9978]がそのように
考えることを予測するんじゃないだろうか、とか思っちゃったもんだから。

ところで、そうやって確率や期待値で判断することをありとするなら、
問題の3番目の条件はなくても多分一緒の答になるんじゃない?
つまり、
(1)自分が死ぬ確率が最も低くなるような行動をとる
(2)自分が死なないなら、自分の取り分の期待値が最も多くなるような行動をとる
(1)(2)の条件で最良と考えられる行動が複数ある場合は、そのうち一つを等確率で選ぶ。
としておいても、同じ結果になるように思う。

644 名前:132人目の素数さん:2009/06/04(木) 05:03:03
>>616
とっとと証明を書けよ!(笑)

645 名前:132人目の素数さん:2009/06/04(木) 05:05:26
>>644
>>641

646 名前:132人目の素数さん:2009/06/04(木) 10:15:02
東大デモクラシー

647 名前:132人目の素数さん:2009/06/04(木) 11:01:22
そのダジャレは最近の若者にはわかるまい
ん?

648 名前:132人目の素数さん:2009/06/04(木) 12:18:19
灯台モトクラシー

649 名前:132人目の素数さん:2009/06/04(木) 14:26:41
>>488について考えたが途中でわからんくなった
ACとDEを延長した交点をFとする
更に、AFをF側に延長し、その上に点Oを、∠ADO=78度になるようにとる。三角形ADOは、AO=DO底角78度頂角24度の二等辺三角形になる
この二等辺三角形の底辺であるADの垂直二等分線をひき、半径OAの円との交点をXとすると、DXやXAを一辺とする正三十角形が書ける
これによって問題中の∠ADCや∠CDEが全部円周角になって、解ける、かと思ったが

AEを延長した直線が、AからD側に数えて6つ角を挟んだ先の正三十角形の頂点と交わることを示せるはずなんだが示せない、方針まずいのかなあ
OF=FAを使おうかと思ったがこれも使えないし・・・

650 名前:132人目の素数さん:2009/06/04(木) 15:27:53
ゴリ押しのダサイ答案なんて要りません!
エレガントな解法があれば、私のところに来なさい!

651 名前:132人目の素数さん:2009/06/04(木) 21:12:37
正五角形と正三角形の組合せでできないかと試行錯誤したが、
それっぽい図を書いても結局肝心な部分の角度が証明できないので、
結局>>641よりスマートな証明は見つからなかった。

>>649
整角四角形が正多角形の対角線を全部書いた図に埋め込めるのは当然なのであって、
それを利用すれば証明を作れるわけじゃない。

652 名前:132人目の素数さん:2009/06/04(木) 23:23:50
わざわざスレ立ててしょーもない問題出してる奴が居た。
暇つぶしに解いてみる?

い 48→32→6
ろ 46→24→8
は 35→15→5
に 68→14→5
1つを除いて、ある法則の上に並んでいます。
仲間はずれはどれでしょう?
理由も添えて答えなさい

ちなみにそのスレでもう答は出てる。

653 名前:132人目の素数さん:2009/06/04(木) 23:31:23
こういう問題って、凝った「法則」を考えれば
どれを仲間はずれにすることも出来るから、
やる気にならないw

654 名前:132人目の素数さん:2009/06/04(木) 23:33:21
その糞スレでもう答えは出てただろ

655 名前:132人目の素数さん:2009/06/05(金) 00:13:47
>>652
>>653
まあ答を言うと、
法則は「十の位と1の位をかけ合わせた数が次の数値」で、
仲間はずれは「に」だ。これは簡単。

では>>653 が言うように、
「に」以外を別の法則でもって仲間外れにするというのを考えてみよう。

656 名前:132人目の素数さん:2009/06/05(金) 00:18:05
面白くないから却下

657 名前:132人目の素数さん:2009/06/05(金) 00:28:28
関数F(x)は
F(46)=24,
F(24)=8,
F(35)=15,
F(15)=5,
F(68)=14,
F(14)=5,
F(48)=0,
を満たす関数とする。
例えば、適当なa〜hに対して
F(x) = ax^7 + bx^6 + cx^5 + dx^4 + ex^3 + fx^2 + gx + h
とすれば良い。

このとき "ろ,は,に" はx→F(x)という法則にしたがっているが "い" は従っていない。

うん、やっぱつまらん。

658 名前:617:2009/06/05(金) 00:51:43
>>643
その2条件だと[9979]〜[10000]の提案は同じになるけど
[9978]は[9979]と同じ案
[9980,9981,9983,9985,9987,9989,9991,9993,9995,9997,9999の11人の中の10人に一枚ずつ与える]
を提案すれば、金貨を貰える10人と[9978]自身は賛成、11人中で金貨を貰えない1人は反対
残りの11人は[9978]と[9979]のどちらが可決になってもよい(必ず一方は可決される)
と考えて、1/2で賛成/反対する
この提案が否決になるのはこの11人が全員反対するときのみなので
その確率は(1/2)^11 = 1/2048 であり、可決される確率は 2047/2048 になる

[9977]は[[9979]〜[10000]の誰か10人に一枚ずつ与える]と提案をして、必ず可決
[9976]〜[9974]は[9977]と同じ提案をすることで
提案した時 1/4 の確率で可決
と考えていくと
[1789]は[[5885]〜[10000]の誰か10人に一枚ずつ与える]と提案をして、必ず可決
[1]〜[1788]は[1789]と同じ提案をすることで
提案した時 1/8192 の確率で可決
となるので617とは異なるけど、問題として考えると面白いかな

659 名前:132人目の素数さん:2009/06/05(金) 02:51:39
>>648
Today MTX

660 名前:132人目の素数さん:2009/06/05(金) 19:46:28
正四面体、正六面体、正八面体、正十二面体、正二十面体
以上5種類のダイスがそれぞれの面の数だけある。

正四面体:4個
正六面体:6個
正八面体:8個
正十二面体:12個
正二十面体:20個

それぞれのダイスには1〜面数の整数が一つずつ書かれている。
重心の片寄りなどはなく、同じ面体ならどの目が出る確率も均一である。

これらのダイスを、正二十面体を一つ残して他を全部振ったら、
全てのダイスが1の目を出した。

この状態で残りの一つである正二十面体のダイスを振った時、1以外が出る確率はいくらか。

661 名前:132人目の素数さん:2009/06/05(金) 19:56:43
19/20
という答えはもちろん想定して無いんだろうな

662 名前:132人目の素数さん:2009/06/05(金) 20:39:05
たぶんそれいかさま

663 名前:132人目の素数さん:2009/06/05(金) 21:20:26
正方形ABCDの辺AB上にAO:OB=3:1となるように点Oをとり、DOをひく。
次に、辺CD上にCQ:QD=3:1となるように点Qをとり、DOに平行な線を正方形ABCD内にひく。
次に、辺AD上にAR:RD=1:3となるように点Rをとり、RCをひく。
次に、BC上にBP:PC=3:1となるように点Pをとり、RCに平行な線を正方形ABCD内にひく。
また、DOとAP、RCの交点をそれぞれE、Fとし、BQとRC、APの交点をそれぞれG、Hとする。
このとき、四角形EFGHはどのような四角形になるか。また、そうなることを証明しなさい。

664 名前:660:2009/06/05(金) 22:43:57
ひっかからないか・・・・つまらん。

665 名前:132人目の素数さん:2009/06/05(金) 22:47:18
>>663
感覚的に正方形なんだが証明が面倒。

666 名前:132人目の素数さん:2009/06/05(金) 23:10:39
正四面体だけ地面に底がついたとき丁度真上を向く面が無いよね
だから上からは見える面には書いてない数を出目とするか
もしくは頂点に数を書いた奴を作らないといかん
だったら正八面体に1〜4を2つずつ書いた奴を作ったほうが実用的だわな

667 名前:132人目の素数さん:2009/06/05(金) 23:15:38
四面体の使いにくさは異常
TRPGでも使用ダイスの中に含まれているものがあるが正気か?

668 名前:132人目の素数さん:2009/06/06(土) 00:45:39
>>660
その表現だと、ダイス1個につき1個ずつしか数が書いてないって意味にならんか?

…と揚げ足取っても仕方ないようなつまらん問題だな。

669 名前:132人目の素数さん:2009/06/06(土) 01:17:54
どの多面体のサイコロでも使えるように、
サイコロを振って床に付いた面に書かれた数字をそのサイコロの出目とする、
これで、出目の曖昧さは解消だ。


670 名前:132人目の素数さん:2009/06/06(土) 02:00:12
>>669
それだと、普通のサイコロとの整合性が失われてしまうので、

 2*[書かれた数字の総和] / [面の数] - [床に付いた面に書かれた数字] 
  をサイコロの出目とする。

でどうだろう。
つまり、四面体ダイスで床に付いた面に書かれた数字が1なら、4の目が出たとみなすわけだ。

671 名前:132人目の素数さん:2009/06/06(土) 02:19:00
(昔の)サッカーボール型のサイコロに
正五角形に1から12までの数字を、正六角形に13から32までの数字を書いたとき、
1と32の出やすさの違いは、実際のところどうなんだろう。
面積の広い正六角形の方が安定してて出やすいだろうとは思うが

672 名前:132人目の素数さん:2009/06/06(土) 02:20:35
動きが止まった瞬間に内部回路で乱数を生成しその数から目を求めて
どの面にも取り付けてある液晶画面に結果を出力すれば解決

673 名前:132人目の素数さん:2009/06/06(土) 08:40:45
>>670
どの多面体でも、ということで、
普通の6面体サイコロでも床に付いた面を出目にする、という使い方に変える。
視認性は考慮せず、これまでの使い方も忘れる。

674 名前:132人目の素数さん:2009/06/06(土) 09:29:26
> 視認性

ガラステーブルの下に寝転んで
テーブルの上でダイスをふればよろしい。

675 名前:132人目の素数さん:2009/06/06(土) 09:39:19
頭いい!


676 名前:132人目の素数さん:2009/06/06(土) 11:11:59
そんな問題どうでもいから663やろうぜ・・・

677 名前:132人目の素数さん:2009/06/06(土) 11:25:52
>>676
それが本当に面白い問題だとでも思っているのか?

678 名前:132人目の素数さん:2009/06/06(土) 11:34:04
3:4:5

679 名前:132人目の素数さん:2009/06/06(土) 11:55:49
>>672
>>670でもどの多面体でも使えるじゃん。
相対する面の和が常に一定になるサイコロなら通常の使い方と結果として同じになるし。

680 名前:132人目の素数さん:2009/06/06(土) 21:58:21
>>665
>>663だが、この問題誰に出してもそう言われる
自分でも面倒すぎて解答出してないし

681 名前:132人目の素数さん:2009/06/06(土) 22:30:01
どこが面倒なのかもどこが面白いのかも分からん。
>>663=>>665=>>680が宿題でも持ち込んだとしか思えん。


682 名前:132人目の素数さん:2009/06/06(土) 22:48:15
面白くはないが宿題とも思わんなあ
仮に宿題ならこんな問題出す出題者のセンスを疑う

683 名前:132人目の素数さん:2009/06/06(土) 23:00:57
>>681
証明が湾曲して感じる
散らばった三角形がどうも面倒に感じるんだがなあ
俺と周囲が面倒に思ってるだけで、他の人は面倒じゃないのか

684 名前:132人目の素数さん:2009/06/06(土) 23:03:15
あからさまに合同な三角形を見つけるのは面倒のうちに入らん
なにせ見りゃ分かるんだから

685 名前:132人目の素数さん:2009/06/06(土) 23:13:31
>>663
問題文中
> DOに平行な線を正方形ABCD内にひく
> RCに平行な線を正方形ABCD内にひく
が全く無駄なんだが。

686 名前:132人目の素数さん:2009/06/06(土) 23:37:51
1) 自己交差しない任意の閉曲線Cから、相異なる3点を正三角形をなすように選ぶことは出来るか?

2) 自己交差しない任意の閉曲線Cから、相異なる4点を正方形をなすように選ぶことは出来るか?

687 名前:132人目の素数さん:2009/06/06(土) 23:53:05
1は回転させると簡単で、しかもこれよりかなり強力なものが証明できるけど
2って同じ論法でいけたっけなあ


688 名前:132人目の素数さん:2009/06/07(日) 00:20:07
正方形ならいけるとオモ。
しかし5角形はムリポ

689 名前:132人目の素数さん:2009/06/07(日) 01:27:59
正方形いける? 無理っぽい気がするけどなー。

690 名前:132人目の素数さん:2009/06/07(日) 19:09:20
有名問題だけど、条件をはっきりさせてみた
問題に不備とかあったら指摘していただけるとありがたい

・N人の人たちがある会場に集められた
 彼らは全員、論理的思考力と計算能力が十分にあるが
 ポーカーフェイスで、おまけに交通量調査のアルバイトで
 数を数える能力が十分に培われていた
・N人の中のm人には額に赤い印を
 残りのN−m人には額に白い印を描いた
 自分の印の色を確認することができないが
 自分以外の者の印については赤/白のどちらかなのかが瞬時に判断することができる
・N人は互いにコミュニケーションをとったり、勝手に情報の伝達をすることはないとする
・司会者(N人以外の別の人)がやってきて、次のことを全員の前で同時に告げる
(1)あなたたちは全員、論理的思考力と計算能力が十分にあり
 ポーカーフェイスで、おまけに交通量調査のアルバイトをしていて
 数を数える能力が十分に培われている
(2)あなたたちは全員、額に赤または白の印がついている
(3)額に赤い印がある者は少なくとも一人はいる
(4)これから間隔をあけて、幾度もベルをならす。自分の額の色がわかった者は
 わかった後になるベルの直後に、他の者にわかるよう大きく手をあげること
 それでははじめる(1回目のベルをならす)


どのようなことがおきるか?

691 名前:132人目の素数さん:2009/06/07(日) 21:23:55
ベルが鳴る間隔がじゅうぶんに長くないと
混乱が起こる。


692 名前:132人目の素数さん:2009/06/07(日) 21:44:22
m回目のベルの直後に赤の人全員が手を上げ
m+1回目のベルの直後に白の人全員が手を上げる

693 名前:132人目の素数さん:2009/06/07(日) 22:32:54
>>690
mの値が参加者に伝えられているかどうかは明記するべき

694 名前:132人目の素数さん:2009/06/07(日) 22:49:17
>>693
伝えられてる必要ないだろ

695 名前:132人目の素数さん:2009/06/07(日) 22:52:32
必要かどうかではなく
結果が変わるだろ。

696 名前:132人目の素数さん:2009/06/07(日) 23:04:11
問題文に書かれている情報以外を参加者が与えられていると考える方がどうかしている。

697 名前:132人目の素数さん:2009/06/07(日) 23:27:36
つまり早く分かりたいと思う人(←利益がないのに)は鏡のあるところに行くということで

698 名前:132人目の素数さん:2009/06/07(日) 23:28:05
教えられてたら一瞬でこのゲーム終わるな

699 名前:132人目の素数さん:2009/06/08(月) 04:58:50
参加者にひとり、指で触ると色を感じることができる人が混じっていて
最初のベルの直後に手を上げた。

その後はどうなるだろうか。

700 名前:132人目の素数さん:2009/06/08(月) 07:30:25
たけのこニョッキゲームになる

701 名前:132人目の素数さん:2009/06/08(月) 11:58:48
>>699
その人がそういう能力を持っていることを他の参加者が知っているかどうかによる。

702 名前:690:2009/06/08(月) 23:56:57
さすがに簡単、というより有名すぎましたか。
条件をいくつか加えれば、愚か者がまじっていても成り立つので
[論理思考力があるかどうかのテスト]に使えるのではないかと考えています
(恐ろしく実用性がないのが難点w)

もっと複雑な問題(こちらも有名か)
・全員の額には印ではなく、0以上の実数が書かれている(但し全員0ではない)
・会場の壁にはN枚以下の紙が貼られていて、1枚につき1つの実数が書かれている
・N人の額の数字の合計は、どれかの紙に書かれた数になる
・これらの条件も、全員の前で同時に教えられる
[この時、どうなるか?]
というのもあるけど、私にはせいぜいN=2の時しかわからないので
出題できないよorz

703 名前:132人目の素数さん:2009/06/09(火) 01:47:18
>>702
どうなるかと言われても、前の問題のように
なにかがわかったらどうしろとかいう規則がないと
何も起きない

704 名前:702:2009/06/09(火) 07:59:24
スマン。加えた条件と690の問題文を
額の印の色→数
と対応させて読んでくれ
つまり、自分の額の数がわかった者が手を挙げなければならない
などとなる

705 名前:132人目の素数さん:2009/06/09(火) 22:48:54
自分に書かれている数字の可能性が実質無限だから、
何回ベルを鳴らそうと誰も手を挙げられない。

というか論理思考力があったら出題の時点で全員投げる。

706 名前:132人目の素数さん:2009/06/09(火) 23:42:03
>>686はどうやんの?

707 名前:132人目の素数さん:2009/06/09(火) 23:46:04
>>705
>自分に書かれている数字の可能性が実質無限だから、
N 通りしかないでしょ。

708 名前:132人目の素数さん:2009/06/10(水) 01:46:26
>>686のi)
閉曲線C上の任意の点をAとする。
閉曲線C上のAから一番遠い点(閉曲線上の距離ではなく、空間中の直線距離)をBとする。
(もし一番遠い点が複数あるなら、そのうちのどこでもいいので1点を選ぶ)
Pは閉曲線C上を移動する点である。

線分APの中点MでAPに垂直に交わる平面S上にMを中心とした半径AB*((√3)/2)の円Rを描く。
閉曲線Cと平面Sとの交点Qについて
・C上の点AとPは、平面Sのそれぞれ異なる面側にあるので、CとSの交点Qは存在する。
・CとSの交点はCが閉曲線であるから、必ずふたつ以上存在する。それぞれの点をQ[n](n=1,2,...)とする。

・PがAの十分近傍にあるときには、交点Q[1..n]のひとつはMにあり、他の交点はRの外にある。
・PがBにあるときには、交点Q[1..n]は全て周上を含むRの内側にある。
 (もしRの外に交点があるなら、BがAから一番遠い点だということに矛盾する)
・PがAからBまで移動する間、CとSの交点Q[1..n]のうち少なくともひとつは一度以上、円Rの周上を通る。
 (ここは厳密性に欠けるけど、まあわかってもらえると思う)
・円Rの周上に交点Q[m]があるとき、APQは正三角形である。

709 名前:132人目の素数さん:2009/06/10(水) 01:53:47
なおしてる最中に送っちまった…

訂正
・円Rの周上に交点Q[m]があるとき、A・P・Q[m]は正三角形である。

以上のことから、 閉曲線C上の任意の点Aに対して
 閉曲線C上にうまく他の2つの点P、Qを選ぶことにより
正三角形APQを構成できるといえる。



710 名前:132人目の素数さん:2009/06/10(水) 03:55:53
>>708-709
↓反例。

R^2上に3点S(1,0),T(-1,0),U(0,100000000)を取り、△STUの周を
閉曲線Cとして設定する。
C上の点AとしてA=Uを採用するとき、他の2点P,Qをどのように
取っても、∠PAQは常に60°より小さくなり、よって、△APQは
絶対に正三角形にならない。

711 名前:132人目の素数さん:2009/06/10(水) 04:07:08
閉曲線上の点を取る
点を中心に閉曲線を60度回転させる
回転前の閉曲線と回転後の閉曲線は必ず回転の中心以外の点で重なる
回転後の閉曲線の回転中心近傍の点は、どちらかが元の閉曲線の内部へ行き、もう片方が閉曲線の内部へ出る
これも閉曲線であるから、回転中心以外の点で内部と外部の境界を横切る
この横切る点と回転中心、回転によって横切る点へ移動する点を結ぶと、正三角形になる

任意の点で微分可能な閉曲線ならこれでいける?

712 名前:132人目の素数さん:2009/06/10(水) 08:18:25
>>710
なるほど
> ・PがAの十分近傍にあるときには、交点Q[1..n]のひとつはMにあり、他の交点はRの外にある。 
ここだな。

Pは任意の点ではなく十分近傍にあるときに、他の交点はRの外にあるような点でないとならないか。

713 名前:132人目の素数さん:2009/06/10(水) 10:38:08
>>707
そういえば合計値はわかってるんだったorz
もっかい考え直す。

714 名前:132人目の素数さん:2009/06/10(水) 11:52:38
>>702
> ・会場の壁にはN枚以下の紙が貼られていて、1枚につき1つの実数が書かれている
> ・N人の額の数字の合計は、どれかの紙に書かれた数になる

これって、全員の合計の数字はどれかの紙に書いてあるけど、それ以外の紙には
適当な数値が書いてある、ってこと?
とすれば、その適当な数値に寄って結果は異なるよね。

715 名前:132人目の素数さん:2009/06/10(水) 18:12:35
>>714
> その適当な数値に寄って結果は異なるよね。 
そりゃそうだ。

716 名前:132人目の素数さん:2009/06/10(水) 22:30:32
>>711
当然、
>点を中心に閉曲線を60度回転させる
>回転前の閉曲線と回転後の閉曲線は必ず回転の中心以外の点で重なる
は、>>710が反例になるわけだが、全ての点が反例になるような閉曲線ってなさそげな感じなので……これを変えれば証明できそうだな


717 名前:132人目の素数さん:2009/06/10(水) 23:27:06
そのような点は60度より小さな角でしかありえないんじゃないか?


718 名前:132人目の素数さん:2009/06/10(水) 23:45:43
>>717
フラクタル曲線に角は定義できるのか?

719 名前:132人目の素数さん:2009/06/11(木) 00:27:55
みんな分かってるんだとは思うけど、>>708は3次元空間の中の閉曲線、
>>711は2次元の平面上の閉曲線についての解答。
どちらもそれはそれで一つの問題だけど、あくまで別物なので、前提は
明らかにしとかないと。
まあ、もし3次元で証明できれば2次元では当然成り立つわけだけど。

>>686の2)の正方形の問題は、3次元では簡単に反例が作れると思う。
2次元だとどうなるかな。

720 名前:132人目の素数さん:2009/06/11(木) 00:31:51
>>718
その点で連続ならば角でないと言えそうなので
非連続なら角と定義したらどうだろうか?

721 名前:132人目の素数さん:2009/06/11(木) 00:39:35
>>719
確かに別物だけど、どっちもまだ解決してないんだから、どちらでもいいから解決するほうが先決。

722 名前:132人目の素数さん:2009/06/11(木) 00:42:16
>>721
解決してない?

723 名前:132人目の素数さん:2009/06/11(木) 00:54:43
すまん、細かくは見てなかったんだが……
>>708-709も、>>711も、>>710で反論されているんじゃないのか?

724 名前:132人目の素数さん:2009/06/11(木) 11:36:15
>>723
「閉曲線上の任意の点」ではなく、
「閉曲線上の曲率∞ではない(曲率半径0ではない)点」で考えれば
成立するのかな。

その場合、あらゆる場所でなめらかでないフラクタル曲線のような場合が問題になるのだが、
そういうものを含めなければ。

725 名前:132人目の素数さん:2009/06/11(木) 14:14:15
>>686 は閉曲線状の任意の1点を頂点とする正三角形ないし正方形を作る問題
ではないと思うんだが。>>710 の例では、(0,0)を一つの頂点とする正三角形や
ST上に一辺を持つ正方形をなすような点がC上に取れるが、これは >>686 の要件
を満たしているんではないか?

726 名前:132人目の素数さん:2009/06/11(木) 14:28:21
だから、>>708-709>>711では任意の1点を頂点とする正三角形ができるという
証明になってしまっているからそれの反例が>>710って話だろ。
別に>>710>>686の反例だという話はだれもしていない(ハズ)
>>708-709>>711以外にはまだ証明らしきものが提示されていないから
それをだれか示してくれ、ってのが現状。

727 名前:132人目の素数さん:2009/06/11(木) 14:33:09
>>711は最後の行の仮定からして>>710の三角形を取り扱ってないんじゃないか?

728 名前:132人目の素数さん:2009/06/11(木) 17:40:00
>>726
>>710>>708-709の、「任意の一点を頂点とする」に対する反例であって
>>712の修正を採用すれば「ある条件下の点をひとつの頂点をとれば」ということで
>>686の証明に(多少乱暴ではあるが)なっていると思うんだが、どうか?

729 名前:132人目の素数さん:2009/06/11(木) 17:46:58
>>712の修正を入れた証明に欠けている所はふたつあると思う

ひとつは最初に本人が厳密性が低いと指摘しているここの証明。
> ・PがAからBまで移動する間、CとSの交点Q[1..n]のうち少なくともひとつは一度以上、円Rの周上を通る。 

もうひとつはここ
> Pは任意の点ではなく十分近傍にあるときに、他の交点はRの外にあるような点でないとならないか。 

このような点が必ず存在することの証明がない
そのような点は微分可能な点であったり、60度以上の角の頂点で十分なので
こちらもほぼ自明なことだといえると思うが。

こんな証明したい奴いるの? どちらもまともにやるとけっこう面倒なとこだと思うよ。


730 名前:132人目の素数さん:2009/06/11(木) 18:11:50
一言だけ言っておく。

心からGJ

っと。

731 名前:132人目の素数さん:2009/06/12(金) 00:30:18
等面四面体(四つの面が全て合同な三角形の四面体)の展開図は、元の四面体の一つの面と相似なものが含まれる。
このような多面体は、等面四面体以外に存在するといえるか?

732 名前:132人目の素数さん:2009/06/12(金) 03:07:09
>>731
凸多面体でなくていいなら、存在するわな。

展開図:正三角形ABCの内側に、重心を共有して向きが180°違う
小さい正三角形PQR(1辺は外側の正三角形の1/6ぐらい)を書き、
AQ,AR,BR,BP,CP,CQを結ぶ。さらに、BC,CA,ABの中点をL,M,Nとし、
LP,MQ,NRを結ぶ。

これを、AMとAN、BNとBL、CLとCMが合わさるように折ると、
正三角錐の3つの斜面に三角錐をくっつけたような図形ができる。
(LP,MQ,NRのとこだけ谷折り)

この方法なら、正方形でもできそうだな。

733 名前:132人目の素数さん:2009/06/12(金) 18:46:05
有名(というか基本)問題と、その改変
改変の方は自称オリジナル

・A,Bは賢い(以下,ここでは論理的思考力と計算能力が十分あることと同値とする)
・A,Bの2人がいる前で同時に「A,Bは2人とも賢い」ことが教えられる
・A,Bに次の規則を教える:
 A,Bは00:00から00:15の15分間の中のどこかの連続した1分間(ちょうど)
 必ず,面会室に入っていなければならない
・A,Bは面会室の中で相手に会いたいと思っており,相手もそう思ってると考えている
・A,Bが面会室の中で会うとは
 A,Bが同時に面会室の中に入っている状態が一瞬(0秒以上)でもあることとする
・A,B間で情報の伝達や,あらかじめ取り決めがなされることはないとする


(1)A,Bがどの1分間に面会室に入っているかが任意(ランダム)の場合
 A,Bが面会室の中で会う確率は?
(2)A,Bがどの1分間に面会室に入っているかが自分の意志で決めることができ
 相手も相手の意志で入室している時刻を決めることができることを知っている場合
 A,Bが面会室の中で会う確率は?

問題の条件は過不足のないようつけたつもり
不備とかあったら教えてくだせい


734 名前::2009/06/12(金) 19:48:45
直角双曲線 y=1/x 上に相異なる3点を任意に取ったとき、
それらを結んで出来る三角形の垂心も常にもとの双曲線上にある
ことを証明せよ。

735 名前:工学部:2009/06/12(金) 20:00:39
>>733
(1)41/196
(2)1

かなぁ。
でも(2)を解いてるとき、「いつ執行されるかわからない死刑」
の問題といてる気分になったから、違ってるかも。

736 名前:132人目の素数さん:2009/06/12(金) 22:13:42
27/196 と 1 になった。

737 名前:132人目の素数さん:2009/06/12(金) 22:22:23
27/196 と 1かな。

738 名前:132人目の素数さん:2009/06/13(土) 19:20:18
容量が
20cm×20cm×20cmの箱がある。
ここに、直径1cmの球を出来るだけ多く入れる。

その際、球を球の中心を通る面で二等分しても良い。

2等分された半球3つで通常の球1つ分とカウントするとして、
箱に入る球の最大数はいくつか。

739 名前:733:2009/06/13(土) 22:53:47
>>736,737が正解
くやしいので追加w
(3)
問題は(2)と同じ。但し
>>733の問題で2つ目の条件
・A,Bの2人がいる前で同時に「A,Bは2人とも賢い」ことが教えられる

・A,Bのを別の場所に隔離して、単に「A,Bは2人とも賢い」とだけ教える
に変更し、以下の条件を追加
・A,Bは論理的に「時刻aから時刻bのどこかの1分間に入るのが最適」と推論した場合
 どの1分間に入るかはその範囲の中でランダムに決めるとする

740 名前:132人目の素数さん:2009/06/14(日) 14:22:18
>>738
とりあえず、

切らない球が
 20*20*13 + 19*19*14 = 10254 (個)
半球が
 20*20*2 + 19*14*4 + 10*10 = 1964 = 3*654+2 (個) 
全部で
 10254+654 = 10908 (個)

は入るかな。もっと入る?

741 名前:132人目の素数さん:2009/06/14(日) 19:26:14
半球ってそんなに入るか?
どういう構造になった?

742 名前:740:2009/06/15(月) 08:54:21
ごめん、ちょっと計算間違って1mmくらい箱からはみ出てた。
半球は
 20*20*2 + 19*14*4 = 1864 = 3*621+1 (個) 
全部で
 10254+621 = 10875 (個)
でいいかな。

743 名前:738 :2009/06/15(月) 10:41:44
>>742
正解。

・・・・・まあ俺の計算が間違ってなければの話だけども。


744 名前:132人目の素数さん:2009/06/15(月) 12:52:42
>>739
19%かな。

745 名前:132人目の素数さん:2009/06/15(月) 16:04:24
A〜Zまでの文字を並べて得られる列を文字列と呼ぶ。
文字列x,yに対し、関数d(x,y)を次のように定義する。

【定義】
 文字列xに対して、挿入・削除・置換(詳細後述)を繰り返して、文字列yに変換する場合の、挿入・削除・置換の最小実行回数をd(x,y)とする。

  [挿入]
 文字列xに場所を指定しつつ、一文字だけ文字を挿入する。たとえば、文字列abdの2文字目より後ろにcを挿入するとabcdとなる。

  [削除]
 文字列xの場所を指定し、一文字だけ文字を削除する。たとえば文字列abecの3文字目を削除すると、abcとなる。

  [置換]
 文字列xの場所と、置換後の文字を指定し、一文字だけ置換する。たとえば文字列abedの3文字目をcに置換すると、abcdとなる。


(関数値の例)
d("abdef", "abcdef") = 1

このとき任意の文字列x,y,zに対し、d(x,y) + d(y,z) ≧ d(x,z)を示せ。

746 名前:132人目の素数さん:2009/06/15(月) 16:35:59
>>745
ほとんど自明だが...

ある操作手順mに対し、その中に含まれる各操作の数をN(m)と表すものとし、
操作手順m_1を施した後、操作手順m_2を施すような操作をm_1+m_2と表すものとする。
さらに、文字列xに操作手順mを施した結果の文字列をF(x,m)で表すものとする。
定義より明らかに
F(x,m_1+m_2)=F(F(x,m_1),m_2)
N(m_1+m_2)=N(m_1)+N(m_2)

dの定義より、
N(m_1)=d(x,y),F(x,m_1)=yとなるような操作手順m_1が存在する。
同様に
N(m_2)=d(y,z),F(y,m_2)=zとなるような操作手順m_2が存在する。
すると、
F(x,m_1+m_2)=F(F(x,m_1),m_2)=F(y,m_2)=zであり、
dの定義より、
d(x,z)≦N(m_1+m_2)
また、
N(m_1+m_2)=N(m_1)+N(m_2)=d(x,y)+d(y,z)なので、結局
d(x,y)+d(y,z)≧d(x,z)となる。

なお、F(x.m)は、操作手順mがxに対する適切な操作手順であるようなx,mの組合せに
対してのみ定義されるものとする。

747 名前:132人目の素数さん:2009/06/15(月) 16:40:18
編集距離と言えば話が早いのに

748 名前:132人目の素数さん:2009/06/15(月) 16:40:55
要するに、「文字列」というものに、
三角不等式が成立するような「距離」という概念が導入できると言うことか。

749 名前:132人目の素数さん:2009/06/15(月) 17:10:29
最近第二版が出た「コンパイラ」とか
そのあたりの本にそういう演習問題がある

750 名前:132人目の素数さん:2009/06/15(月) 18:23:16
>>747
数学板なので、編集距離知らんやつ多いと思った。

751 名前:132人目の素数さん:2009/06/15(月) 18:31:38
ご配慮かたじけない

752 名前:132人目の素数さん:2009/06/15(月) 18:36:33
それが「距離」という概念の1つであることを示せという問題であるように
見えるのだが、最初から「距離」と言えば話が早いとはこれいかに?

753 名前:740:2009/06/15(月) 18:49:02
>>743
考え直してみたら、もうちょっと入った。

切らない球が
 20*19*27 = 10260 (個)
半球が
 20*19 + 20*2*27 + 20*20 = 1860 = 3*620 (個) 
全部で
 10260+620 = 10880 (個)

最後に半球を並べるところで、普通に入れると20*19になるところを
ちょっとずらして20*20入れるところがミソ。
積んだ高さはトータルで
 27*(√2)/2+(√3)/2=19.9579…
なので、今度はちゃんと入るはず。

754 名前:132人目の素数さん:2009/06/15(月) 18:53:32
>>752
「編集距離がたしかに距離であることを示せ」
割と知られた本の割と知られた問題を書くぐらいだったら、ねw

755 名前:132人目の素数さん:2009/06/15(月) 23:25:24
>>754
参考文献きぼんぬ

756 名前:132人目の素数さん:2009/06/16(火) 13:38:16
俺も知りたい

757 名前:132人目の素数さん:2009/06/16(火) 13:39:56
じゃあ俺も

758 名前:132人目の素数さん:2009/06/16(火) 13:48:47
俺も俺も

759 名前:132人目の素数さん:2009/06/16(火) 22:35:54
いやいや私が

760 名前:132人目の素数さん:2009/06/17(水) 06:13:15
上島:じゃぁ俺も

761 名前:132人目の素数さん:2009/06/17(水) 09:53:18
どうぞどうぞ

762 名前:132人目の素数さん:2009/06/20(土) 00:32:28
みごとなスレッドストッパーぶりだな

763 名前:739:2009/06/20(土) 21:41:19
>>744
正解
遅くなってスマソ

764 名前:132人目の素数さん:2009/06/21(日) 02:19:33
>>763
おっせぇーんだYO!

765 名前:132人目の素数さん:2009/06/21(日) 17:02:44
イタリア行きで注目されて、代表の試合で君が代を歌ってなかったことが右翼にバレた中田が危害を加えるとの脅迫を受け、
身辺警護のために日本に帰国した際にボディガードをつける

空港で当たり屋のように中田にぶつかって来たカメラマン(連中の常とう手段、野茂や伊良部にもやっていた)を蹴り倒したボディガードの姿にガクト、痺れる

真似してボディガードをつける
その際、海外セレブの真似して黒人のゴツイボディガードをつける

766 名前:猫のトラウマ ◆ghclfYsc82 :2009/06/21(日) 17:09:40
怖い話だ。この国には「思想信条の自由」も無いのか!


767 名前:132人目の素数さん:2009/06/22(月) 00:47:35
Gackt(やっぱ小文字で変換されるな)って国家独唱の時に君が代の歌詞を間違って歌ってたような

768 名前:132人目の素数さん:2009/06/25(木) 22:18:58
もしかしたら有名問題かもしれんが出してみる。

一本の紐があり、これに次の操作を繰り返し加えて分割する。
操作:「今ある紐の断片からランダムに一つ選び、3等分する。」
この操作をn回繰り替えしたのち、ランダムに2つ紐の断片をえらんだとき、
長さが等しい確率をp(n)とする。

lim[n→∞] p(n) を求めよ。


769 名前:132人目の素数さん:2009/06/27(土) 09:03:11
ロピタルを使わずに、高校数学レベルで、エレガントに次式の値を求めてね

lim[x→0] log|x - sin(x)| / log|1-cos(x)|

  /.:::::::,'    /.::::::::::::::::::::::::::/  |:::::!、::::::::::::::\::::::::::: :: :: :: : :ヽ
 /.:::::::::::ハ.  ./.::::::::::::/|:::::::::/   !::::! \::::::::::ヽ\:::::::::: :: :: : : :ヽ
. {.::::::::::/  `'''/.::::::::::::/ i:::_,,/....,,_  !:::|    \:::_:,,ヽ,,.ヽ,:::::::::: :: :: : :',
. l.:::::::N   /.::::::::::::/‐T:::/   `' ヽ{     ''\:::::ヽ ヽ::::::::::::: :: : :l
 ',:::::|.    /:::::::::::::/  l::/             \::ヽ ヽ:::::::::::::::::::::!
  ヽ.{   ,イ::::::::::::::l  |´    __       _,   \}  ヽ::::::::::::::::::|
     ∠::イ:::::::::::l    , ===、        ,.===、.`   ヽ};:::::::::::::!
       |::::;::::;;:l,.  /,. 'ひ~ヽ         /ひ~ ヽ\   レ‐、::::l
       レ´|;/丶! / {::::::::::::::}      {::::::::::::::} 丶   /.. |::::!
         l, 〈. |    丶::;;;::ノ      丶::;;;;::ノ     〉:: iヽ{
           \_.l .:::::::::.        ,       .::::::::::.  /:: /  `  いっしょにがんばろう
.            !          ,へ           、_ ,/
              ヽ、      i .|_____       ,イ::::/
.               > 、_.   | .|、  ノ   _, イ/|/
               _/   マ '' ‐-| .L ,,,,... -‐ ' ´y \___
         ,. '" ̄ヽ     \. ,!  |_,,.. -、/~ 〉'′ /  ヽ
.         /       \.     `/ "´     〉 /   /   `,
         {.         \  /   ー<;: { / ,. '′     l
          |         `'‐i′  _,,__ `Y/||´       |

770 名前:132人目の素数さん:2009/06/28(日) 10:21:12
>>768
むづかしいな。でも、20%くらいだろ。

771 名前:132人目の素数さん:2009/06/28(日) 19:06:13
20%くらいじゃないのっ?

772 名前:768:2009/06/28(日) 22:06:22
すんません。実は私も答えしらないです。。
計算機で計算した結果だと確かに20%くらいになりそうです。


773 名前:132人目の素数さん:2009/06/30(火) 23:46:14
>>769
解答はまだかね、ロリータくん!

774 名前:132人目の素数さん:2009/07/04(土) 06:37:47
>>769 エレガントな方法は思いつかん。

lim[t→0] {sin(t)}/t = 1 と倍角公式から
lim[x→0] {1-cos(x)}/(x^2) = lim[x→0] 2*[{sin(x/2)}/{2*(x/2)}]^2 = 1/2。
これより、f(x)={1-cos(x)}/(x^2) (x≠0) とおくと、
0<|x|<a (aはある正数)のとき 1/3 ≦ f(x) ≦ 2/3 とすることができるから、
|x|<a のとき (x^2)/3 ≦ 1-cos(x) ≦ 2*(x^2)/3。
上式の各辺を0からy (0<y<a) まで積分すると、(y^3)/9 ≦ y-sin(y) ≦ 2*(y^3)/9。
g(x)={x-sin(x)}/(x^3) (x≠0) とおくと、g(-x)=g(x)だから、結局 0<|x|<a ならば 1/9 ≦ g(x) ≦ 2/9。
0<|x|<a において、log|1-cos(x)| = log|x^2| + log|f(x)| = 2*log|x| + log|f(x)|、
log|x-sin(x)| = log|x^3| + log|g(x)| = 3*log|x| + log|g(x)|。
そして、lim[x→0] log|x| = ∞、lim[x→0] log|f(x)| = log(1/2)、
さらに 0<|x|<a のとき log(1/9) ≦ log|g(x)| ≦ log(2/9) だから、
lim[x→0] log|1-cos(x)|/log|x| = 2、lim[x→0] log|x-sin(x)|/log|x| = 3。
以上より、lim[x→0] log|x-sin(x)|/log|1-cos(x)|=3/2。

高校レベル縛りがなかったら、0 の近傍においてTaylorの定理より
sin(x) = x - (x^3)/6 + O(x^5)、cos(x) = 1 - (x^2)/2 + O(x^4)だから
x-sin(x) = (x^3)/6 * {1 + O(x^2)}、1-cos(x) = (x^2)/2 * {1 + O(x^2)}なので、
log|x-sin(x)| = 3*log|x| - log6 + O(x^2)、log|1-cos(x)| = 2*log|x| - log2 + O(x^2)、
log|x-sin(x)|/log|1-cos(x)| = 3/2 + {log(2/9)}/(4*log|x|) + O(1/{log|x|}^2)}。


775 名前:132人目の素数さん:2009/07/05(日) 04:14:27
先に断っておくと、解答は不明です。

n進法(n>1)で表された、一の位が0でない自然数aがある。
aのdigitsを反転させたn進数をbとする。(a=1234ならb=4321)
このとき、a/b=r が整数になる場合がある。問題は、

「n進法のとき、r として出てくる整数値を全て決定せよ。」

なお、r=1となるケースは自明なので考察から除外する。
たとえば十進法の場合、9801/1089=9, 8712/2178=4 であり、
この後300万くらいまで検索しても4と9以外は出てこない。

一般にn進法のとき、r=n-1 が常に出現することは示せたけど、
それ以上のことは全くわからない。
参考までに、n=3〜30 について、それぞれ300万まで調査した結果を貼っておく。

776 名前:132人目の素数さん:2009/07/05(日) 04:16:44
[n進法] rの値
[3]2
[4]3
[5]2,4
[6]2,5
[7]3,6
[8]2,3,5,7
[9]2,4,8
[10]4,9
[11]2,3,5,7,10
[12]2,3,5,11
[13]5,6,12
[14]2,3,4,6,9,13
[15]2,3,4,7,11,14
[16]3,7,15
[17]2,4,5,8,10,11,16
[18]2,5,8,17
[19]3,4,6,7,9,18
[20]2,3,4,6,9,13,19
[21]2,5,6,10,13,20
[22]10,21
[23]2,3,4,5,7,11,13,15,17,22
[24]2,3,4,5,7,11,19,23
[25]4,9,12,24
[26]2,8,10,11,12,25
[27]2,3,6,8,13,26
[28]3,6,13,27
[29]2,4,5,8,9,14,16,19,28
[30]2,4,5,9,14,29

777 名前:132人目の素数さん:2009/07/05(日) 04:57:57
なぜこんなことを考えたのか

778 名前:132人目の素数さん:2009/07/05(日) 07:17:43
>>775
10進数で r = 1,4,9 しかない理由

a の最上位の数字を x、最下位の数字を y とする

a = rb
の最上位を考えると
ry ≦ x < r(y+1)
最下位を考えると
rx ≡ y (mod 10)

上の2式を同時に満たす1桁の数の組 (x,y,r) で
r≠1,4,9 のものは存在しない

779 名前:132人目の素数さん:2009/07/05(日) 16:10:39
>>768って何の問題?

780 名前:132人目の素数さん:2009/07/05(日) 18:57:21
>>775さんの創作

781 名前:132人目の素数さん:2009/07/06(月) 00:16:56
>>769 エレガントな方法は思いつかん。

 f(x) = 1-cos(x) ≧ 0,
これをひたすら積分する。
 g(x) = x-sin(x) > 0 (x>0),
 (1/2!)x^2 -1 + cos(x) ≧ 0,         ・・・・・・・(1)
 (1/3!)x^3 -x + sin(x) >0 (x>0),       ・・・・・・(2)
 (1/4!)x^4 -(1/2!)x^2 +1 -cos(x) ≧ 0,     ・・・・・(3)
 (1/5!)x^5 -(1/3!)x^3 +x -sin(x) >0 (x>0),   ・・・・(4)

(1),(3) より
 1/2! ≧ f(x)/x^2 ≧ 1/2! - (1/4!)x^2,
 f(x)/x^2 → 1/2! (x→0)

(2),(4) より
 1/3! > g(x)/x^3 > 1/3! -(1/5!)x^2,
  g(x)/x^3 → 1/3! (x→0)

(左辺) = {3log|x| + log(g(x)/x^3)} / {2log|x| + log(f(x)/x^2)}

782 名前:132人目の素数さん:2009/07/08(水) 00:48:21
有名過ぎるかな・・・
==========

地球上で、南へ1km、東へ1km、北へ1km 歩くと、出発点に戻るような地点を、全て求めよ。
なお、地球は、完全なる球体とみなす。

(って、数学じゃないかな?)

783 名前:132人目の素数さん:2009/07/08(水) 01:44:28
カンタンだけど、答えがオモロイので・・・・・


ある国に、男女のカップルがN組いる。
この国では、皆、男の子しか望んでいないので、
どのカップルも、男の子が生まれるまで、せくーす&出産し続ける。
男の子が生まれた時点で、そのカップルはせくーすをやめる。
この国の、男の子と女の子の比率が収束する値を求めよ。

※不倫、死亡、不妊、中絶は考えない。
 また、子供同士のせくーすも考えない。


784 名前:132人目の素数さん:2009/07/08(水) 04:12:22
>>782
北極点。 南極点から1+(1/(nπ))km の同心円。

>>783
1:1
国全体で考えれば、第n子の性別は常に男女同数。

785 名前:132人目の素数さん:2009/07/08(水) 05:01:53
>>784
>北極点。南極点から1+(1/(nπ))kmの同心円。
どして?

786 名前:783:2009/07/08(水) 05:13:59
>>784

あーそーか。

自分、男の子の総数をpとすると、
女の子の総数は、2/p + 4/p + 8/p + ....... = p
って考えてたw

787 名前:786:2009/07/08(水) 05:15:31
あ、失礼。書き間違い。

2/p + 4/p + 8/p + ....... = p
じゃなくて、
p/2 + p/4 + p/8 + .......... = p

788 名前:132人目の素数さん:2009/07/08(水) 05:18:25
>>786
>自分、男の子の総数をpとすると、
>女の子の総数は、2/p + 4/p + 8/p + ....... = p
どして?

789 名前:132人目の素数さん:2009/07/08(水) 16:44:47
それらの男の子のうち半数には姉がいて
さらに半数にはその姉がいて
さらに半数にはその姉がいて

という意味だろ

790 名前:132人目の素数さん:2009/07/08(水) 23:49:32
>>782
北極点および南極点周辺

>>783
1/2


791 名前:>>783=>>786=>>788:2009/07/09(木) 10:06:03

>>788

>>787 みてね
=============

>>789
そうそう。ま、自分は、
男の子が全員でPいるとして、
第1子の女の子はP/2,
第2子・・・・・・・・・第1子が女の子だった家庭が女の子を産むと考えると、P/4
以下、P/8,P/16,P/32,P/64.......................... でこれらを全部たすと、Pに収束するって考えた。

でも
>>784

>1:1
>国全体で考えれば、第n子の性別は常に男女同数。
があっさりだねw


792 名前:782:2009/07/09(木) 10:14:50
>>790

<北極点および南極点周辺>・・・南極点周辺ってのをもちっと具体的に。
(ひっかけ?にはまってる可能性あるので)

>>785
たぶん考え方はわかってるんだけど、おかしいな、計算があわなくて(?)答えが微妙に違う。。。自分の計算ミスかしら
((1/(nπ))km)の部分。

793 名前:782:2009/07/09(木) 10:17:49
ちなみに、これ有名だろうなーとは思ったものの・・・。

MS社の面接問題です。(筆記じゃなくて面接)

794 名前:132人目の素数さん:2009/07/09(木) 14:06:45
>>792
>((1/(nπ))km)の部分。
失礼。ミスしてた。

地球が半径rの球だとすると
1+r・sin^-1(1/(2nπr)) kmだね

795 名前:132人目の素数さん:2009/07/09(木) 14:15:18
>>793
Googleって聞いたけど。

796 名前:132人目の素数さん:2009/07/09(木) 15:07:34
MSもGoogleも無かった頃からの頻出問題だよ。
頭の体操にも出ていたと思う。

797 名前:132人目の素数さん:2009/07/09(木) 17:07:04
頭の体操自体がパクリだらけだから、もっと前からってことだな。

798 名前:132人目の素数さん:2009/07/09(木) 18:13:02
頭の体操も含めて、昔の本だと答は北極点だけ

799 名前:132人目の素数さん:2009/07/09(木) 20:45:17
それは1巻だろう >北極点だけの頭の体操

800 名前:132人目の素数さん:2009/07/09(木) 21:01:38
でも実際はそんな地点ないんだけどね。

801 名前:132人目の素数さん:2009/07/09(木) 21:02:05
ついでにMSの入社試験で出た、というのは
とあるビジネス書の煽り文句だけど、実際には出ていない。

802 名前:132人目の素数さん:2009/07/09(木) 21:24:00
東へというのはジョジョに右や左に曲がっていくのかまっすぐ進むのか。


803 名前:132人目の素数さん:2009/07/10(金) 06:10:51
球面上でどうやってまっすぐ東に進むんだ? 赤道にでもいるのか?

804 名前:132人目の素数さん:2009/07/10(金) 07:45:25
犬が西向きゃ尾は東
犬に聞け

805 名前:132人目の素数さん:2009/07/10(金) 16:19:50
>>801
>とあるビジネス書
「ビル・ゲイツの面接試験」かな?
これみると、古典だけど、MSがアレンジしたとか書いてあった・・・ようなきが・・・

806 名前:132人目の素数さん:2009/07/10(金) 16:21:42
>>803
東経、西経ってあるじゃん

807 名前:132人目の素数さん:2009/07/10(金) 19:42:50
>>806 
あるよ。 

経線に沿って進むということは
南北方向に動くことだけどね。

また、緯線に沿って進むということは
東西に動くということだけど
赤道以外では、曲がって進むことになる。

808 名前:132人目の素数さん:2009/07/10(金) 20:17:49
東京から見て、アルゼンチンは真東の方向にあるんだっけ?


809 名前:132人目の素数さん:2009/07/10(金) 21:51:51
赤道以外では「東に1km歩く」と「東を向いて1km歩く」で動きが変わっちゃうんだよな。
しかも「東を向いて1km歩く」も言葉のニュアンスでどっちとも取れるような・・・

810 名前:132人目の素数さん:2009/07/10(金) 21:57:18
進み始めるときに「こっちが東!」って決めてあとはずっとその向きに進むと大円コースで、
常に方角を確認しながら進んでいくと同経度線上を進むことになる、でいいんだっけ


811 名前:132人目の素数さん:2009/07/10(金) 23:55:50
>>810

あってるんじゃないかな・・・
と思ったら、
「同経度線上を進む」じゃなくて、「同緯度線上」じゃない?
ややこしいけど。

812 名前:132人目の素数さん:2009/07/11(土) 00:10:12
>>811
指摘の通りだ、経度を保つことと同緯度線上を進むことを見事にごっちゃにしてた

813 名前:132人目の素数さん:2009/07/11(土) 00:23:30
なんかまだおかしいな、もうやだ

814 名前:132人目の素数さん:2009/07/13(月) 13:57:45
横が緯線で縦が経線。
「横井さんが磔刑(横緯縦経)」と覚えろと中学で習った。

815 名前:132人目の素数さん:2009/07/13(月) 14:01:15
「北経」や「東緯」という言葉を見て読んで気持ち悪い、なんか変だと思えるなら
そんな憶え方は必要ない。

816 名前:132人目の素数さん:2009/07/13(月) 14:08:16
なんにせよ中学でそれは遅いだろ・・・
漢字の意味を教えるならまだしも

817 名前:132人目の素数さん:2009/07/13(月) 15:12:14
勉強に速い遅いはなかろう。
はじめて習ったというのならともかく
間違えないようにするための方法でしかないんだから。

818 名前:132人目の素数さん:2009/07/13(月) 16:49:21
まあ授業の合間の豆知識みたいなモンだからw
でも覚えやすかったからそれ以降はそれで覚えてる。

819 名前:132人目の素数さん:2009/07/13(月) 21:11:10
>>814
磔刑の読みが分からなかった。はりつけけいかと思った。

820 名前:132人目の素数さん:2009/07/13(月) 21:41:02
>>819
某神父の名言はあまり浸透していないようで残念だ

821 名前:132人目の素数さん:2009/07/13(月) 23:48:04
素数でも数えとけ

822 名前:132人目の素数さん:2009/07/14(火) 11:48:40
素数が1つ、
素数が2つ、
素数が3つ、
素数が4つ、
素数が5つ・・・・・

823 名前:132人目の素数さん:2009/07/25(土) 07:44:52
3個のさいころを同時に振るとき、出る目の積が12の倍数になる確率を、いろんな方法で求めよ

824 名前:132人目の素数さん:2009/07/25(土) 08:09:32
>>823
まず、そのいろんな方法とやらを列挙してみなされ。


825 名前:132人目の素数さん:2009/07/25(土) 08:21:02
てけと〜にプログラムくんで
てけと〜にシュミレーションすれば
だいたいの当たりはつくけどこれはあり?


826 名前:132人目の素数さん:2009/07/25(土) 08:27:43
てけと〜はあかん。
3個のサイコロの出目の組216通りを全部書き出して、積が12になっている組を数えるプログラムを書く

これが一つ。
ひとつ、人間コンピュータになって紙に書き出してみ。


827 名前:132人目の素数さん:2009/07/25(土) 08:30:53
>>823
エレガントに解いたぜ!
ふふふ…

828 名前:132人目の素数さん:2009/07/25(土) 08:32:39
10万回くらいやればだいぶ良い値になると思うんだけど
まぁ数学ではないな

829 名前:132人目の素数さん:2009/07/25(土) 08:34:08
http://detail.chiebukuro.yahoo.co.jp/qa/question_detail/q1019928635

8の倍数になる確率、間違がっとるよな?

830 名前:132人目の素数さん:2009/07/25(土) 08:36:46
さいころの目に現れる12の約数は1,2,3,4,6の6種。12の素因数分解は、2*2*3。
これより順番違いは同じものとして、掛けて12になる3つの目の組を全部書き出す。

831 名前:132人目の素数さん:2009/07/25(土) 08:44:13
>>829
3つのさいころの出た目の積が8の倍数になる確率が3/8だと!
んなアホな
それとも俺がおかしいのか?

832 名前:132人目の素数さん:2009/07/25(土) 08:48:27
>>782
∞^2
こんなの数学の答えじゃねぇ。

833 名前:132人目の素数さん:2009/07/25(土) 10:51:32
>>832

平面上の点の濃度は実数の濃度と同じ

834 名前:見方によってはかなりインチキ臭い国際大会:2009/07/25(土) 16:26:13

>2009年:1位-中国、2位-日本、3位-ロシア、4位-韓国、5位-北朝鮮、6位-アメリカ

>国際数学オリンピックの引率の先生がラジオで言ってたんだけど、問題は前日に配られて、
>それを言語のできる " その国の引率の先生 " が各自翻訳するらしいです。
>だからと言って生徒に、問題や解答が事前に漏れてるとは言ってませんでしたよ。

前からこの辺りが胡散臭いと思っているんだけど、見方によってはかなりインチキ臭い国際大会。

835 名前:記憶馬鹿には絶対解けない数学問題集:2009/07/25(土) 16:30:14

4角柱の問題 → http://www.geocities.jp/eig35153/m-1/Mobius.html
球体から反射された光線が到達する地点 → http://www.geocities.jp/eig35153/m-1/Q-4.html
反転ゲームの最短回数 → http://www.geocities.jp/eig35153/m-1/h-1.html
( 縦横とも2n個の時の一般解も出して頂けると、すごいと思います )

アリの戦争 → http://www.geocities.jp/eig35153/m-1/b-h.htm
立方体の通路 → http://www.geocities.jp/eig35153/m-1/l-1.html
( 頂点から頂点までの通路は、他の通路と交差している交差点があっても直進する )

回転する光の通過速度 → http://www.geocities.jp/eig35153/m-1/4-1.html
入れ子になった回転リングの軌跡 → http://www.geocities.jp/eig35153/m-1/ling-1.htm

836 名前:なるべく予備知識無しで解いて欲しい数学難問:2009/07/25(土) 16:33:51

問題 : ミサイル曲線
xy平面の原点に地対空ミサイルが設置されている。 時刻t=0に上空(0,h)を敵戦闘機が速さvでx軸に平行に
xの負の向きに一定の速さvで飛行している。 このミサイルは常に目標をめがけて一定の速さVで飛行する。 時刻t=0で発射されたミサイルの
(1) 軌道を表す曲線の方程式を求めなさい。 (2) 戦闘機が撃墜される時間はいくらか。
ただし v<V とする。   戦闘機もミサイルも点と考えてよい。

問題 : 伸びるゴムひも上を移動する虫
1mのゴムひもの左端を固定します。左端に虫をおきスタートと同時に虫がゴム上を5cm/sで歩き、
ゴムひも自体を右端を5cm/sで引き延ばした場合に虫が右端に到達する時間を求めなさい。

問題 : 蛇口から流れ落ちる水流の曲線
水道の蛇口から少量の一定の水を流すと落下につれて水流が細くなってきます。
蛇口の中心から下方へx軸、それと垂直方向にy軸をとった場合、落下水流の形を示す方程式y=f(x)を求めなさい。ただし粘性率=0
S:蛇口の断面積、  v0:蛇口での流速、   g:重力加速度とします。 また水は自然落下するとします。

837 名前:132人目の素数さん:2009/07/25(土) 20:02:57
どうだすっきりしたか

838 名前:132人目の素数さん:2009/07/25(土) 20:19:36
ふぅ・・・
さて勉強するか

839 名前:132人目の素数さん:2009/07/25(土) 22:30:43
>>836
ニュー速+のすう折スレでその問題見た。

けど、いい問題じゃないよね。

840 名前:132人目の素数さん:2009/07/26(日) 01:16:34
有名小学校の入試に出てきそうな問題。

たけし君は薄暗いへやにいて、懐中電灯を持っています。
へやには鏡があり、たけし君は懐中電灯の光を鏡にあてて遊んでいました。
たけし君が鏡に映ったあるところに懐中電灯の光をあわせたところ
たけし君は「あっまぶしい!」といいました。
さて、たけし君は鏡に映った何に懐中電灯の光を合わせたのでしょうか。



841 名前:132人目の素数さん:2009/07/26(日) 01:33:09
光に反応して爆発する爆弾

842 名前:132人目の素数さん:2009/07/26(日) 06:31:29
>>840
自分の目

843 名前:132人目の素数さん:2009/07/26(日) 07:00:00
尼武士。


844 名前:132人目の素数さん:2009/07/26(日) 07:03:41
>>840
懐中電灯

845 名前:840:2009/07/26(日) 08:24:09
>>844
正解!



846 名前:132人目の素数さん:2009/07/26(日) 08:30:50
>>845
鏡が複数枚ある場合はうまく行かない場合がある。

847 名前:132人目の素数さん:2009/07/26(日) 08:42:08
たしかに複数枚の時はかんがえてなかったけど。
うまくいかない場合というのはどういう場合?
『「鏡に映った鏡」に映った懐中電灯』は『鏡に映った懐中電灯』と違うと考えるってこと?



848 名前:132人目の素数さん:2009/07/26(日) 08:46:02
840=845=847です。


849 名前:132人目の素数さん:2009/07/26(日) 08:46:30
今あわせたが光が見えん。

850 名前:840:2009/07/26(日) 08:55:31
えー。
ここは数板なんだから、もちっと具体的に教えてくれ。


851 名前:132人目の素数さん:2009/07/26(日) 10:46:13
鏡の中の懐中電灯に向かって照射じゃなくて、鏡の上の懐中電灯の位置に照射ってこと?

852 名前:132人目の素数さん:2009/07/26(日) 11:07:22
「鏡に映ったあるところに懐中電灯の光をあわせた」という言葉の意味が
実は明確でない。
その鏡が反射率100%だったら、鏡面上で懐中電灯の光の
当たっている場所なんて認識できないはずだ。
その場合、「懐中電灯の光をあわせる」というのは、鏡の向こうにあるように
見えている像に対して懐中電灯のビームを向けるという意味になるから、
当然、像の目に向ければ、像の懐中電灯からの光は自分の目に
向けられることになる。

鏡として映っている像の他に、表面での乱反射成分として懐中電灯の光の
当たっている範囲が見えていて、その光円を、像の何と重ねるかという
ことであれば、もちろん、懐中電灯の像と光円を重ねれば
像の懐中電灯からのビームが目に向かうからまぶしくなる。

で、どうやら出題者の意図は後者のようだが、
そうすると、たとえば2枚の鏡を90度になるように置いたような場合には
照らすべき場所は当然、鏡面上の懐中電灯の見えている場所ではない。

853 名前:840:2009/07/26(日) 11:25:13
なんだか自分でも問題の意図があやふやになってきたが…

問題の解釈として
1.まぶしいと感じた→ビームは懐中電灯の像に合わせられている。
2.ビームは懐中電灯の像に合わせられている。→まぶしいと感じた。
どちらを問うているかというと1のつもり。

で、鏡が複数枚あった場合でも
「まぶしいと感じた」→「ビームは懐中電灯の像(『「鏡に映った鏡」に映った懐中電灯』なども含む)に合わせられている」

が成り立ちそうな気がするがどうだろうか。




854 名前:132人目の素数さん:2009/07/26(日) 12:14:10
>>853
>1.まぶしいと感じた→ビームは懐中電灯の像に合わせられている。
通常、鏡において「像」といった場合は、鏡面ではなく、
鏡の向こうにあるように見えるもののことを指す。
だから、「像」という言葉を使うならば、懐中電灯を向ける先は、
あくまでも自分の目の像でなければならない。
その結果、懐中電灯の光が鏡面に当たる位置が、
自分の目と、懐中電灯の像を結ぶ直線上にある、というだけ。

90度に合わせた鏡の場合も、懐中電灯を向ける先は
鏡の向こうの自分の目。
その結果、自分の目と、懐中電灯の光が最初に鏡に当たる位置を結んだ先に
何があるかは不明。

自分で図を描いて確認してみそ。

(そもそも、その1と2の違いなんて、だれも言及してねーよ。)

855 名前:840:2009/07/26(日) 13:04:35
うーん。その辺の用語の使い方はちゃんと理解できてないorz.

じゃあ、90度に合わせた鏡の場合、鏡のどこに光円を重ねればいいの?
「鏡面上の懐中電灯の見えている場所」でないなら、
「鏡面上のにもう一つの鏡が見えていて、その鏡の中の鏡の上に懐中電灯が見えている場所」
だと思うんだが。

とりあえず、飯食ってきます。


856 名前:132人目の素数さん:2009/07/27(月) 14:54:56
数学っつーか物理の問題じゃないか?

857 名前:132人目の素数さん:2009/07/27(月) 18:04:47
物理の問題だと思う理由を述べよ

858 名前:132人目の素数さん:2009/07/27(月) 18:16:13
鏡使った光の反射とか物理の範疇じゃね?

手の位置が目の位置から○○cm下で鏡が地面に対し○○度で設置されて〜とかの要素があれば数学的だけど。


859 名前:132人目の素数さん:2009/07/27(月) 19:06:40
(光線の)ベクトルが(鏡の)平面で面対称に反射すると考えれば
十分数学的だろう

860 名前:132人目の素数さん:2009/07/27(月) 19:08:53
おそらく858には
「数学といえば計算」
というイメージがあるのだろう。

861 名前:132人目の素数さん:2009/07/27(月) 19:11:55
幾何光学は幾何学か光学か

862 名前:132人目の素数さん:2009/07/27(月) 21:21:01
工学

863 名前:132人目の素数さん:2009/08/01(土) 15:58:56
>>861
心理学は社会学か統計学か
と同じくらい意味の無い質問じゃないか?

864 名前:132人目の素数さん:2009/08/01(土) 18:19:28
心理学はどちらでもない
あえて分類するなら人文科学

865 名前:132人目の素数さん:2009/08/20(木) 18:40:46
数学じゃなくて数学者の問題だけど
Xを除くA〜Zの25個のアルファベットについて、名前がそれで始まる数学者をあげよ
例) G:Gauss or Galois or Grothendieck

866 名前:132人目の素数さん:2009/08/20(木) 18:45:17
いかん、外人の場合はどれが苗字でどれが名前とか分からんから
やっぱ865はフルネームの中のどれでもいいということにしてくれ

867 名前:132人目の素数さん:2009/08/20(木) 20:14:16
Xを除くのはなんでなん?

868 名前:132人目の素数さん:2009/08/20(木) 20:30:00
http://en.wikipedia.org/wiki/List_of_mathematicians


869 名前:132人目の素数さん:2009/08/20(木) 21:46:21
X以外という条件なら>>868とか見なくても思い浮かべられるだろってことです

870 名前:132人目の素数さん:2009/08/23(日) 17:21:09
f(x,t)=sin(x)+sin(x+t)と置いた時
∫[0,2π]∫[0,2π]|f(x,t)|dxdt=4∫[0,π]∫[-t/2,π-t/2]f(x,t)dxdt
を証明せよ。

871 名前:132人目の素数さん:2009/09/02(水) 22:59:35
>>864

なんにでも心理学ってつければそれらしい日本語になるよ

http://ja.wikipedia.org/wiki/%E5%BF%83%E7%90%86%E5%AD%A6

872 名前:132人目の素数さん:2009/09/02(水) 23:03:24
>>502

>√N以下の約数



ってちゃうの?

873 名前:132人目の素数さん:2009/09/03(木) 00:13:24
>>871
付けるのはかまわないが、心理学である以上は人文科学。
それともあなたが人間以外の心理も考えたいというのならとめはしないが

874 名前:132人目の素数さん:2009/09/03(木) 00:30:36
>>872
整数論の問題を出し合うスレ
http://science6.2ch.net/test/read.cgi/math/1106654316/502-503


875 名前:132人目の素数さん:2009/09/03(木) 05:33:54
>>873
心理を物理現象として捉える分野(数理心理学)は
心理学とつくけど人文科学とみなすのはどうか、という話もあるよ

876 名前:873:2009/09/03(木) 12:11:40
>>875
そういや、この板にも数理心理学のすれあるね。

>>873
いいたいことはわかる。
まあ個人的に、学際主義なので、あんまり人文、とか理系とかにこだわってない。

あーあと、正式な学術用語じゃないかもしれないけど、経済心理学とか芸術心理学とかもある。
あ、でも後者は、美学とか美術史とか表象文化論とかかな。

877 名前:132人目の素数さん:2009/09/03(木) 12:12:44
>>874

ごめんそれ誤爆

878 名前:876:2009/09/03(木) 12:35:29
ふと思ったが、人工知能なんてまさに、心理学っぽいけど、人文科学に限定されないんちゃうかな。

879 名前:132人目の素数さん:2009/09/03(木) 12:38:24
面白い問題おしえて〜な

880 名前:878:2009/09/03(木) 12:49:23
確かに、おもろい問題ほしいところ。思いつかないから自分の知識から。

任意の四面体において、4つの面の面積がすべて等しいとき、
これら4つの面はみな合同であることを証明せよ。

===
って有名?
ただ、文字だけだと証明ここに書きづらいな。

881 名前:878:2009/09/03(木) 12:57:33
えーと、もひとつ。でもぱくり。今売ってる大学への数学にのってたSEGの広告から。
(とりあえず、補題なしで問題書いてみる)
===
酔っぱらいのA氏は、店を出てからxメートル歩いときに、
次のちょっとの距離△xメートル歩く間に立ち止まって寝てしまう確率が△x/(10^3)である。
A氏が店から1キロメートル離れた自宅に無事たどりつける確実を求めよ。
(広告にはのってなかったけど、寝たら一生起きないことを前提とする)

882 名前:132人目の素数さん:2009/09/03(木) 13:20:41
麻雀知らないとわからないけど。

「天和かつ字一色であがる確率は?」

●東場第1局1本場で自分が親であることを前提とする
●4人麻雀
●いかさまなし
★ヒント:(麻雀の公式ルールにのっとった)微妙なひっかけあり
●九種九牌は考えないことにする

※てか、九種九牌と天和が同時に成立する場合、ルール上どうなるんだろう・・・

883 名前:882:2009/09/03(木) 13:26:41
あ、大三元とかついて3倍役満になっちゃう場合も含めます。
(まあ、実際、そんなのむちゃくちゃ確率0にちかいけど)

884 名前:132人目の素数さん:2009/09/03(木) 15:14:37
>>882
麻雀は上がり優先だよ。
ひっかけって、清老頭だと無い話かな?ひっかけの言葉を見る前に真っ先にそれを考えたから違うか。

885 名前:882:2009/09/03(木) 16:56:38
>>884
清老頭、まずあがれないから、存在すら忘れてた。でも、字一色とは方向性が違うような。

あと、ひっかけっていっても、たいした引っかけじゃないかなぁ・・・というか、
数学的なひっかけでなく、麻雀初心者だとひっかかるかも、という^^

886 名前:132人目の素数さん:2009/09/03(木) 18:29:17
使える牌が6種類と7種類じゃ勝手が変わってくるってんだろう

887 名前:132人目の素数さん:2009/09/03(木) 18:38:27
公式ルールってどこの?


888 名前:132人目の素数さん:2009/09/03(木) 18:42:02
まさか宇宙麻雀準拠ではあるまい

889 名前:>>882 = >>885:2009/09/03(木) 19:22:00
>>886
>使える牌が6種類と7種類
ごめんそれ知らない・・・。

てか、言っちゃうと、チートイツになってる場合です。

>>885 の最後にわざとらしく^^(ニコニコ)って書いたから、気づくかなーとおもったw

890 名前:132人目の素数さん:2009/09/03(木) 20:00:00
公式ルールでは字一色七対子は倍満。


891 名前:132人目の素数さん:2009/09/03(木) 20:59:44
>>889
>>884で当たりなわけね

892 名前:132人目の素数さん:2009/09/03(木) 21:50:00
>>884>>886が清老頭七対子がないと言ってるのに
>>885>>889がわかってない。


893 名前:132人目の素数さん:2009/09/03(木) 22:00:21
11112222333344は七対子?


894 名前:132人目の素数さん:2009/09/03(木) 22:03:04
ならない。ローカルで四枚使いの七対子を認める場合はあるが。

895 名前:132人目の素数さん:2009/09/03(木) 22:04:25
>>893
同じのなら
三暗刻とか二盃口とかの方がいい気がする

896 名前:>>885=>>889:2009/09/03(木) 22:36:20
あ-、「清老頭」ってよか、「清老頭七対子」ってことか。なるほど。

清老頭七対子(必ず、同じ稗が4つある)がチョンボになるかどうかは、ローカルルールしだいだよね。

で、字一色七対子の場合、「同じの稗が2つずつ×7」って場合もありうるんだよね。
(・・・・・・・って、かなりのヒントだけど)
中国だと、こういう場合2倍役満にする場合もあるみたい。

で、めんどいので、
同じ稗が4つあるような「字一色七対子」はチョンボにしちゃいましょう。


897 名前:132人目の素数さん:2009/09/03(木) 22:42:11
白白白 発発発 中中中 白発中 一一

898 名前:132人目の素数さん:2009/09/03(木) 22:43:21
>>896
ヒントって何のヒント?


899 名前:882 >>898:2009/09/03(木) 23:07:59
>>887 え、「白発中」を1つの面子にするようなローカルルールてあるの?

>>896
●字稗とにらめっこすればわかる
●それか、>>896 が、「簡単すぎてヒントじゃないよこんなの」って思ってるかもしれない

のどっちか?

900 名前:>>882:2009/09/03(木) 23:19:22
しかし、上のまあじゃんの問題、いろいろめんどいので、
15分から30分で解ける麻雀問題で、小休止。

「二盃口をあがるときの牌のくみあわせの数を求めよ」

条件
●アカウーのたぐいはなしとする
●ロン、ツモは同一とみなす
●4人麻雀
●いかさまなし

理屈的には、これが麻雀じゃないとしたら、
大学入試とかでもでそう。

901 名前:132人目の素数さん:2009/09/03(木) 23:20:34
宇宙麻雀

902 名前:>>900 ◆xqjbtxNofI :2009/09/03(木) 23:24:38
●追加
11112222333344 みたいに、ローカルルールによっては、ちーといつとみなせるような場合
も、ちーといつとはみなさない
(同じ稗4枚があっても、それをといつ2ペアとはみなさない。)

●自信がすこしないので、
トリップに半角で、答えを書いといた。

903 名前:>>900:2009/09/03(木) 23:30:59
>>901

ま、まーじゃんとしては、宇宙麻雀だとしても、
順列組み合わせの問題つくるのが楽なのよ。許して。
((ローカルルールを追い詰めると、めんどいけど)

904 名前:132人目の素数さん:2009/09/03(木) 23:43:10
>>899
宇宙麻雀ていうね

905 名前:132人目の素数さん:2009/09/07(月) 13:43:58
正の整数を1から順にカウントしていったとき、
素数が三回以上連続でカウントされることは有り得るか否か。

906 名前:132人目の素数さん:2009/09/07(月) 15:42:47
>>905
106900031
「いちおくろっぴゃくきゅう」「じゅうまんさん」「じゅういち」
とかはダメ?

907 名前:132人目の素数さん:2009/09/07(月) 19:03:36
22・23・24
にじゅう【に・にじゅうさん・に】じゅうよん

これでいいじゃん。

908 名前:132人目の素数さん:2009/09/07(月) 19:08:02
で、でででわっ、かっかかぞっかぞっかぞえっますっ。
い、いっいいちっ。に、ににっ。ささっさっさんっ。よっよよん。
このあたり--------↑

909 名前:132人目の素数さん:2009/09/08(火) 20:25:48
とんちで答える問題なの?
「ない」でいいと思うが。

910 名前:132人目の素数さん:2009/09/08(火) 20:35:10
面白い問題おしえて〜な

911 名前:132人目の素数さん:2009/09/09(水) 21:58:42
>881
1/e

912 名前:132人目の素数さん:2009/09/10(木) 17:39:52
xy座標平面において、
「原点を中心として半径が1024である円Cがある。」
円Cの円周上に点M(a,b)があり、
点Mから、
x軸上におろした垂線の足を点P、
y軸上におろした垂線の足を点Q、
とする。

∠MOP=π/64 であるとき、線分PQの長さを求めよ。



===
で、ここまで読んだ段階で・・・・
3分で答えやがれお願いします。

913 名前:912:2009/09/10(木) 17:41:54
ちなみにトンチではないよ

914 名前:132人目の素数さん:2009/09/10(木) 17:43:36
>>912
無駄な設定は省いてもっと問題を要約してくれ

915 名前:912:2009/09/10(木) 17:46:28
追加問題

(2)
>>912 において、
∠MOP = π/3 であるとき、線分PQの長さを求めよ。


===
これも、3分で答えてくださいやがれ。

916 名前:912:2009/09/10(木) 17:47:44
>>914

みごとにひっかかってますよw

917 名前:912:2009/09/10(木) 17:50:28
いや、あるいみ、「数学的なトンチ」なのかもしれん

918 名前:132人目の素数さん:2009/09/10(木) 17:55:40
1024

919 名前:912:2009/09/10(木) 17:58:58
>>918

正解。っていうか、答えわかるまでどのくらいかかった?

920 名前:132人目の素数さん:2009/09/10(木) 18:02:34
>>916
無駄な設定は省いて
無駄な設定は省いて
無駄な設定は省いて

921 名前:912:2009/09/10(木) 18:04:58
>>920

いや、「無駄な設定」が、この問題のポイントっしょ?

922 名前:912:2009/09/10(木) 18:08:47
ちなみに、とある本・・・マイクロソフトの新入社員はみんな読まされるらしい・・・に載ってる問題を、
数字を変えて、さらに少しいじった。
てか >>921 の発言がヒントになってしまった・・・



923 名前:132人目の素数さん:2009/09/10(木) 19:15:40
Oって何

924 名前:132人目の素数さん:2009/09/10(木) 19:27:51
パズル板ならともかく、数学板では出題者のセンスの無さが疑われる問題だな。

ちなみに、少なくともマイクロソフト本社では「新入社員がみんな読まされる本」なんてないし、
日本でもそんな本は無い。ほかの国は知らんが。

925 名前:912:2009/09/10(木) 20:28:09
>>923
あ、Oは原点。

>>924
てか、ごめ、もとの本だと、
単に、円と(>>912みたいな)長方形の図があって、で円の半径は5とかって書いてあって、で、
ここ(線分PQのとこ)の長さは?ってかんじの問題。

もちろん、∠MOP=なんとか、なんて条件は、自分がつけたしただけ。
で、数字も1024とか64とか意味ありげなのに変えてるから。

↓元ネタはこの本。
いかにして問題をとくか
http://www.amazon.co.jp/dp/4621045938

しかし、いま気づいたんだけど、このアマゾンの本のページの「この商品を買った人はこんな商品も買っています」の本が、
みんなおもろそうだな。このスレ的な本が多そう。

926 名前:132人目の素数さん:2009/09/10(木) 20:58:50
>>925
amazonではティモシーガウアーズとかシャーマンスタインとか結城浩とか出てきたけど、
こういうのが好きなら数学板よりもパズル板に行くべきだと思う。
少なくとも数学的には面白いものじゃない。

927 名前:132人目の素数さん:2009/09/10(木) 22:51:28
最近この板はこういうかわいそうな子が多いな。何でだろ

928 名前:925:2009/09/10(木) 23:39:42
>>926

パズル板というものの存在を知らなかった。
結城浩しか名前を知らないけど、
今パズル板をさらっとみたら、なるほどこれはパズル板だわな。

929 名前:132人目の素数さん:2009/09/10(木) 23:41:30
この問題、頭の体操だか何だかで昔みたことがある。
幾何学的には、長方形OPMQの対角線の長さが等しいことから
即座に「1024」と分かる。しかし、


半径をrとし、∠MOP=θ とおけば、(a,b)=(rcosθ,rsinθ) と表せるので
PQの長さは√(a^2+b^2)=√(r^2(cos^2θ+sin^2θ))=r (角度θに依らない)となる。


このように、普通に極座標表示しても、暗算レベルの超簡単な計算ですぐに
答えは出でしまうので、数学パズルとしては破綻しているというか、全然面白くない。
もちろん、普通の数学の問題として見ても面白くない。

まあ、こういうツールを知らない「中学生」くらいまでなら、面白いと
思うんじゃないかな。出題者が高校以上なら、いい加減こういう低レベルな
世界からは卒業しろと思うがね。

930 名前:132人目の素数さん:2009/09/11(金) 00:47:23
>>929
>数学パズル

パズル板少し見てきたけど、、、
「1,0,-1,0,★・・・・・・っていう数列の★は?」
みたいなのがあったりして(「1,1,2,3,5,8,☆」の☆は? ってのもあった)
まぁ、この板の人たちのほうがパズル板よりかしこそうなことだけはわかった。

で、なんだか、
このスレ的に、どんなのが「数学的におもろい」パズルで、
どんなのがそうじゃないかわかんなくなってきた・・・。

たとえば↓みたいなのはどうなの?(おもいつきなので、答えはだしていない)
=====

将棋盤(通常の9 x 9 ます)と桂馬(動き方は通常と同じ)のコマ1つのみを用いる(他のコマは使わない)

===========
まず、桂馬を81コマのどこかに、ランダム(同じ確率)におく。・・・(1)
以下、桂馬を、ランダム(50%,50%で、同じ確率)で移動させることを、可能なかぎり続ける。
(桂馬を動かせるかしょが、1カ所のときは100%の確率でそこへ移動させる)
桂馬を動かせるかしょがなくなったじてんで、操作を終了とする。
「成り」は考えない
===========

「最初の(1)も含めた一連の操作」は全部で何種類存在するか?
(対称性は一切考えない)

みたいな。
将棋ってパズルの一種だけど・・・。

931 名前:132人目の素数さん:2009/09/11(金) 01:49:54
ウルトラクイズの東京ドーム(か後楽園球場)の予選問題

どの年のカレンダーにも「13日の金曜日」が存在する・・・・・○か×か?

932 名前:132人目の素数さん:2009/09/11(金) 06:47:44
13日の曜日は金曜が一番多い

933 名前:132人目の素数さん:2009/09/11(金) 07:58:55
>>930
面白いの定義は人によるところで,スレでも何度も揉めてるけれど,
解法に何らかの新規性があるものは面白いんじゃないかと俺は思う.

912の問題は何をやっても一瞬で解けるのを問題文でごまかすタイプで,
多くの人が面白くないと感じると思う.
一方,930の問題はちょっとうまい数え方があるので912より良いと思うが,
それも標準的な手法(動的計画法)なので,いまさら感がある.
n×nに一般化すると少し手間のかかる問題になるけれど,解法自体は難しくない.

934 名前:132人目の素数さん:2009/09/11(金) 09:00:51
【デュードニー】 パズルのネタ本 【藤村幸三郎】
http://gimpo.2ch.net/test/read.cgi/puzzle/1092488979/

へこたれない野郎がいるなw
ネタ本の一冊や二冊で天狗になってないでパズル板で揉まれてこい

935 名前:132人目の素数さん:2009/09/11(金) 11:32:52
>>931
http://blog.livedoor.jp/enjoy_math/?p=2

936 名前:132人目の素数さん:2009/09/11(金) 12:40:10
レントン先生にも出てた

937 名前:132人目の素数さん:2009/09/22(火) 17:21:21
凸六面体を、ある平面に正射影した時、その正射影はN角形になった。

Nの最大値を求めよ。

938 名前:132人目の素数さん:2009/09/22(火) 17:43:09
8角形

939 名前:132人目の素数さん:2009/09/22(火) 21:33:29
すまん、間違った。

凸六面体を、ある平面に正射影した時、その正射影はN角形になった。
Nの最大値を求めよ。

凸M面体の時はどうなるか。


940 名前:132人目の素数さん:2009/09/28(月) 14:53:30
2(N-2) だろうか。

941 名前:768:2009/09/30(水) 22:40:00
ども。768です。
768の類似問題どぞ。
自分的にどういう議論を展開すればいいのかよくわからん問題です。

正方形のなかに円盤を敷き詰めたい。
その際、円盤同士は重ならず、円盤は正方形の境界とは交わらないようにする。
敷き詰める円盤は異なる半径のものを任意の個数使ってよい。

これを次の手順で円盤を敷き詰めていくとする。
1.正方形の中で円盤の敷かれていない点をランダムに選ぶ。
2.その点を中心にもち、他の円盤と正方形の境界と交わらない最大の円盤を敷く。

この操作をn回繰り替えしたとき、(円盤がカバーしている面積/正方形の面積)=r(n)とおく。

lim [n→∞] r(n) = 1といえるか?

すんません。実は私も答えしらないです。

942 名前:132人目の素数さん:2009/10/01(木) 13:48:30
急にレベルの低い問題ですまん
どっかの中学の入試問題らしいんだが、一瞬おやっ?となって面白かったので


忘れんぼ村のA君が「1」〜「20」までの整数を全部覚えようとしています。
以下のような条件で覚えていくとき、20の数字を全部覚えるには最短で何日かかるでしょう?

条件
1、覚え始めは、まだどの数字も覚えていない。
2、1つの数字を覚えるに1日かかる。
3、2のつく数字は覚えにくいので、2のつく数字を覚えた日には、それまで覚えていた数字を、覚えた順が古いものから6個忘れてしまう。
4、覚えている数字が5個以下のときは、2のつく数字を覚える事が出来ない。
5。覚えたい20個以外の数字は覚えられない。

943 名前:132人目の素数さん:2009/10/01(木) 15:37:08
44日

944 名前:132人目の素数さん:2009/10/01(木) 16:02:07
>>943
正解です
でも小学生に出すにはなかなか面白い問題かなと

945 名前:132人目の素数さん:2009/10/02(金) 22:55:49
ある山の頂上から、真西に向かって平地に降りる道と、真東に向かって平地に降りる道がある。但し、どちらも単調減小に降りてゆく(下りばかり)とは限らず、途中何回かちょっとした登りもあるとする。平坦な部分もあっても良い。

頂上に二人の人がいる。この二人それぞれが真西、真東の道を通って平地にまで下るとする。但し同一時刻には二人は同一の高さに存在しなければならない。(即ち一方あるいは両方の人が少しは逆戻りもしなければならない。)これは可能である事を示せ。

更に真北、真南に向かう下山路があり、頂上に4人が居て、それぞれ異なる方向へ降りる場合これは可能か

946 名前:132人目の素数さん:2009/10/02(金) 23:17:05
>>942
こういう問題は何から手とつけたらいいのか分からない!
おしえてサミヤどん!

947 名前:132人目の素数さん:2009/10/02(金) 23:48:37
凸凹の数に関する帰納法とかどうだろう。

もっと解析的な解き方のほうがきれいそうだけど、思いつかない。

948 名前:132人目の素数さん:2009/10/03(土) 00:10:04
>>947
仮定するのは単に「連続曲線」だろう。その場合、高木関数みたいなのを
持ってくれば凹凸どころの問題じゃない(各点で微分不可能)から、
帰納法とか使えない。

949 名前:132人目の素数さん:2009/10/03(土) 00:15:13
>>948
そこまで考えねーよ
常識的に解釈してくれ

950 名前:132人目の素数さん:2009/10/03(土) 01:27:06
>>945
一方が、1分あたり標高χm下がるような速度で登るor下るとき(勾配の緩急により速度は異なる。以下同じ)→他方は同じ高さを下るような速度で登ればor下ればよい。
一方が、平地を歩いているとき→他方は静止していればよい

これじゃだめ?
しかしどうやって登る下るの連絡をとるんだwww

951 名前:132人目の素数さん:2009/10/03(土) 02:27:21
とっても長い糸電話で交信

952 名前:132人目の素数さん:2009/10/03(土) 06:48:42
           /\
          /    \/\
     /\/           \
    /                   \/\
  /                        \
/                            \
 ̄ ̄ ̄ ̄ ̄ ̄ ̄ ̄ ̄ ̄ ̄ ̄ ̄ ̄ ̄ ̄ ̄ ̄ ̄

953 名前:132人目の素数さん:2009/10/03(土) 10:17:54
>>950
そんな簡単にいくわけねーでや

>しかしどうやって登る下るの連絡をとるんだwww
可能性を論じているのであるからそれは無視。

954 名前:132人目の素数さん:2009/10/03(土) 11:39:31
                           /\
                         /     \
          /\          /        \/\
         /    \   /\/              \
       /       \/                    \
     /                                   \/\
    /                                         \
  /                                             \
 ̄ ̄ ̄ ̄ ̄ ̄ ̄ ̄ ̄ ̄ ̄ ̄ ̄ ̄ ̄ ̄ ̄ ̄ ̄ ̄ ̄ ̄ ̄ ̄ ̄ ̄ ̄ ̄ ̄ ̄ ̄ ̄ ̄ ̄

955 名前:132人目の素数さん:2009/10/03(土) 13:44:36

                                /\
                              /     \
               /\          /        \/\
                 /    \   /\/              \
               /       \/                    \
            /                                   \/\
     /\   /                                         \   /\
     /  \/                                            \/ \
     /   \
/ \                                               
 ̄ ̄ ̄ ̄ ̄ ̄ ̄ ̄ ̄ ̄ ̄ ̄ ̄ ̄ ̄ ̄ ̄ ̄ ̄ ̄ ̄ ̄ ̄ ̄ ̄ ̄ ̄ ̄ ̄ ̄ ̄ ̄ ̄ ̄ ̄ ̄ ̄ ̄ ̄ ̄ ̄ ̄ ̄ ̄ ̄


956 名前:132人目の素数さん:2009/10/03(土) 14:20:53
>>948
そういう関数だと、>945が本当に成り立つか怪しい気がするなぁ。

957 名前:132人目の素数さん:2009/10/03(土) 15:08:55
>>955
お前AAもまともに描けねーのかよ
それじゃあ途中でクレバスに落ちて死んぢまうw

>>956
当然一般には成立しねーよ
下りと登りと平坦場面が有限回あるだけと考えてくれ

958 名前:132人目の素数さん:2009/10/03(土) 17:01:22
>>957
一般に成立しないなら、反例が欲しいなぁ・・・。

959 名前:132人目の素数さん:2009/10/03(土) 18:08:17
4つの都市A、B、C、Dがあり、それらの都市は一辺をx[Km]とする正方形の頂点上に存在している。
ABCDすべての都市を相互に行き来できる道路を建設したい。
建設する道路をできるだけ短くしたとき、その道路の総距離をxを用いて表せ。

答えは分かるんですけど、答えの求め方がわかりません…

960 名前:132人目の素数さん:2009/10/03(土) 18:22:44
>>959
フェルマの原理を用いる。
正方形ABCDの中に点P,Qを取ってAP,BP,PQ,QC,QDなる線分を書く。
これらのなす角度が全部120°になると言うのがフェルマの原理。

961 名前:132人目の素数さん:2009/10/03(土) 18:53:34
>>960
>これらのなす角度が全部120°になると言うのがフェルマの原理。
これらのなす角度が全部120°になるときが最短、というのが・・・
ということを言いたいのだろうが、
>>959は「その答えはわかっているがなぜそうなるかがわからない」と言っているようにしか見えない。

962 名前:132人目の素数さん:2009/10/03(土) 19:29:52
>>959-960
おっとそれらの都市は地球上にあるのだったな。
それらの都市が赤道を4等分する地点にあるなら、
地下に穴を掘って地球の中心を通らなければならない。
出来るかな?

963 名前:132人目の素数さん:2009/10/03(土) 19:43:49
不景気だから人は集まるだろうが予算が足りない

964 名前:132人目の素数さん:2009/10/03(土) 21:37:07
>>945って成り立たないぞ
仮に真西・真東の平地の高さ(海抜)が違ったりするとそこからどっちかは下がれないからな
あるいは、どっちの平地も同じ高さだとしても
例えば真東の方は単調減少で平地(海抜0としよう)まで行き
真西に行く道に海抜0より低くなる場所が途中に存在すればそこから進めない


965 名前:132人目の素数さん:2009/10/03(土) 21:45:26
>>964
もちろん図にあるように平地の高さは同じとする。
>海抜0より低くなる場所
は存在しないとする。

966 名前:132人目の素数さん:2009/10/03(土) 22:54:52
>>959
 3点E,F,GからPまでの距離の和を EP + FP + GP = L(P) とおく。
 PをdP↑だけ微小変位させたときの L の変化は
 dL(P) = Grad{L(P)}・dP↑
   = ∇{L(P)}・dP↑
   = {EP↑/|EP| + FP↑/|FP| + GP↑/|GP|}・dP↑
   = (e + f + g)・dP↑
ここに e、f、g は単位ベクトル。
 (e・e) = (f・f) = (g・g) = 1,
 L(P) が最小のときは、どんなdP↑に対しても dL(P) =0,
∴ e + f + g = 0↑,
これと e、f、g との内積をとる。
∴ (e・f) = (f・g) = (g・e) = -1/2 = cos(120゚),
これは e,f,g が 120゚ をなし従って、EP、FP、GP が 120゚ をなすことを表わす。 >>960

967 名前:132人目の素数さん:2009/10/03(土) 23:55:32
>>965
駄目じゃんそういうこと問題文に書いてないと
問題文書きなおせカス

968 名前:132人目の素数さん:2009/10/04(日) 00:26:54
カーッ(゚Д゚≡゚д゚)、ペッ

969 名前:132人目の素数さん:2009/10/04(日) 07:46:09
>>966
駄目ぢゃんそういう物理的な解答したら

970 名前:132人目の素数さん:2009/10/04(日) 08:18:21
>>942-944とか、他にも上の方で幾つかあるけど、
出題者は回答者が回答のみレスした場合は途中の考え方も書いてほしいわ。
回答に正解レスしただけでは、何でその回答が正解なのかさっぱり分からん

971 名前:132人目の素数さん:2009/10/04(日) 09:07:50
>>967
そのくらい図から読み取れ

972 名前:132人目の素数さん:2009/10/04(日) 12:04:52
>>956
>当然一般には成立しねーよ
>下りと登りと平坦場面が有限回あるだけと考えてくれ

一般には成立しないなら反例を教えてくれ。

973 名前:132人目の素数さん:2009/10/04(日) 12:54:19
>>972
それは出題の範囲外。自分で考えよ。

974 名前:132人目の素数さん:2009/10/04(日) 13:05:18
>>973
いやいや、「 当 然 一般には成立しねーよ」と言ったのは
お前なんだから、当然、反例を知ってるんだろ?教えてくれよ。
それとも、自分でも反例知らないのにテキトーにレスしてたのかお前は。

975 名前:132人目の素数さん:2009/10/04(日) 13:20:15
>>974
自分で考えろ馬鹿

976 名前:132人目の素数さん:2009/10/04(日) 13:23:46
>>975
結局、お前も答えられないんだな。自分でも反例知らないくせに
>当然一般には成立しねーよ
こういうことを言っていたわけだ。ウソツキ!

977 名前:132人目の素数さん:2009/10/04(日) 14:02:32
ここ36時間ほど2ちゃんに張り付いているが、
このスレ>>967辺りからイチャモンばっかりだなw

978 名前:132人目の素数さん:2009/10/04(日) 14:24:53
平坦な部分は縮めて考える。
最初に谷にきたほうをaとして、もう1人のほうをbとする

まずbが(同時に)谷にこないとき
bは戻ることによって最初の高さと谷の高さの間を自由に移動できる
よってaは進んで再び谷の高さまで移動できる
ここで谷の数が有限なら何度か進めば下りになる。
このときbはaと同じ高さに来ていて二人とも下りで
aとbの谷の数の合計は1つ以上減っている。

979 名前:132人目の素数さん:2009/10/04(日) 14:31:36
180÷60+45×80=35

上記の式を4ヶ所だけ訂正して式を成立させなさい。


980 名前:132人目の素数さん:2009/10/04(日) 14:48:31
>>978
そんな単純な帰納法は通用しない。
後戻りする時も、一般にずっと先へ行ってからも
何度も山や谷を通って後戻りしなければならない。
aとbが同調して動けるか不明。

981 名前:132人目の素数さん:2009/10/04(日) 14:54:22
>>979
スレタイ嫁
もう少し面白い問題を書いて欲しい。
もっとも何が面白いかは主観にもよるが、ほかに書き込むべき適当なスレがいくらでもある。

982 名前:132人目の素数さん:2009/10/04(日) 14:57:51
同じ高さから始めて
はじめてa、bどちらかが
谷に来るまでのあいだで考えて
1度目が終わった時点で
最初と同じ状態に戻ってるので
繰り返しでいいと思いますがどうでしょうか。

983 名前:132人目の素数さん:2009/10/04(日) 16:39:07
直感的にも論理的にも不十分。
>同じ高さから始めて
どっちも登りだったらどうするんだ?


984 名前:132人目の素数さん:2009/10/04(日) 16:46:27
>>963
では、最短経路ではなくて、最もコストが低い経路を求めよ。

985 名前:132人目の素数さん:2009/10/04(日) 16:49:57
>>983
山頂で上りはないとおもうけど


986 名前:132人目の素数さん:2009/10/04(日) 18:10:43
>>977
イチャモンだあ?てめーなめてんのか?あぁ?
問題文ってのはもっと洗練されているべきなのだよ
あとからこーはだめあーはだめじゃ糞問なんだよ
てか最初からいろいろな可能性を考えた上で問題文作れ
満たすべき仮定や条件をきっちり書くこれ数学の常識だから
出直してこいやカス

987 名前:132人目の素数さん:2009/10/04(日) 19:42:41
次にa,bが同時に谷にきたとき
二人とも登っていって

(1)a,b同時に山にきたら、
aとbの谷の数の合計は2つ減っている。

(2)そうでないとき、aが先に山にきたほうとして
一旦、2人とも前の谷に戻ってaは山頂のほうへ戻り
bは麓のほうへ進んでいって、ひとつ山を越えた後
aのさっき来た山の高さまで降りることができる。
そのとき、そこが谷であれば、谷の数が有限であることから
さらに進んで下りになる。
このときbはaと同じ高さに来ていて二人とも下りで
aとbの谷の数の合計は1つ以上減っている。

988 名前:132人目の素数さん:2009/10/04(日) 20:06:10
チンピラみたいな口調の恫喝を目にすると吹き出してしまう

989 名前:987:2009/10/04(日) 20:16:29
ふもとに進む間に通る山の高さは前の山の高さを超えない
という条件が必要ですね。


全部 最新50
DAT2HTML 0.35d Converted.